Problem Book in Mathematical- Vol I Arithmetic

Page 1

លឹម សុ វណ្ណវិចិត្រ

កម្រងលំហាត់គណិតវិទ្យា កត្មិរវិទ្យាល័យ ភាគ ១- នព្វន ត

ផ្សាយល

ើកទ ី៣

សី ហា ២០១០


blank


លឹម សុ វណ្ណវិចិត្រ

កម្រងលំហាត់គណិតវិទ្យា កត្មិរវិទ្យាល័យ ភាគ ១- នពវន ត ផ្សាយល

ើកទ ី៣

សី ហា ២០១០ i


ភាគ១ - នព្វនត ផ្សាយល

ើកទ ី១ : គណិតវទាអូ ិ ឡាំព្ ិច -នព្វនត ២០០៨

ផ្សាយល

ើកទ ី២ : កម្រង

ាំ ហាត់គណិតវទា-នព្វ ិ នត ២០០៩

ភាគ២ - ព្ ីជគណិត វភាគ ិ

ដោយ

លឹ ម សុ វណ្ណវិចិត្រ

ទ្យំនាក់ ទ្យន ំ ង - វវបសាយ

http://www.dahlina.com/

- អុ ៊ីវមល

lsvichet@yahoo.com

ii


<<ប ើ ផលប ើ តបេញពី ុ ណ្យរ ស់ខ្ញមា ុំ នមែន ជាតិ បរោយសូ ែឱ្យខ្ញឆា ុំ ់ ែ ប ើ តបៅ ែព ញជា ែដ ងប ៀត>> (ត្ពះបាទ្យឧទ្យយាទ្យិរយទ្យ៊ីព៊ីរ, សវ. ទ្យ៊ី១១)

iii


iv


មាតិកា វផែកទ្យ៊ី១ ត្ទ្យឹស៊ីតបទ្យ ១. និយមន័យ ទ្រឹស្ដីបរ ................................................................................... 1 ១.១ និយមន័យ................................................................................... 1 ១.២ គោលការណ៍ ទ្រុ ងទ្ាប ................................................................ 3 ១.៣ លក្ខខណឌចាំបាច់ និងទ្រប់ ទ្ោន់ ....................................................... 4 ២. ភាពចចក្ដាច់ ............................................................................................. 7 ២.១ ភាពចចក្ដាច់ .............................................................................. 7 ២.២ និយមន័យ................................................................................... 7 ២.៣ ចាំនន ួ បឋម .................................................................................. 7 ២.៤ ទ្រឹស្ដីបរ ..................................................................................... 9 ២.៥ ទ្រឹស្ដីបរ ..................................................................................... 9 ិ ចី ចក្ចបបអឺរលដ ២.៦ វធ ី ........................................................................ 12 ២.៧ ភាពស្មមូ ល............................................................................... 14 ២.៨ ទ្រឹស្ដីបរ ..................................................................................... 14 ២.៩ ភាពចចក្ដាច់ នឹង ៩ ...................................................................... 16 ៣. ការបាំ ចបក្ជាក្ត្តាផលរុ ណ .......................................................................... 17 ៣.១ តួចចក្រួមធាំបាំផុត PGCD .............................................................. 17 ៣.២ ពហុ រុណរួមតូចបាំ ផុត PPCM ....................................................... 18 ៣.៣ ទ្រឹស្ដីបរ Bachet-Bézout ........................................................... 19 ៣.៤ ក្ូ រចូ លរអឺរលដ ី ............................................................................... 20 ៣.៥ ទ្រឹស្ដីបរ ..................................................................................... 20 v


៣.៦ ទ្រឹស្ដីបរ ..................................................................................... 22 ៣.៧ ទ្រឹស្ដីបរអឺរលដ ី ............................................................................. 23 ៣.៨ ទ្រឹស្ដីបរ..................................................................................... 23 ៣.៩ ទ្រឹស្ដីបរ ..................................................................................... 24 ៣.១០ ទ្រឹស្ប ដី រ .................................................................................. 24 ៣.១១ ទ្រឹស្ដីបរ ................................................................................. 25 ៣.១២ ទ្រឹស្ដីបរ ................................................................................. 25 ៣.១៣ ទ្រឹស្ដីបរ ................................................................................. 26 ិ ចី ចក្ចបបអឺរលដ ៣.១៤ ទ្បមាណវធ ី ........................................................ 28 ៣.១៥ ទ្រឹស្ដីបរ .................................................................................. 28 ៣.១៦ ទ្រឹស្ដីបរ.................................................................................. 28 ៣.១៧ ទ្រឹស្ប ដី រ ................................................................................. 30 ៤. ភាពស្មមូ លក្នង ុ ស្ាំ ណុាំចាំនន ួ រត់ .................................................................... 31 ៤.១ ស្មីការស្មមូ លលីគនចអ៊ែរ ............................................................ 31 ៤.២ ទ្រឹស្ដីបរ ..................................................................................... 31 ៤.៣ ទ្រឹស្ដីបរ ..................................................................................... 33 ៤.៤ ក្ូ រចូ លរ ........................................................................................ 33 ៤.៥ ទ្រឹស្ដីបរទ្បូ បិន ............................................................................ 34 ៤.៦ ទ្រឹស្ដីបរ...................................................................................... 36 ៤.៧ ទ្រឹស្ដីបរស្ាំ ណល់ ចចក្ចិន .............................................................. 36 ី ស្ុន .......................................................................... 37 ៤.៨ ទ្រឹស្ដីបរវល ៤.៩ ក្ូ នទ្រឹស្ដីបរចភមា ......................................................................... 39 vi


ិ ៤.១០ វបាក្ ........................................................................................ 40 ៥. ចផន ក្រត់ ...................................................................................................... 41 ៥.១ ចផន ក្រត់ ចផន ក្រស្ភារ .................................................................. 41 ៥.២ ទ្រឹស្ដីបរ ..................................................................................... 41 ៥.៣ ទ្រឹស្ដីបរ ..................................................................................... 43 ៥.4 ទ្រឹស្ដីបរឌីបូ លីញក្់ ...................................................................... 44 ៦. អនុរមន៍នពវ នា ............................................................................................. 47 ៦.១ អនុរមន៍នពវ នា.............................................................................. 47 ៦.២ ទ្រឹស្ដីបរ ..................................................................................... 48 ៦.៣ ទ្រឹស្ដីបរអឺចល ............................................................................. 52 ៦.៤ ទ្រឹស្ដីបរ...................................................................................... 53 ៦.៥ ទ្រឹស្ប ដី រ ...................................................................................... 54 ៦.៦ ទ្រឹស្ដីបរអឺចល ............................................................................. 55 ៦.៧ ទ្រឹស្ដីបរឡឺ ស្ង់ ........................................................................... 57 ៦.៨ ទ្រឹស្ដីបរខម ាំ ័រ ............................................................................. 58 ៦.៩ ទ្រឹស្ដីបរ ...................................................................................... 59

វផែកទ្យ៊ី២ លំហារ់ ឡូស្ុិច ........................................................................................................... 61 ទ្បព័នរធ បាប់ ចាំនន ួ រត់ ភាពចចក្ដាច់ .................................................................. 67 ទ្បព័នរធ បាប់ ......................................................................................... 67 ចាំនន ួ រត់ .............................................................................................. 68 ភាពចចក្ដាច់ ....................................................................................... 69 vii


ស្ាំ ណល់ ភាពស្មមូ ល .......................................................................... 72 ចាំនន ួ កាគរ ............................................................................................ 73 ចាំនន ួ ចដលមានរាងណាមួ យ ................................................................... 74 ពហុ រុណរួមតូចបាំ ផុត តួចចក្រួមធាំបាំផុត .............................................................. 77 ចាំនន ួ បឋម ........................................................................................... 77 ចាំនន ួ ពហុ រុណ ..................................................................................... 77 ពហុ រុណរួមតូចបាំ ផុត តួចចក្រួមធាំបាំផុត .................................................. 78 បាំ ចបក្ជាក្ត្តាបឋម .............................................................................. 79 ចផន ក្រត់ ........................................................................................................... 81 ស្មីការដយូផង់.................................................................................................. 83

វផែកទ្យ៊ី៣ ដំដ

ះត្សាយ

ឡូស្ុិច ........................................................................................................... 89 ទ្បព័នរធ បាប់ ចាំនន ួ រត់ ភាពចចក្ដាច់ .................................................................. 107 ទ្បព័នរធ បាប់ ......................................................................................... 107 ចាំនន ួ រត់ .............................................................................................. 112 ភាពចចក្ដាច់ ....................................................................................... 114 ស្ាំ ណល់ ភាពស្មមូ ល .......................................................................... 130 ចាំនន ួ កាគរ ............................................................................................ 131 ចាំនន ួ ចដលមានរាងណាមួ យ ................................................................... 138 ពហុ រុណរួមតូចបាំ ផុត តួចចក្រួមធាំបាំផុត .............................................................. 143 ចាំនន ួ បឋម ........................................................................................... 143 ចាំនន ួ ពហុ រុណ ..................................................................................... 145 viii


ពហុ រុណរួមតូចបាំ ផុត តួចចក្រួមធាំបាំផុត .................................................. 146 បាំ ចបក្ជាក្ត្តាបឋម .............................................................................. 152 ចផន ក្រត់ ........................................................................................................... 155 ស្មីការដយូផង់.................................................................................................. 163

ix


x


១ - និ យមន័ យ.ទ្រឹ ស្តីបរ និយមន័យ ចំនន ួ គរ់ ធមមជារិ ជាបណ្ត ា ចំនន ួ

ចំនន ួ គរ់ : ចំនន ួ គរ់ វជ្ ិ មា ជ ន ចំនន ួ គរ់ អវិជ្មា ជ ន ចំនន ួ គរ់ មិនអវិជ្មា ជ ន ចំនន ួ អសនិទាន ជាចំនន ួ ដែលមិនអាចសរសសរជារាង ចំនន ួ គរ់ គូ រឺចន ំ ន ួ គូ ជាចំនន ួ ដែលមានរាង

ដែល

បាន ដែល

ជាចំនន ួ គរ់ ។

ជាចំនន ួ គរ់ ។ ឧទាហរណ្៍

ជាចំនន ួ

គូ។ ចំនន ួ គរ់ សសស រឺចន ំ ន ួ សសស ជាចំនន ួ ដែលមានរាង

ដែល

ជាចំនន ួ គរ់ ។ ឧទាហរណ្៍

 ឧទាហរណ៍ : ប ើ

ជា២ចំនន ួ គត់ ដែល

យ ើ ងនឹងបង្ហាញថា យបើ េនមរថា ចំ នន ួ គរ់

គូ ប ោះ

រតូវដតមានលក្ខណៈគូ បេ​េែូចគ្នា។

ចសមលើយ

ររូវតរមានលក្ខណៈគូយេ​េផទុ គ្នា យ ោះ

ររូវតរមានលក្ខណៈគូយេ​េផទុ គ្នា។ យ ើ ងេនមរថា និង

តដល

និង

គូ យ ើ

យេ​េ។ ដូយចាោះ យ ើ ង យេ​េ។ ដូយចាោះមាន

។ ដូយចាោះ

ជាចំ នន ួ យេ​េ។

លឹម សុ វណ្ណវិចត្ិ រ | 1.1. និយមន័យ

1


ឧទាហរណ៍ :

ចូរ ង្ហាញថា

ជាចំនន ួ អេនិទាន។

ចសមលើយ យ ើ ងេនមរថា េំ យណើខាងយលើមិនព ិរ។ មានន័ ថា តដល

ជាចំ នន ួ េនិទាន ដូយចាោះ មានចំ នន ួ គរ់

។ ក្ាង ុ ចំ នន ួ គរ់ តដលអាចមានទាំងយ ោះ េនមរថា a, b រូចជាងយគ។

ររូវតរជាចំ នន ួ គូ។

តដល

ជាចំ នន ួ គរ់ មិនេូ នយ

  ដូយចាោះ

ររូវតរជាចំ នន ួ គូ។ តដល

និង

រូចបំ ផរ ុ យេ ផទុ ព ីេមមរក្ ិ មម។ មានន័ ថា

តដល

ជាចំ នន ួ គរ់ មិនេូ នយ។

មានន័ ថា

មិនតមនជាគូរំលល

ជាចំ នន ួ អេនិទាន។

ឧទាហរណ៍ : ចូរ ង្ហាញថា

ជាចំនន ួ គត់ គូ បហើ យថា ិ មា ចំប ោះចំនន ួ គត់ វជ្ ជ ន b និង

ចំប ោះចំនន ួ គត់

ចសមលើយ ក្រណី

គរ់ គូ ព ិរ ព ិរ។

េនមរថាព ិរ រ ូ រដល់

តដល

និង

ដូយចាោះ

ជាចំ នន ួ គរ់ វ ិជ្ជមាន។

យ ើ ងមាន 2

១ - និយមន័យ.ត្រឹសីា បរ | លឹម សុ វណ្ណវិចត្ិ រ


ជាចំ នន ួ គរ់ គ។ ូ យ ើ

ដូចគ្នា

ដូយចាោះេំ យនើខាងយលើ ព ិរ ចំ យ ោះ ចំ យ ោះចំ នន ួ គរ់ វ ិជ្ជមាន

និង ចំ យ ោះរគប់ ចំ នន ួ គរ់

សោលការណ្៍ត្រង ុ ត្ាប

រូបរេុងរ ប

សបើមានត្ាបចំនន ួ

សហើ យ ត្រុងមានរនធចន ំ ន ួ

ស ោះវាត្រវូ ដរមាន

ត្ាបយ៉ាងរិច២ ដែលឋិ រសៅក្នុងរនធដរមួយ។ សោលការណ្៍សនោះ សមើលសៅហាក្់ ែច ូ ជា ងាយណ្តស់ នរណ្តក្៏ ែង ឹ ដែរ ដរវាសំខាន់ ខាលំងណ្តស់ ។ ឧទាហរណ្៍ខាងសត្កាមនឹងបងា​ាញពីត្បសយជ្ន៍ របស់ វា។

លឹម សុ វណ្ណវិចត្ិ រ | 1.2. សោលការណ្៍ត្រុងត្ាប

3


 ឧទាហរណ៍ :

តាង

ើ បរ ើេបចញពី េវវីតពីជ្គណិ ត បរជ្េ

ជាេំ ណុំននចំនន ួ គត់

ង្ហាញថា រតូវដតមានចំនន ួ គត់

បសេងគ្នាដែលមានសល ូ ក្បេមើ

។ ចូរ

ចសមលើយ យ ើ ងតចក្ ធារុទាង ំ

របេ់ េវុី រយនោះ យៅជា

រក្ុម

។ យោ យ ើ ងររូវយរជ្ើេយរ ើេ ក្ចំ នន ួ គរ់ ចំនន ួ

យចញព ី េំ ណំ ុ ចំ នន ួ

ខាងយលើ យ ោះតាមយគ្នល

ការណ៍រេុងរ ប វាររូវតរមានចំ នន ួ គរ់ តដលឋិ រយៅក្ាង ុ េំ ណំ ុ តរមួ លនបណ្ត ា េំ ណំ ុ ខាងយលើ យ ើ

តដលមានផលបូក្

លក្ខខណ្ឌចំបាច់ នង ិ ត្គប់ ត្ោន់ បតើ ក្យថា លក្ខ ខណ្ឌចាំបាច់ និ ងគ្រប់ គ្ាន់ មានន័យថាប ៉េ ច?

លក្ខ ខណ្ឌចាំបាច់ ជាលក្ខខណឌដែលចំបាច់ បែ ើ

បីឱ្យេំ បណើ ួ យពិត។ ដតភាពចំបាច់ បនោះ ិនរបាក្ែ

ថារគ ់ រគ្នន់ បែ ើ បីឱ្យេំ បណើ ួ យប ោះពិតប ើ យ។  ឧទា

រណ៍ េំ យណើររីយកាណ

បុនររីយកាណ

។ យបើររីយកាណទាំងព ីរបុនគ្នាយ ោះជាចំបាច់ ររូវ

តរ មុំទាំងបីរបេ់ វាបុនគ្នា។ តរររីយកាណតដលមានមុំបីបុនគ្នា មិនរគប់ រគ្នន់ នង ឹ ឱ្យយ ើ ងេនាោ ិ ា នបាន ថាវាបុនគ្នាបានយេ។ តរយបើមំ ុទាំងបីរបេ់ វាមិនបុនគ្នា យ ោះររីយកាណព ីរយនោះក្៏មិនបុនគ្នាតដរ ដូយចាោះ យ ើ

បានជាយគនិយា ថា វាជាលក្ខខណឌចំបាច់ ។ លក្ខខណឌចំបាច់ មិនមានតរមួ យេ។ ក្រណីររី

យកាណបុនគ្នា យ ើ ងមានលក្ខខណឌចំបាច់ យរចើន៖ -

រជ្ុងមួ បុនគ្នា

-

រក្លាលផទបុនគ្នា

-

មុំមួ បុនគ្នា

-

…។ល។ 4

១ - និយមន័យ.ត្រឹសីា បរ | លឹម សុ វណ្ណវិចត្ិ រ


ដូយចាោះយ ើ ងយឃើ ញថា យបើររីយកាណព ីរបុនគ្នា យ ោះ ឱ្ ំ យបណ្ត ា លក្ខខណឌ « រជ្ុងមួ បុនគ្នា, រក្លា លផទបុនគ្នា, មុំមួ បុនគ្នា,.. » ព ិរទាំងអេ់ ។ លក្ខខណឌចំបាច់  ឧទា

រណ៍ េំ យណើ

យ ោះ

។ យ ើ ងទាញបានជាចំបាច់ មិនអាចយេមើ

លក្ខខណឌ

លក្ខខណឌរគប់ រគ្នន់

បានយេ។ ដូយចាោះ

មិនរគប់ រគ្នន់ ឱ្ ំ យ

។ ដូយចាោះ

ររូវតរធំ ជាងេូ នយយរ ោះយបើ ជាលក្ខខណឌចំបាច់ មួ ។ តរ

បានយេ។ យដើមបីឱ្យ

ជាលក្ខខណឌរគប់ រគ្នន់ យដើមបីឱ្យ

ទាល់ តរ

។ ក្ាង ុ ក្រណីយនោះ យ ើ ងមានលក្ខខណឌ

ចំបាច់ យរចើនរាប់ មិនអេ់ ដូចជា

ក្យគនលោះឺ បសេងប ៀត ដែលបយើ ងបរចើនជ្ួ រ

ោះមានែូចជា “

…បបើ និងមានតែបបើ …” និង

« ..នាំឱ្យ … និ ង គ្ចសមក្វិ ញ » ព ិរយបើនង ិ មានតរយបើ

ព ិរ។ ឃ្លលយនោះមានន័ ថា មានតរឬទាល់ តរ

ព ិរ តរបុយណ្តណោះ យេើប

អាចព ិរបាន។  ឧទា

រណ៍ដូចជា

ព ិរ យបើនង ិ មានតរយបើ

យផទៀងផ្ទទរ់ លក្ខខណឌយនោះបានយេ។ ឧទា មានតរយបើ ថា

បានយេ យរ ោះ យបើ

: គ្នមន

ណ្តយផេងយេៀរ តដល

រណ៍មួ យេៀរ យ ើ ងមិនអាចនិយា ថា ក្៏យផទៀងផ្ទទរ់

យបើនង ិ

បានតដរ។ តរយ ើ ងអាចនិយា

បាន។

«… យបើ និងមានតរយបើ … » = « …if and only if … » = «…si et seulement si …»  ឧទា

រណ៍

យ ើ

អងគទាង ំ ព ីរយេមើ គ្នា យបើនង ិ មានតរយបើ

មានន័ ថា គ្នមនលក្ខខណឌណ្តយផេងយេៀរ យរៅព ី ឱ្ ំ យ

ឧទា

។ ឃ្លលយនោះ

រណ៍

តដល

បានយេ។ លឹម សុ វណ្ណវិចត្ិ រ | 1.3. លក្ខខណ្ឌចំបាច់ និងត្គប់ ត្ោន់

5


ចមងល់៖ យរើអាចជ្ំនេ ួ ក្យ « យបើនង ិ មានតរយបើ » យោ

ក្យ « ទាល់ តរ-លុ ោះរតាតរ » បានតដរ

ឬយេ? ពិត

ឱ្ ំ យ

ិ ។ ឃ្លលបនោះមានន័យថា ប ើ ពិត និងរចេ ក្វញ

ិ ប ើ បហើ យរចេ ក្វញ  ឧទា

ពិត

រណ៍

ឧទា

ិ ។ ែូបចាោះវាមានន័យែូចគ្នានឹង ពិត វញ

ឱ្ ំ យ ឱ្ ំ យ

។ រចេមក្វ ិញ យបើ

យ ោះ

ឱ្ ំ យ

ដូយចាោះឃ្លលយនោះខុេររង់ «

ឱ្ ំ យ

និងរចេមក្វ ិញ» ។

 ឧទា

រណ៍ យ ើ ងមិនអាចនិយា ថា « ឱ្ ំ យ

6

ឱ្ ំ យ

និងរចេមក្វ ិញ » បានយេ យរ ោះ ក្៏យផទៀងផ្ទទរ់ លក្ខខណឌ

ឱ្ ំ យ

ឱ្ ំ យ

និងរចេមក្វ ិញ » បានយេ ក្៏យផទៀងផ្ទទរ់ លក្ខខណឌ

ឱ្ ំ យ

និងរចេមក្វ ិញ» ។

១ - និយមន័យ.ត្រឹសីា បរ | លឹម សុ វណ្ណវិចត្ិ រ

តដរ។

» ។ របយយាគតដលររូវ ររូវនិយា ថា «

តមន តររចេមក្វ ិញ

រ។ ដូយចាោះឃ្លលយនោះខុេររង់ «រចេមក្វ ិញ និយា ថា «

ដែរ។

ព ិរ។

តមន តរ

ឱ្ ំ យ

យរ ោះ

ពិតដែរ

និងរចេមក្វ ិញ។ យរ ោះ

រណ៍ យ ើ ងមិនអាចនិយា ថា »

រចេមក្វ ិញ

ពិត ប ោះ ឱ្ ំ យ

តដ

» ។ របយយាគតដលររូវ ររូវ


២ - ភាពចែកដាែ់ ភាពចចកដាច់ បបើ

ជាចំនន ួ គរ់ ប ើ ងនិយា ថា

ចែល

។ ប ើ ងសរបសរថា

ជា រួចចក របស់

ជា ពហុ គុណ្ នន

 ឧទាហរណ៍ : មាន

និង

ចែក

ចចកដាច់

បបើសិនជា មានចំនន ួ គរ់

មួ

ដាែ់ ។ យ ើ ងមាន

និង

ជាតួចែករបស់

។ តួចែករបស់

ខ្លន ួ វា។

និ មន័ ចំនន ួ កាបរ ជាចំនន ួ ចែលមានរាង ចំនន ួ គូប ជាចំនន ួ ចែលមានរាង  ឧទាហរណ៍ :

ចែល ចែល

ជាែំ នន ួ កាយរ យររោះ

ជាចំនន ួ គរ់ ។ ជាចំនន ួ គរ់ ។ ។

ជាែំ នន ួ កាយរយររោះ

ចំនន ួ បឋម ចំនន ួ បឋម

ជាចំនន ួ គរ់ វជ ិ មា ជ ន ធំជាង ១ ចែលរួចចករបស់ វាមានចរបលខ

និង

បបើចន ំ ន ួ គរ់

មិនចមនជាចំនន ួ បឋម ប ើ ងបៅវាថា ចំនន ួ ពហុ គណ្ ុ ។ ប ើ ងអាចសរ

បសរ

ចែល

ជា

លឹម សុ វណ្ណវិចត្ិ រ | 2.1. ភាពចចកដាច់

7


ឧទាហរណ៍ ែំ នន ួ បឋមៈ ែំ នន ួ ពហុ គុណ មិនមមនជាចំនន ួ បឋម ហ ើ យក៏មិនមមនជា ចំនន ួ ព ុ គុណមែរ។ ហយើ ងហ ើ ញថា

ជាចំនន ួ បឋមគូ មែមួ យគែ់ ហ ើ យ

និង

ជាចំនន ួ គែ់ ែហរៀងគ្នាមែមួ យគែ់ មែល

បឋមទំង២។  ឧទាហរណ្៍: ចូរកំណ្រ់ ចន ំ ន ួ គរ់ វជ ិ មា ជ ន

ចែល

ែំប

និង

ជាចំនន ួ បឋម។

ោះត្ា

ែំ នន ួ ទាំងបីបូកែូលគ្នន យសមើ នង ឹ

ជាយលខ្គូ។ ដូយែនោះកនង ុ

ែំ យោមយលខ្ទាំងបី រតូវមានយ៉ាងយោែោស់ មួ ចដលជាយលខ្គូ។ យ ើ ងដឹងថា មានយលខ្ មួ ប៉ា ុយោណោះ ចដលែំ នន ួ បឋមគូ។ យ ើ ងមាន មានចត

និង

ចត

ជាយលខ្យសស ដូយែនោះែំ នន ួ ចដលគួរចតយសមើ

យបើ

។ យបើ

យ ើ ងទាញបាន ែំ នន ួ ទាំងបីយសមើ នង ឹ

យបើ

។ យបើ

យ ើ ងទាញបាន ែំ នន ួ ទាំងបីយសមើ នង ឹ

យផទៀងផ្ទទត់ ។ មិនយផទៀងផ្ទទ

ត់ ។ ដូយែនោះ

។៚

 ឧទាហរណ្៍: [AHSME 1976] បបើ បផេងគ្នា ចូរគណ្នា

និង

និង

និង

ចដល

ោះត្ា

ជាឬសគត់ វ ិជ្ជមានព ីរយផេងគ្នន។ ដូយែនោះ និង

ដូយែនោះ

និង

និង

។៚ 8

មានឬសគរ់ ពីរ

ែំប តាង

ជាចំនន ួ បឋម និង

២ - ភាពចចកដាច់ | លឹម សុ វណ្ណវិចត្ិ រ

។ យដា ។ យដា

និង

ជាែំ នន ួ បឋម យ ោះ ជាែំ នន ួ បឋមទាំងព ីរ យ ោះ


ត្រឹសីតបរ 1° បបើ

ជាចំនន ួ គរ់ ចែល

បនាោះ

2° បបើ

ជាចំនន ួ គរ់ ចែល

បនាោះ

3° បបើ

និង

បនាោះ

4° បបើ

និង

បនាោះ

ចំប ោះត្គប់ ចន ំ ន ួ គរ់

ត្រឹសីតបរ ត្គប់ ចន ំ ន ួ គរ់ វជ ិ មា ជ នទាំងអស់ មានចំនន ួ រួចចកវិជមា ជ នជាចំនន ួ គូ បលើកចលងចរវាជាចំនន ួ កាបរ។

សត្មា បញ្ជជក់ តាង

ជាែំ នន ួ គត់ វ ិជ្ជមានមួ ។ យ ើ ងដឹងថា យបើ ។ ដូយែនោះ យបើ

ជាតួចែករបស់

មិនចមនជាែំ នន ួ កាយរ យ ើ ងមាន ចដល

និង

ករណី

ដាែ់ យ ោះ

យ ោះយ ើ ងមាន

ែំ យរោះរគប់

យររោះ

ក៏ជាតួចែករបស់

។ ករណី

ជាតួចែករបស់ ែំ នន ួ គត់

តាងយដា

ក៏ចែក

ដាែ់ ចដរ យររោះ ចដរ។ ករណី

ជាែំ នន ួ កាយរ យ ើ ងអាែរកបាន

ជាែំ នន ួ កាយរ។

ដូយែនោះតាមលកខណៈយនោះ យបើ យ ៀតរបស់

ចែក

ចដល

យ ោះ យគអាែរកបានតួចែកយផេង

មិនចមនជាែំ នន ួ កាយរ យ ើ ងមាន

យររោះយបើមាន

ជាែំ នន ួ កាយរ។ ដូយែនោះ ែំ នន ួ តួចែករបស់

យសមើ

យ ោះ

ជាែំ នន ួ គូ។ តួចែករបស់

មាន

។ ករណី

ជាែំ នន ួ កាយរ យ ើ ងអាែរកបាន មួ

ចដល

ជាែំ នន ួ កាយរ។ ដូយែនោះតួចែករបស់

យររោះយបើមាន

យ ោះ

មាន

លឹម សុ វណ្ណវិចត្ិ រ | 2.4. ត្រឹសីត បរ

9


។ យ ើ ងទាញបាន មានតួចែកែំ នន ួ

ជាែំ នន ួ យសស។

ឧទាហរណ៍ ែំ យរោះ

ជាែំ នន ួ កាយរ យរៅព ីយលខ្

យ ោះ យ ើ ងអាែរកបាន

ចដលជាតួចែករបស់

របស់

និង

ក៏ជាតួចែករបស់

របស់

ចដរ។ល។ យដា ផ្ំគ្ន ុ ន ជាគូៗ តួចែករបស់

ដូយែនោះែំ នន ួ តួចែករបស់ របស់

យសមើ

ចដរ។

មួ យែញ យបើ

ចដរ បានជានិែ។ ច ដូែជា

ជាតួចែកមួ របស់

ជាតួចែកមួ

និង

ក៏ជាតួចែក

មានទាំងអស់

ចដលជាែំ នន ួ យសស។

 ឧទាហរណ្៍: សិ សេបគចបរៀនចំនន ួ នមៃនាក់ បានបែើរឆលងការ់ រូដាក់ សមាៃរៈមួ អស់ បហើ

ជាតួចែករបស់

ជួរចែលបិរទា​ារជិរទាំង

បង់បលខពីបលខ១ ែល់ បលខ២០។ សិ សេរីមួ បបើកទា​ារទាំងអស់ បឡើង។ បនា​ាប់ មកសិ សេ

រីពីរបិរទា​ាររបស់ រូទាំងអស់ ចែលមានបលខ ២, ៤, ៦, ៨, ១០, ១២, ១៤, ១៦, ១៨, ២០។ សិ សេរីបីបបើក បិររូទាំងអស់ ចែលមានបលខ ៣ ៦ ៩ ១២ ១៥ ១៨ : បបើរូ

បិរ សិ សេបនាោះបបើក, បបើរូ

បបើក

សិ សេបនាោះបិរវិញ។ សិ សេបផេងបរៀរបធាើចបបបនោះបនតបនា​ាប់ មកបរៀរ បដា សិ សេរី បបើកបិររូ ចែលមានបលខជាពហុ គុណ្នន : បបើរូ

បិរ សិ សេបនាោះបបើក, បបើរូ

បបើក សិ សេបនាោះបិរវិញ។

បរើរូបលខបុនា​ានចែលនឹងចំហទា​ារ បត្កា បពលសិ សេទាំងនមៃនាក់ បនាោះបធាកា ើ របបើកបិរមដងមា​ាក់ រច ួ រាល់ បហើ

?

ែំប យ ើ ងដឹងថា

ោះត្ា

ូ ី នឹងែំ ហ យបើវារតូវយបើកបិ ែំ នន ួ យសសដង និងថា

ទាល់ ចត ជាតួចែករបស់

ូ ី រតូវយបើកបិ យដា សិ សេ ី

។ ដូយែនោះ ែំ នន ួ យបើកបិ យៅ ូ ី យសមើ នង ឹ ែំ នន ួ តួចែករបស់ យលខ្ ។

ឧទាហរណ៍ យូ លខ្៤ រតូវសិ សេ ១ ី យបើក សិ សេ ២ ី បិ សិ សេ ៤ ី យបើក ដូយែនោះែំ នន ួ យបើកបិ មាន៣ដ ង។ យូ លខ្៦ រតូវសិ សេ ១ ី យបើក សិ សេ ២ ី បិ សិ សេ ៣ ី យបើក សិ សេ ៦ ី បិ ដូយែនោះែំ នន ួ យបើកបិ មាន ៤ដង។

10

២ - ភាពចចកដាច់ | លឹម សុ វណ្ណវិចត្ិ រ


យ ើ ងដឹងយ ៀតថា ែំ នន ួ គត់ វ ិជ្ជមានចតមានែំ នន ួ តួចែកជាែំ នន ួ យសសគឺមានចតែំ នន ួ កាយរចតប៉ា ុយោណោះ។ ែំ នន ួ កាយរព ី ដល់

មាន

។ ដូយែនោះ យូ លខ្ ១ យលខ្៤ យលខ្៩ និងយលខ្១៦ រតូវយបើកបិ

ែំ នន ួ យសសដង ដូយែនោះជាែុងយរកា រតូវយៅែំ ហ។ ដូយែនោះ ចូ ដលែំ ហមានែំ នន ួ បួន៕៚

 ឧទាហរណ្៍: តាង ជាប់ គ្នា ; (b)

ជាចំនន ួ គរ់ ធជា ំ ង

។ ចូរបង្ហាញថា (a)

ជាផលបូកននចំនន ួ គរ់ បសសពីររ

ជាផលបូកននចំនន ួ គរ់ បីរគ្នា។

ែំប

ោះត្ា

(a) យ ើ ងមាន ដូយែនោះជាផលបូកននែំ នន ួ យសស

និង

។ ែំ នន ួ យសសព ីរយនោះ

ជាែំ នន ួ យសសតជាប់ គ្នន។ (b) យ ើ ងមាន ដូយែនោះជាផលបូកននែំ នន ួ គត់ បត ី ជាប់ គ្នន៕៚  ឧទាហរណ្៍: តាង បសសចំនន ួ

ជាចំនន ួ គូ ។ បរើបគអាចសរបសរបលខ

ជាផលបូកននចត្មាសរបស់ ចំនន ួ

បផេងគ្នាបានចែរឬបរ។

ែំប

ោះត្ា

យ ើ ងសនមតថា យគអាែសរយសរបាន

ចដល

ជាែំ នន ួ យសស។ កយនោមខាងយលើសមមូលនឹង

ចដល យ ើ ងមាន

។ ជាែំ នន ួ យសស និង

ននែំ នន ួ យសសែំ នន ួ គូដង(យររោះ

សុ ចធ តជាែំ នន ួ យសស។ យ ើ ងដឹងថា ផលបូក

ជាែំ នន ួ គូ) ឧទាហរណ៍ 2ដង 4ដង 6ដង។ល។ ជាែំ នន ួ គូ។ ដូយែនោះ

លឹម សុ វណ្ណវិចត្ិ រ | 2.5. ត្រឹសីត បរ

11


អងគខាងយវវងននសមភាពជាែំ នន ួ គូ យហើ

អងគខាងស្​្ំជាែំ នន ួ យសស ដូយែនោះអងគទាង ំ ព ីរមិនអាែយសមើ គ្នន

បានយ ៕៚  ឧទាហរណ្៍: ប

បត្ជើសបរើសបលខចំនន ួ ត្បាំបចញពីសំណ្ុំ

។ បបើប

ត្បាប់ បៅនារី

អំពីផលគុណ្ននចំនន ួ ទាំងត្បាំ បនាោះនារីមិនអាចកំណ្រ់ បានថាផលបូកននចំនន ួ ទាំងត្បាំបនោះគូឬបសសបាន បរ។ បរើផលគុណ្ននចំនន ួ ទាំងត្បាំចែលប

បានបត្ជើសបរើសបសាើ បុនា​ាន?

ែំប យ ើ ងមាន

ោះត្ា

។ ដូយែនោះ របាប់ ព ីផលគុណននែំ នន ួ គត់ របាំ ចដលយ

ដូែជារបាប់ ព ីផលគុណននែំ នន ួ ព ីរចដលយ

៉ាបានយរជ្ើសយរ ើស គឺ

៉ាមិនបានយរជ្ើសយរ ើសចដរ។ យបើស្គល់ ផលគុណននែំ នន ួ ព ីរ

ចដលយៅសល់ ធមមតា រ ីអាែទាញបានែំ នន ួ ទាំងព ីរយ ោះជាអវី រួែទាញបានែំ នន ួ ទាំងរបាំ ចដលយ យរជ្ើសយរ ើស

ឱ្ ំ យ រ ីអាែកំណត់ បានថាផលបូកននែំ នន ួ ទាំងរបាំ ថាគូឬយសសបាន។ ចតមានផលបូកព ីរ

ចដល រ ីមិនអាែកំណត់ បាន យ ោះគឺ ។ ចតមិនចមនជា យសស យហើ

ចដលអាែជា

យ យររោះយបើជា

និង

យ ោះ យ ើ ងមាន

យដា

ចដលអាែជា

យសស និង

គូ យ ោះ រ ីនឹងកំណត់ បានថាផលបូកនន

ែំ នន ួ ទាំងរបាំ ចដលយ

៉ាបានយរជ្ើសយរ ើសជាែំ នន ួ យសស។ ដូយែនោះផលគុណននែំ នន ួ ព ីរចដលយ

យរជ្ើសយរ ើសរតូវយសមើ

។ ដូយែនោះផលគុណននែំ នន ួ ទាំងរបាំ ចដលយ

បបើ

ជាចំនន ួ គរ់ វជ ិ មា ជ ន បនាោះ បគមានចំនន ួ គរ់

ឧទាហរណ៍៖ ែំ យរោះែំ នន ួ គត់

និង

យ ើ ងមាន

២ - ភាពចចកដាច់ | លឹម សុ វណ្ណវិចត្ិ រ

៉ាមិនបាន

៉ាបានយរជ្ើសយរ ើសរតូវចតយសមើ នង ឹ

2.6. ត្រឹសីតបរ- វិធចី ចកចបបអឺគែ លី

12

បា ៉ា ន

មួ

ចដល

គរ់ ចែល

ជាសំ ណល់ យហើ

និង


សត្មា បញ្ជជក់ ដំ បូងយ ើ ងបង្ហាញថា មាន

។ សំ យណើព ិត យដា យ ើ ង ក និង

ចដល

តាងឱ្យចផនកគត់ នន

; ឧទាហរណ៍

យ ើ ងរតូវបង្ហាញថា

។ យ ើ ងមាន

0  r  b។

។ យ ើ ងដឹងថា

ប ទ ប់ មកយ ៀត យ ើ ងបង្ហាញថា យគមាន

ចតមួ គត់ ។ យ ើ ងឧបមាថា យគអាែរកយ ើ ញ

ចដល

ដូយែនោះ

ចែក

ដាែ់ ចត

ដូយែនោះ មានចត

ឱ្ ំ យ

រួែយហើ

។ ិ មី ចកមបបអឺគលែ មួ យ តាមវធ ី ហគអាចមចកសំ ណុំចំនន ួ គែ់ ហៅ

គួ រកែ់ សម្គាល់ ថា ចំហ ោះចំនន ួ គែ់ ិ មី ចកនឹង តាមសំ ណល់ របស់ វធ

ឧទ រណ៍ ហធៀបនឹង , កាង ុ ចំហោមចំនន ួ គែ់ ទំងអស់ ហយើ ងអាចមចកជាចំនន ួ មែលម្គនរាង មែល

។ ឧទ រណ៍

ឧទ រណ៍ ហធៀបនឹង , កាង ុ ចំហោមចំនន ួ គែ់ ទំងអស់ ហយើ ងអាចមចកជាចំនន ួ មែលម្គនរាង មែល

។ ឧទ រណ៍

ឧទ រណ៍ ហធៀបនឹង , កាង ុ ចំហោមចំនន ួ គែ់ ទំងអស់ ហយើ ងអាចមចកជាចំនន ួ មែលម្គនរាង មែល

។ ឧទ រណ៍

។  ឧទាហរណ្៍: តាង

ជាចំនន ួ គរ់ វជ ិ មា ជ ន។ ចូរបង្ហាញថា

ចចកដាច់ នឹង

ចរចចកមិនដាច់ នឹង

ែំប

ោះត្ា លឹម សុ វណ្ណវិចត្ិ រ | 2.5. ត្រឹសីត បរ

13


យ ើ ងមាន

ជាែំ នន ួ យសស ដូយែនោះ

ជាែំ នន ួ គូ ដូយែនោះ ចែកដាែ់ នឹង

យ ើ ងមាន យ ើ ងមាន

តាមរូបមនតយ ធា វ ញត ុ ូ ន ក

និង

និង

យ ើ ងទាញបាន

តួនម ិ ួ ៗកនង ុ ផលបូកខាងយលើ យលើកចលងចតតួខាងែុង សុ ចធ តជាពហុ គុណនន ដាែ់ នឹង

។ យ ើ ងទាញបាន

ចែកនឹង

ែំ នន ួ ‹គត់ ។ ដូយែនោះ

សល់ សំ ណល់

ចែកនឹង

ដូយែនោះតួទាង ំ យនោះចែក

។ ដូយែនោះ

សល់ សំ ណល់

ចដល

៕៚

. ភាពសមមូល សរបសរថា មានន័ ថា

អានថា ចចកដាច់

ឬមានន័ មយងបរៀរថា

សមមូល

រឺ និង

ចែល

តាម(

) ។

ជាចំនន ួ គរ់ ។

មានសំណ្ល់ែច ូ គ្នា បពលចចកជាមួ

ឧទាហរណ៍ ឬ

។ ដូយែនោះ

. ត្រឹសីតបរ តាង

ចែល 1° 14

២ - ភាពចចកដាច់ | លឹម សុ វណ្ណវិចត្ិ រ

និង

បនាោះ

ជា


2° 3° 4° 5°

បបើ

ជាពហុ ធាមានបមគុណ្ជាចំនន ួ គរ់ បនាោះ

សត្មា បញ្ជជក់ យដា

និង និង

យ ោះ យគអាែរកបាន

ចដល

។ ដូយែនោះ

ដូយែនោះយ ើ ងទាញបានលកខណៈយលខ្ 1 ដល់ យលខ្ 3 ។ ក

ជ្ំនស ួ ែូល កនង ុ យលខ្ 3 យ ើ ងទាញបាន

យដើមបីរស្ បញ្ជជក់ លកខណៈយលខ្

តាង

តាមលកខណៈយលខ្ 4 យ ើ ងទាញបាន យដា

តាមលកខណៈយលខ្ 3 យ ើ ងទាញ

បាន យហើ

តាមលកខណៈយលខ្ 1 យ ើ ងទាញបាន

ឧទាហរណ្៍:

ចូរគណ្នា រនមលសមមូលរបស់ ចំនន ួ កាបរ តាម 13 ។

ចបមលើ សំ នួរគឺែង់ បាន យ ើ ងមាន

។ ។

លឹម សុ វណ្ណវិចត្ិ រ | 2.8. ត្រឹសីត បរ

15


ដូយែនោះ ែំ នន ួ កាយរ សមមូលនឹង

. ត្រឹសីតបរ-ភាពចចកដាច់ នង ឹ ៩ ចំនន ួ គរ់ ធមាជារិ ចចកដាច់ នឹង

មួ

ចចកដាច់ នឹង បបើផលបូកននបលខត្គប់ ខាងទា ់ ំងអស់ បញ្ចូលគ្នា

សត្មា បញ្ជជក់ តាង

យដា ដូយែនោះយបើផលបូកតួយលខ្របស់

(យសមើ នង ឹ

រ។

16

២ - ភាពចចកដាច់ | លឹម សុ វណ្ណវិចត្ិ រ

)ចែកដាែ់ នឹង

យ ោះ

ចែកដាែ់ នឹង

ចដ


៣ - ការបំ បបកជាកត្ត ា ផលគុណ 3.1. រួចចករួមធំបផ ំ រ ុ  ឧទាហរណ្៍ គណ្នារួចចករួមនៃ

ៃិង

យ ើ ងមាន

ដូយចនេះតួចចករបស់ វាមាន

វាមាន

។ ដូយចនេះ

យនេះ តួចចករួមចដលធំ ជាងយគគឺ

និង

និង

មានតួចចករួម

ដូយចនេះតួចចករបស់ ។ កនង ុ ចំ យោមតួចចករួមទំង

។ យ ើ ងនិយា ថា តួចចករួមធំ បំផត ុ របស់

និង

យសមើ

ៃិយមៃ័យ បបើ

មិៃសូ ៃយទាំង២រពមគ្នា បនាោះ ចំៃ​ៃ ួ គរ់ ធប ំ ផ ំ រ ុ ចែលចចក

ថា រួចចករួមធំបផ ំ រ ុ របស់ បគតាងបដាយ

ដាច់ ទាំងពីរ បៅ

ៃិង ។

រឺបដាយ

លកខណ្ៈ បបើ

ៃិង

បនាោះ

 ឧទាហរណ្៍ គណ្នា PGCD នៃ

ៃិង

ៃិង

យ ើ ងមាន

 ឧទាហរណ្៍ គណ្នា PGCD នៃ

លឹម សុ វណ្ណវិចរិ រ | 3.1. រួចចករួមធំបផ ំ រ ុ

17


យ ើ ងមាន

យ ើ ងយឃើ ញថា

លកខណ្ៈ បបើ ែូបចាោះ បបើ

បនាោះបគៃិយាយថា ៃិង

ៃិង

បឋមៃឹងគ្នា។

បឋមៃឹងគ្នា បនាោះចំៃ​ៃ ួ ២បៃោះមិៃអាចមាៃកតា​ារួមធំជាង ១បេ។

3.2. ពហុ គណ្ ុ រួមរូចបំផរ ុ  ឧទាហរណ្៍ គណ្នាពហុ គុណ្រួមនៃ យ ើ ងមាន ចំ នន ួ ចដលជាពហុ គុណនន

ៃិង

ផងនិងនន

មាន

។ ចំ នន ួ ទំងយនេះយៅថា ចំ នន ួ ពហុ គុណរួមនន យរចើនរាប់ មិនអស់ ។ ចតចំ នន ួ ចដលតូចជាងយគ គឺ ដូយចនេះ

ជាពហុ គុណរួមតូចបំ ផត ុ រវាង

និង

និង

។ ចំ នន ួ ចបបយនេះមាន

។ ។

ៃិយមៃ័យ បបើ ផងៃិង

មិៃសូ ៃយទាំង២រពមគ្នា បនាោះ ចំៃ​ៃ ួ គរ់ វជ្ ិ មា ជ ៃរូចបំផរ ុ ចែលជាពហុ គុណ្នៃ ផង បៅថា ពហុ គុណ្រួមរូចបំផរ ុ នៃ

បគតាងបដាយ

រឺ

 ឧទាហរណ្៍ គណ្នាពហុ គុណ្រួមរូចបំផរ ុ នៃ យ ើ ងមាន

18

៣ - ការបំចបកជាកតា​ាផលគុណ្ | លឹម សុ វណ្ណវិចរិ រ

ៃិង

ៃិង


លកខណ្ៈ បយើ ងបឃើ ញថា បបើ

ៃិង

បនាោះ

3.3. រេឹសីាបេ Bachet-Bézout រួចចករួមធំបផ ំ រ ុ របស់ រគប់ ចៃ ំ ៃ ួ គរ់ ពីរ តាងបដាយ ៃិង

បាៃជាៃិចច មាៃ​ៃ័យថាមាៃចំៃ​ៃ ួ គរ់ ពីរតាងបដាយ

អាចសរបសរជាបៃសំលបី ៃច ៊ែ រនៃ ចែល

សរមាយបញ្ជជក់  ឧទហរណ៍

។ យ ើ ងមាន

 ឧទហរណ៍

។ យ ើ ងមាន

 ឧទហរណ៍

យររេះ (

មិនអាចសូ នយទំងព ីរ។

មានធាតុតច ូ បំ ផត ុ តាងយោ

។ យ ើ ងនឹងបង្ហាញថា ជាតួចចករួមរបស់

បំ ផត ុ របស់

)និងថា យបើ

ជាធាតុ

។ ដូយចនេះ យគមាន

។ ដូយចនេះ យ ើ ងរតូវបង្ហាញថា និង

យ េះ

(

ជាតួចចករួមធំ

)។

ជាដំ បូងយ ើ ងនឹងបង្ហាញថា ចដល

យោ

។ យ ើ ងដឹងថា មានមួ កនង ុ ចំ យោម ចដល

យបើ

។ យ ើ ងមាន

តាង របស់

យហើ

យ េះ

។ តាមរបមាណវ ិធីចចកចបបអឺ គដ ល ី យ ើ ងអាចរកបានចំ នន ួ គត់

ចដល

។ យោ

យ េះ វាមានតំនលតូចជាងធាតុតច ូ ជាងយគរបស់

ដូយចនេះវាផទុ ព ីការសនមត។ ដូយចនេះ

ចដលតាង

លឹម សុ វណ្ណវិចរិ រ | 3.3. រេឹសីា បេ Bachet-Bézout

19


ដូចគ្នន យ ើ ងអាចបង្ហាញថា

សនមតថា

ចំ យរេះចំ នន ួ គត់

មានន័ ថា

។ ដូយចនេះ

។ ដូយចនេះ ររឹសតីបររតូវបានរា បញ្ជាក់ ។

សមា​ាល់ បយើ ងអាចចចងរេឹសីដ បេបៃោះតាមមួយចបបបេៀរថា « រគប់ បៃសំលបី ៃច ៊ែ រនៃ

ចចកដាច់ ៃឹង

។»

3.4. កូរចូ លរ ឺគែ លី បបើ

ចចកដាច់

ៃិង បបើ

បនាោះ

ចចកដាច់

សរមាយបញ្ជជក់ យោ

យ េះ តាមររឹសតីបរ Bachet-Bézout យគមានចំ នន ួ គត់ ។ យោ

ដូយចនេះ យ ើ ងទញបាន

យ េះ យគមានចំ នន ួ គត់

មួ

ចដល

ចដល

។ យ េះ

3.5 រេឹសីាបេ 1° បបើ 2° បបើ

បនាោះ

ជាចំៃ​ៃ ួ គរ់ វជ្ ិ មា ជ ៃ បនាោះ

3° ចំប ោះចំៃ​ៃ ួ គរ់ មិៃសូ ៃយ

បគមាៃ

4° ចំប ោះចំៃ​ៃ ួ គរ់ មិៃសូ ៃយ

បគមាៃ

20

៣ - ការបំចបកជាកតា​ាផលគុណ្ | លឹម សុ វណ្ណវិចរិ រ


សរមាយបញ្ជជក់ 1° តាមររឹសតីបរ Bachet-Bézout យគមានចំ នន ួ គត់ ។ យ ើ ងមាន រគប់ បនសំលយី នចអ៊ែ រនន

និង និង

ចចក

ចដល

។ ដូយចនេះ

ជាចំ នន ួ គត់ ។ យ ើ ងដឹងថា

។ យោ មានបនសំលយី នចអ៊ែ រមួ

ចចកោច់ គឺ

ោច់ ។ យ ើ ងទញបាន

ចដលយសមើ

យ េះ

និង

2° តាង

និង

។ យ ើ ងបង្ហាញថា

យ ើ ងមាន

ដូយចនេះតាមររឹសតីបរ Bachet-Bézout យគអាចរកបានចំ នន ួ គត់

ចដល

។ តាមររឹសតីបរ Bachet-Bézout ដចដល យគមាន

រគប់ បនសំលយី នចអ៊ែ ររវាង

និង

ចចកោច់ នឹង

។ ដូយចនេះ

តាមររឹសតីបរ Bachet-Bézout យគអាចរកបានចំ នន ួ គត់ ។ ចត ចំ នន ួ គត់

ជាបនសំលយី នចអ៊ែ រនន

ចដល

ចដល

យហើ

។ ដូយចនេះ

ដូយចនេះ វាចចកោច់ នឹង

មានន័ ថា

។ យគមាន

ចំណ៖ំ ដូចគ្នន យ ើ ងទញបាន

ចំ យរេះរគប់ ចំ នន ួ គត់ មិនសូ នយ

3° យោ

ចចកោច់ ចចកោច់

មយ៉ាងវ ិញយរៀត

ចចកោច់ ចចកោច់ យហើ

4° សនមតថា

និង និង

យ េះ វាចចកោច់ យហើ

។ ដូយចនេះ

ដូយចនេះ

យ េះ វាចចកោច់

និង

។ ដូយចនេះ

ដូយចនេះ វាចចកោច់

។ ដូយចនេះ ចចកោច់

និង

។ តាមររឹសតីបរ 3° យ ើ ងទញបាន

លឹម សុ វណ្ណវិចរិ រ | 3.5 រេឹសីា បេ

21


យោ

យ េះ

តាមររឹសតីបរ3° ដចដល យ ើ ងទញបាន ដូយចនេះ

តាមររឹសតីបរ 1°

ដូយចនេះ

តាមររឹសតីបរ 2° យោ

យ ើ ងទញបាន

3.6 រេឹសីាបេ បបើ

បនាោះ

ចចកដាច់ យាងបោចណស់ ៃឹងចំៃ​ៃ ួ បឋមមួយ។

សរមាយបញ្ជជក់ យោ

យ េះវាមានយា៉ាងយោចោស់ តួចចកមួ ធំ ជាងមួ ។ ដូយចនេះ វាមានតួចចកមួ ចដល

តូចជាងយគ យហើ

22

ធំ ជាងមួ

តាងយោ

។ យ ើ ងថា

៣ - ការបំចបកជាកតា​ាផលគុណ្ | លឹម សុ វណ្ណវិចរិ រ

ជាចំ នន ួ បឋម។ យបើ

មិនចមនជាចំ នន ួ


បឋមយរ យ េះ យ ើ ងអាចសរយសរ ចដលធំ ជាងមួ

និងតូចជាង

ជា

។ ចតយបើដយូ ចនេះ

ចដលផទុ នឹងការសនមត ចដលថា

ជាតួចចករបស់

តូចជាងយគ។

3.7 រេឹសីាបេ ឺគែ លី សំណ្ុំចំៃ​ៃ ួ បឋម ជាសំណ្ុំ ៃ​ៃា។

សរមាយបញ្ជជក់ សនមតថា សំ ណំ ុ ចំ នន ួ បឋមជាសំ ណំ ុ កំណត់ ចដលមានចំ នន ួ បឋមចត ចំ នន ួ គត់ ទង ំ អស់ ចដលធំ ជាងមួ

។ យ ើ ងដឹងថា រគប់

សុ រចធ តចចកោច់ នឹងចំ នន ួ បឋមយា៉ាងយោចោស់ មួ

(ររឹសតីបរ

3.6)។ ដូយចនេះ រគប់ ចំ នន ួ គត់ ទង ំ អស់ រតូវចតចចកោច់ នឹងយា៉ាងយោចោស់ ចំ នន ួ បឋមមួ កនង ុ ចំ យោម

។ យ ើ ងព ិនិតយចំ នន ួ គត់ ធំជាងមួ

បឋមមួ កនង ុ ចំ យោម ោច់ យ េះ

រតូវចតចចក

ចំ នន ួ បឋម ដូយចនេះ

ចដលចចក ោច់ យររេះ

។ រតូវចតមានចំ នន ួ

ោច់ ។ ចតយបើមាន ចចកោច់ នឹង

។ ដូយចនេះ

មួ

ចចក ចត

ជា

។ ដូយចនេះមិនអាច។ យ ើ ងទញបាន សំ ណំ ុ ចំ នន ួ បឋមមិនអាចជាសំ ណំ ុ

កំណត់ បានយរ៕៚

3.8 រេឹសីាបេ ផលគុណ្នៃចំៃ​ៃ ួ គរ់ ពីរចែលមាៃរាង

ក៏ ជាចំៃ​ៃ ួ ចែលមាៃរាង

ចែរ។

សរមាយបញ្ជជក់ យ ើ ងមាន ដូយចនេះសំ យណើព ិត៕៚

លឹម សុ វណ្ណវិចរិ រ | 3.7 រេឹសីា បេ ឺគែ លី

23


3.9 រេឹសីាបេ ចំៃ​ៃ ួ បឋមចែលមាៃរាង

មាៃបរចើៃរាប់ មិៃ ស់ ។

សរមាយបញ្ជជក់ យ ើ ងនឹងរា បញ្ជាក់ តាមវ ិធីផ ទុ យោ

យោ សនមតថាចំ នន ួ បឋមមានរាង ។ យ ើ ងព ិនិតយចំ នន ួ

តាង

ជាតួចចកបឋមមួ របស់

 មិនអាចមានរាង  យបើ

អាចមានរាង

យរ យររេះមិនចមនជាចំ នន ួ បឋម។

: យ េះ

ចំ យោម

ជាចំ នន ួ បឋមមួ ចដលមានរាង (យររេះសំ ណំ ុ ចំ នន ួ បឋមចដលមានរាង

ប៉ា ុយោណេះ)។ ចតកនង ុ ករណីយនេះ យបើ ចំ នន ួ បឋម ចដលធំ ជាង អាចជាតួចចករបស់

មានចំ នន ួ កំណត់ តាង

យ េះ

ជាចំ នន ួ បឋមមួ កនង ុ មានចត

ចត យររេះ

។ ចតយបើ

យ េះ

សុ រចធ តជា

។ ដូយចនេះ

មិន

បានយរ ផទុ ព ីការសនមត។

ដូយចនេះមានចត

។ ដូយចនេះតួចចករបស់

គុណននបោ ត យលខចដលមានរាង

, ដូយចនេះ

សុ រចធ តមានរាង

។ ដូយចនេះ

រតូវចតមានរាង

ចដរ។ យ ើ ងទញបាន

ចតមិនអាចយររេះ ទញបានថា ចំ នន ួ បឋមចដលមានរាង

យសមើ នង ឹ ផល

។ ដូយចនេះ យ ើ ង រតូវចតមានចំ នន ួ យរចើនមិនកំណត់ ៕៚

3.10 រេឹសីាបេ បបើចៃ ំ ៃ ួ គរ់ វជ្ ិ មា ជ ៃ

ជាចំៃ​ៃ ួ ពហុ គុណ្ បនាោះ វាមាៃកតា​ាបឋម

សរមាយបញ្ជជក់

24

៣ - ការបំចបកជាកតា​ាផលគុណ្ | លឹម សុ វណ្ណវិចរិ រ

មួយ ចែល


សនមតថា

។ យបើ មិនព ិត។ ដូយចនេះ

មួ

ចដល

និង

សុ រចធ តធំ ជាង

រតូវមានកតាតមួ ខុសព ី និង

ទំង២ យ េះ ។ ដូយចនេះវារតូវមានកតាតបឋម

៕៚

3.11 រេឹសីាបេ បបើ

ជាចំៃ​ៃ ួ បឋម បនាោះ

ចចកដាច់ ៃឹង

ចំប ោះរគប់

សរមាយបញ្ជជក់ យ ើ ងមាន

ដូយចនេះ

ឱ្ ំ យ

។ យោ

យ េះ

ចចកមិនោច់

។ តាមកូរចូ ៉ា លរអឺ គដ ល ី យ េះ មានចត

៕៚

3.12 រេឹសីាបេ រគប់ ចៃ ំ ៃ ួ គរ់ ទាំង ស់ សុ េធចរអាចបំចបកជាផលគុណ្ ចែលមាៃកតា​ាទាំង ស់ ជាចំៃ​ៃ ួ បឋម។

សរមាយបញ្ជជក់ ឧទហរណ៍

ជាផលគុណនន

ផលគុណរបស់

ចដលសុ រចធ តជាចំ នន ួ បឋម ឬកតា​ា

សុ រចធ តជាចំ នន ួ បឋម។

យ ើ ងនឹងបង្ហាញថា រគប់ ចំ នន ួ គត់ ទង ំ អស់ សុ រចធ តអាចបំ ចបកជាផលគុណដូចខាងយលើបានទំងអស់ ។ យ ើ ងព ិនិតយចំ នន ួ គត់ កករណី

មួ ។

ជាចំ នន ួ បឋម យ េះ

សំ យណើព ិត។

លឹម សុ វណ្ណវិចរិ រ | 3.11 រេឹសីា បេ

25


ខករណី

មិនចមនជាចំ នន ួ បឋម ដូយចនេះយា៉ាងយោចោស់ មានតួចចកបឋម

មួ ចដល

ចដល យ ើ ងមាន២ករណីយរៀត ១ករណី

ជាចំ នន ួ បឋម យ េះ

២ករណី

មិនចមនជាចំ នន ួ បឋម។ ចដល

ជាផលគុណននចំ នន ួ បឋម២។ មានតួចចកបឋម

ដូយចនេះ

មានចំ នន ួ កំណត់ យ េះ មាន

អាចសរយសរជា

និង

តាមវ ិចារដូចគ្នន យ ើ ងទញបាន កតាតបឋមរបស់ ចចករបស់

មួ យា៉ាងតិចចដល

មួ

ចដល ចដល

។ យោ ចំ នន ួ តួ ជាចំ នន ួ បឋម យហើ

ចដល

យសមើ

សុ រចធ តជាចំ នន ួ បឋម (មានខលេះខុសគ្នន

ខលេះយសមើ គ្នន)៕៚ យ ើ ងអាចយ ៀបកត្តាបឋមទាំងយ ោះជារាង

ដែល

ជាបណ្ត ា ចាំនន ួ បឋមខុសៗគ្នា។ យ ើ ងយៅកា បាំ ដបកជាផលគុ ណកត្តាបឋមនន

កត្តាផលគុ ណរាងកាណូនិ ចនន

។ ឧទហ ណ៍

ខាងយលើ ថា

ជា កត្តាផលគុ ណរាងកាណូនិចនន

3.13 រេឹសីាបេរគឹោះនៃ​ៃពវៃ ា រគប់ ចៃ ំ ៃ ួ គរ់ ធជា ំ ងមួយ អាចបំចបកជាកតា​ាផលគុណ្កាណ្ូៃិចបាៃចរមួយចបបគរ់ ។ ឬ ៃិយាយមាងបេៀរថា ចំប ោះរគប់ ចៃ ំ ៃ ួ គរ់

បគមាៃចំៃ​ៃ ួ បឋម ចែលខុសគ្នាពីរៗ

ចរមួយចបបគរ់ ៃិងចំៃ​ៃ ួ គរ់ វជ្ ិ មា ជ ៃ

26

៣ - ការបំចបកជាកតា​ាផលគុណ្ | លឹម សុ វណ្ណវិចរិ រ

ចរមួយចបបគរ់ ចែល


សរមាយបញ្ជជក់ ឧបមាថាវាអាចបំ ចបកបាន២យា៉ាង និង ចដល

ជាចំ នន ួ បឋមខុសគ្ននព ីរៗ និង

ជាចំ នន ួ បឋមខុសគ្ននព ីរៗ។ យ ើ ង

មាន យោ

។ ចត

មានមួ កនង ុ ចំ យោម ទញបាន

សុ រចធ តជាចំ នន ួ បឋម យ េះរតូវ

ចដលយសមើ នង ឹ

និង

។ សនមតថា

។ ដូយចនេះ

។ តាមរយបៀបដូចគ្ននយ ើ ង

អាចោក់ ជាផលគុណកតាតបឋមបានចតមួ

ចបបគត់ ៕៚

ររឹសាីបរខាងយលើ

ឱ្ ាំ យយ ើ ងទញបានរាំ

ក់ រាំនងសរាប់ គណ

និង

ននចាំនន ួ

គត់ ២យបើសិនជាយ ើ ងស្គាល់ ផលគុ ណកត្តាបឋម បស់ វា។ ត្តង

ដែលកាង ុ យ ោះ បណ្ត ា

ខុសគ្នា២ៗ ដតបណ្ត ា

គួ កត់ សា​ាល់ ថា ចាំយ ោះចាំនន ួ ពិត

និង

និង

អាចានខល ោះយសមើសូនយ។ យ ើ ងាន

យ ើ ងាន

ឱ្ ាំ យយ ើ ងទញបាន

លឹម សុ វណ្ណវិចរិ រ | 3.13 រេឹសីា បេរគឹោះនៃ​ៃពវៃ ា

27


3.14 របមាណ្វិធចី ចកចបប ឺគែ លី តាង

ជាចំៃ​ៃ ួ គរ់ វជ្ ិ មា ជ ៃ។ បនា​ាប់ បធវរើ បមាណ្វិធីចចកបៃាបនា​ាប់ គ្នា បយើ ងទាញបាៃបស៊ែ រនី ៃ

វិសមភាពែូចរបៅ៖

បស៊ែ រនី ៃសំណ្ល់ ៃឹងបៅែល់

ចែលៃឹងបសមើ សូៃយ។ បរ ោះថា

ណ ា ចំៃ​ៃ ួ គរ់ ែូបចាោះ សវ៊ែ៊ី របៃោះ មិៃអាចមាៃចំៃ​ៃ ួ រួវជ្ ិ មា ជ ៃបលើសពី

ជាសវ៊ែ៊ី រចុោះនៃប បេ។

3.15 រេឹសីាបេ បបើ

ជាចំៃ​ៃ ួ គរ់ វជ្ ិ មា ជ ៃ បនាោះ

សរមាយបញ្ជជក់ តាង

និង

ដូយចនេះ

ជាតួចចករួមរបស់

មយ៉ាងវ ិញយរៀត មានន័ ថា

។ យោ និង និង

។ ដូយចនេះ

។ ដូយចនេះ

។ ដូយចនេះ ឱ្ ំ យ

យ េះ ជាតួចចករួមនន

3.16 រេឹសីាបេ បបើ

28

ជាសំណ្ល់ ចុងបរកាយ មុៃសូ ៃយ បៅកាុងរបមាណ្វិធចី ចក ឺគលីែ បនាោះ

៣ - ការបំចបកជាកតា​ាផលគុណ្ | លឹម សុ វណ្ណវិចរិ រ

យ េះ និង

និង

និង


សរមាយបញ្ជជក់ យ ើ ងមាន

តាង

។ សមីការរីមួ

ចតយោ យចញព ីសមីការចុងយរកា

ដូយចនេះ

ជាតួចចករួមរបស់

ដូយចនេះ

និង

យហើ

ឱ្ ំ យ

។ រីព ីរ ឱ្ ំ យ

។ បនតប ទ ប់ មកយរៀត

បកយៅយលើវ ិញ យ ើ ងទញបាន

ដូយចនេះ

ឬក៏

៕៚

ឧទាហរណ្៍:

គណ្នា

ចបមលើយ យ ើ ងមាន

សំ ណល់ មុនសូ នយគឺ

ដូយចនេះ

លឹម សុ វណ្ណវិចរិ រ | 3.16 រេឹសីា បេ

29


3.17 រេឹសីាបេ សៃមរថា

ជាចំៃ​ៃ ួ គរ់ ចែល

ការែយូផង់*

។ បបើបគស្គាល់ គូចបមលើយ

នៃសមី

បនាោះចបមលើយបផសងបេៀរនៃសមីការបៃោះ ៃឹងមាៃរាង

ចែល

ៃិង

*សមីការែយូផង់ជាសមីការចែលមាៃឫសឋិ រកាុងសំណ្ុំចំៃ​ៃ ួ គរ់ ។

សរមាយបញ្ជជក់ យ ើ ងថា យបើ

ជាឫសរបស់ សមីការ

យ េះ

ក៏ជា

ឫសចដរ។ ចតយ ើ ងនឹងបង្ហាញថា រគប់ ចយមលើ ននសមីការ នឹងមានរាងចបបយនេះទំងអស់ ។ សនមតថា

យផទៀងផ្ទទត់

។ ដូចគ្ននចដរ

ចចកអងគទង ំ ២នឹង

យោ គត់

យ ើ ងទញបាន

យ េះ ចដល

30

។ ដូយចនេះយ ើ ងមាន

ឱ្ ំ យ

យោ យយាងតាមកូរចូ ៉ា លរអឺ គដ ល ី ។ ដូយចនេះយគមានចំ នន ួ ។ យចញព ីយនេះ យ ើ ងទញបាន

៣ - ការបំចបកជាកតា​ាផលគុណ្ | លឹម សុ វណ្ណវិចរិ រ


៤ - ភាពសមមូ លក្ន ុងសំ ណំ ចំ នន ួ គត់

4.1 សមីការសមមូលលីនេអ ៊ែរ សមីការសមមូ លអញ្ញាត ស័យថាមាស

មួ យកំណត់ ដោយ

ដែល

។ កដសោម

។ ែូដ្នេះ សមីការសមមូ លអញ្ញាត

ដែល

ិ ។ ែូដ្នេះ ដយើ ង មាស្ដមល ើយ សិងត្រាសមកវញ

មាស្ដមល ើយ លេះត្រាដតសមីការែយូផង់ ដ ើ ញថា សមីការសមមូ ល

មាស

មាស្ដមល ើយទាល់ ដត

4.2 ររឹសីតបរ នេឱ្យចនេ​េ ួ េរ់

។ នបើសមីការសមមូល

ន ោះ វាមាេចនមលើយមិេសមមូលគ្នា តាម

ចនេ​េ ួ

មាេចនមលើយមួយ ។

សរមាយបញ្ជាក់ តាងចម្លើយ្ួយម ោះមោយ

។ តា្ររឹស្តីបរ 3.17 មយើ ងទាញបាន ស្​្ីការដយូផង់ លីមនអ ៊ែ រ

មានចម្លើយមានរាង

លឹម សុ វណ្ណវិចរិ រ | 4.1 សមីការសមមូលលីនេអ ៊ែរ

31


មយើ ងឱ្យ

មានតម្​្ល

មបើ

មយើ ងទាញបាន ចម្លើយ្ិនស្​្​្ូលគ្ននតា្

ជាចម្លើយ្ួយមផេងមរៀត អដល

ម ោះមយើ ងស្រមស្រ

ចំ នន ួ

ជា

។ មយើ ងទាញបាន

មានន័យថាឫស្

ស្​្​្ូលមៅនឹងចម្លើយ្ួយអដលមានរួច្កម ើ យកនង ុ ចំ មោ្ ។

ដូមចនោះ មបើស្​្ីការមានចម្លើយ្ួយ ម ោះ មានចម្លើយ្ិនស្​្​្ូលគ្ននតា្ ឧទាហរណ្៍:

ចំ នន ួ

៕៚

ន ោះរាយសមីការ

ចនមលើយ តា្ររឹស្តីបរ 4.2 ស្​្ីការមនោះមានចម្លើយ្ិនស្​្​្ូលគ្ននតា្ អត្ួយគត់ មររោះ ។ ជាដំ បូងមយើ ងមោ​ោះរាយស្​្ីការដយូផង់

មយើ ងទាញបាន

ដូមចនោះ

ឧទាហរណ្៍:

។ មយើ ងមាន

៕៚

3x  6  mod12 

ន ោះរាយសមីការ

ចនមលើយ មោយ

ម ើយ

។ មយើ ងព ិនិតយមឃើ ញថា

ម ោះ ស្​្ីការមានចម្លើយចំ នន ួ

ជាចម្លើយ្ួយ។ តា្ររឹស្តីបរ3.17 រគប់ ចម្លើយទាំង ស្់ មានរាង ។ មោយឱ្យ

ការស្​្​្ូលតា្ 32

គឺ

អដល្ិនស្​្​្ូលគ្ននតា្

ម ោះចម្លើយ្ិនស្​្​្ូលគ្ននទាំង

៕៚

៤ - ភាពសមមូលកាុងសនណ្ុនចនេ​េ ួ េរ់ | លឹម សុ វណ្ណវិចរិ រ

របស្់ ស្​្ី


4.3 ររឹសីតបរ នេឱ្យចនេ​េ ួ េរ់

េិង ចនេ​េ ួ េរ់ មិេសូ េយ

លុោះរតាអរ

ន ោះ

េិងរាសមកវិញ។

សរមាយបញ្ជាក់ មបើ

ម ោះ

ចំ មរោះចំ នន ួ គត់

ោ្ួយ។ ដូមចនោះ

តា្ររឹស្តីបរ 3.5 មយើ ងទាញបាន ទាញបាន

។ តា្ររឹស្តីបរ 3.4 មយើ ង

អចកោច់

ដូមចនោះ

។ ។

រាស្​្កវ ិញ មបើ

មោយ

អចកោច់

ម ោះ

4.4 កូរអូ លរ នបើ ឧទាហរណ្៍ : ចូរេណ្

េិង អែល

ន ោះ

េិង

ចនមលើយ

លឹម សុ វណ្ណវិចរិ រ | 4.3 ររឹសីត បរ

33


មោយ

ដូមចនោះ

។ មោយ

ដូមចនោះ

ដូមចនោះ

។ ដូមចនោះ មផទៀងផ្ទទត់ ស្​្ីការស្​្​្ូលអដលឱ្យ៕៚

ឧទាហរណ្៍ :

ចូរេណ្ ចនេ​េ ួ េរ់ សល់ សនណ្ល់

មួយ អែលនពលអចកេឹង

នហើ យ នពលអចកេឹង

សល់ សនណ្ល់ , នពលអចកេឹង

សល់ សនណ្ល់ ។

ចនមលើយ មយើ ងចង់ បាន

អដល

មយើ ងមាន ដូមចនោះ

៕៚

4.5 ររឹសីតបររបប ូ ិេ នេឱ្យចនេ​េ ួ េរ់ វជ្ ិ មា ា េ

។ នបើ

អែលមិេអាចសរនសរជារាង នសមើ េឹង

ន ោះ ចនេ​េ ួ នេចនេ​េ ួ េរ់ វជ្ ិ មា ា េ ចនន ោះចនេ​េ ួ េរ់ មិេ វិជ្មា ា េ

បាេ

សរមាយបញ្ជាក់ មយើ ងស្នមតមៅថា វ ិជ្ជមាន

អដល

ជាចំ នន ួ អាចបំ អបកបានតា្ ។

មយើ ងព ិនិតយតារាងអដលចំ នន ួ ធាតុ្ិនកំណត់ ខាងមរកា្

34

៤ - ភាពសមមូលកាុងសនណ្ុនចនេ​េ ួ េរ់ | លឹម សុ វណ្ណវិចរិ រ

មបើចំមរោះចំ នន ួ គត់

មគមានចំ នន ួ គត់ ្ិន


មលខកនង ុ ជ្ួរឈរនិ្ួយៗម្នតារាងមនោះ ជាស្វុី តនព វនតអដលមានផលស្ងរួ្មស្មើ ចំ នន ួ គត់

ោ្ួយកនង ុ តារាងមនោះ គឺ

បំ អបកជា

អដល

បាន ម ោះ

។ ចំ នន ួ មៅព ីមរកា្

ជាចំ នន ួ គត់ ធ្មជាតិ។ មយើ ងដឹងថាមបើ

អាច

ក៏អាចបំ អបកបានអដរ មររោះ

។ ដូមចនោះមយើ ងទាញបានថា មបើចំនន ួ គត់

្ួយអាចបំ អបកបាន ម ោះរគប់ ចំ នន ួ

គត់ អដលមៅព ីមរកា្វាស្ុ រអធ តអាចបំ អបកបានទាំង ស្់ ។ រគប់ ចំ នន ួ អដលជាព ុ គុណម្ន b ស្ុ រអធ តអាចបំ អបកបានទាំង ស្់ មររោះ មយើ ងនិយាយថា គ្នមនព ុ គុណម្ន b ព ីរខុស្គ្ននតាងមោយ

និង

អដល

អាច

ឋិ តមៅកនង ុ ជ្ួរឈរអត្ួយម្នតារាងខាងមលើបានមរ។ មររោះថា មបើ្ិន ញ្ច ង ឹ មរ ម ោះមយើ ងនឹងមាន ។ មោយ មានន័យថា

និង

បឋ្គ្នន និងតា្ររឹស្តីបរ 3.4 មយើ ងទាញបាន

មោយ

ម ោះមយើ ងទាញបាន

។ ដូមចនោះ្ិនអាចមានចំ នន ួ ព ុ គុណម្ន

ព ីរ

ខុស្គ្ននឋិ តកនង ុ ជ្ួរឈរអត្ួយបានមរ។ ឥលូ វមយើ ងនឹងបង្ហាញថា រគប់ ចំ នន ួ ទាំង ស្់ កនង ុ ជ្ួរឈរអត្ួយម្នតារាង អដលមៅព ីមលើព ុ គុណម្ន តាងមោយ អត្ួយមលើ

ស្ុ រអធ តជាចំ នន ួ ្ិនអាចបំ អបកបាន។ រគប់ ចំ នន ួ អដលមៅកនង ុ ជ្ួរឈរ មានរាង

ចំ មរោះចំ នន ួ គត់

ម ោះមានចំ នន ួ គត់ ្ិន វ ិជ្ជមាន

្ួយ។ មបើ

អាចបំ អបកបានចំ មរោះ

អដលមផទៀងផ្ទទត់

។ ដូមចនោះ

។ ដូមចនោះ

។ ្យ៉ាងវ ិញ

មរៀត ចំ នន ួ ព ីរឋិ តមៅមលើជ្រួ ឈរអត្ួយ ស្​្​្ូលគ្ននតា្

។ ដូមចនោះ

។ មោយ អលរ 4.4) ផទយ ុ ពី

(តា្កូរ ូ ៉ា

ដូមចនោះ ចំ នន ួ ម្នចំ នន ួ ្ិនអាចបំ អបកបានតា្ រាង

ម ោះ

មស្មើ នង ឹ ចំ នន ួ ម្នចំ នន ួ អដលមៅព ីមលើចំនន ួ អដលមាន

។ មៅមលើជ្រួ ឈររី មគមានចំ នន ួ ចំ នន ួ

ចំ នន ួ ម្នចំ នន ួ អដល្ិនអាចបំ អបកបានតា្

ព ីមលើ

។ ដូមចនោះ

មស្មើ នង ឹ លឹម សុ វណ្ណវិចរិ រ | 4.5 ររឹសីត បររបប ូ ិេ

35


ចំ នន ួ គត់ អដល្ិនអាចបំ អបកបានធំ បំផត ុ ឋិ តមៅចំ ព ីមលើ បំ អបកបានធំ បំផត ុ មានតម្​្លមស្មើ នង ឹ

ដូមចនោះ ចំ នន ួ គត់ អដល្ិនអាច

៕៚

4.6 ររឹសីតបរ នេឱ្យ

ជាចនេ​េ ួ េរ់ បឋមរវាងគ្នា។ សមីការ

មិេ វិជ្មា ា េ

ចនន ោះ

គ្នមេចនមលើយជាចនេ​េ ួ េរ់

។ នបើ

ន ោះ សមីការ

នេោះ មាេចនមលើយជាចនេ​េ ួ េរ់ មិេ វិជ្មា ា េ។

4.7 ររឹសីតបរសនណ្ល់អចកចិេ នេឱ្យ តាង

ជាចនេ​េ ួ វិជ្មា ា េបឋមរវាងគ្នាពីរៗ នហើ យសុ រធអរធនជាងមួយ។ ជាចនេ​េ ួ េរ់ ណាមួយ ន ោះរបព័េស ធ មីការសមមូល

មាេចនមលើយ នហើ យអរមួយេរ់ តាម

សរមាយបញ្ជាក់ តាង

។ តាង

មាន

អដល

អដលមផទៀងផ្ទទត់ លកខខណឌខាងមលើ មររោះរគប់

មយើ ងបមងកត ើ ចំ នន ួ មយើ ងមឃើ ញថាចំ នន ួ ខាងមលើមផទៀងផ្ទទត់ របព័នស្ ធ ្ីការ។

36

៤ - ភាពសមមូលកាុងសនណ្ុនចនេ​េ ួ េរ់ | លឹម សុ វណ្ណវិចរិ រ

បឋ្រវាងគ្ននព ីរៗ

។ មយើ ងដឹងថា ឱ្ ំ យ

បឋ្នឹង


មដើ្បីបង្ហាញថា ចំ មលើយមានអត្ួយ មយើ ងស្នមតថា មាន ចំ មរោះរគប់

។ ដូមចនោះ

និ្ួយៗអចកោច់

។ មោយ

ចំ មរោះរគប់

។ មយើ ងទាញបានថា

បឋ្រវាងគ្ននព ីរៗ ម ោះមយើ ងទាញបាន

ឬនិយាយ្យ៉ាងមរៀត ឧទាហរណ្៍ :

មផេងមរៀត អដល

៕៚

នរើនេអាចរកបាេមួយលាេចនេ​េ ួ េរ់ រនរៀងគ្នាអែលចនេ​េ ួ ទានង ស់ ន ោះ សុ រធអរអចក ច់ េឹងចនេ​េ ួ កានរឬនរ?។

ចនមលើយ តាង

ជាចំ នន ួ គត់ ទាង ំ ្ួយលានបឋ្គ្ននព ីរៗ។ តា្ររឹស្តីបរស្ំ ណល់ អចកចិន

របព័នស្ ធ ្ីការខាងមរកា្មានចម្លើយ

បោ ត ចំ នន ួ

ជា្ួយលានចំ នន ួ គត់ តមរៀងគ្នន អដលនិ្ួយៗអចក

ោច់ នឹងកាមរម្នចំ នន ួ បឋ្៕៚

4.8 ររឹសីតបរវីលសុេ ចនន ោះរេប់ ចេ ន េ ួ បឋម

នេមាេ

សរមាយបញ្ជាក់ មយើ ងដឹងថា មបើ

បឋ្នឹង

ម ោះមគមាន

អដល

ម ោះតា្ររឹស្តីបរ Bachet-Bézout មគមានចំ នន ួ គត់ ។ តាង

។ ដូមចនោះ

អដល

។ មោយ

។ មានន័យថា ចំ មរោះ អត្ួយគត់ ) អដល

។ មររោះមបើ និង

្ួយអដលបឋ្នឹង

។ ដូមចនោះ ចំ មរោះ

មគមាន មគមាន

ម ោះ ្ួយ (ម ើ យមាន អដល

លឹម សុ វណ្ណវិចរិ រ | 4.8 ររឹសីត បរវីលសុ េ

37


និង

។ មបើ

ម ោះ ។

មោយ

ម ោះ

មបើ

ស្នមតថា

។ ដូមចនោះចំ មរោះ

មផេងគ្នន ម ោះមយើ ងមាន

មផេងគ្ននអដរ។

ម ោះររឹស្តីខាងមលើព ិត។ ។ ព ិនិតយ

អដល

។ ចំ មរោះ

្ួយមរៀតអដល ្ិនអាចមររោះ

។ មបើ

និ្ួយៗខាងមដើ្មនោះ មយើ ងកំណត់

។ មបើ

ម ោះ

ម ោះ

។ អត

។ ដូមចនោះ

។ ចំ មរោះ

មផេងគ្ននមយើ ងមាន

មផេងគ្ននអដរ។ មបើ ។ មោយ

ម ោះ

ជាចំ នន ួ បឋ្ម ោះ

។ អត

និង

។ ដូមចនោះ

មយើ ងមាន

និង ចំ មរោះ ម ើ យមបើ

ស្ំ ណំ ុ

មផេងគ្នន មយើ ងររួលបាន

ជាព ីរគឺ និង

ឬ្ិន ញ្ច ង ឹ មរ ។ អត្ិនអាចទាំងព ីរមររោះ ។

្ួយមយើ ងអាចរកបាន

មផេងគ្ននអដរ។ ដូមចនោះមយើ ងអាចបំ អបក អដលមផទៀងផ្ទទត់

។ ស្ំ ណំ ុ

តាង

និង

មានចំ នន ួ ធាតុមស្មើ នង ឹ អដល

ឧទាហរណ្៍ :

38

នបើ

្ួយគត់ អដល

ជាចនេ​េ ួ បឋម អែល

៤ - ភាពសមមូលកាុងសនណ្ុនចនេ​េ ួ េរ់ | លឹម សុ វណ្ណវិចរិ រ

។ និង

ចូរបង្ហាញថា

និង ។ ដូមចនោះ


ចនមលើយ មយើ ងស្មងកតមឃើ ញថា មយើ ងមាន

មោយ

ម ោះ

។ ដូមចនោះ

តា្ររឹស្តីបរវលសុ ន ី

ដូមចនោះ

4.9 កូេររឹសីតបរអែមា ( Fermat’s Little Theorem)

រេប់ ចេ ន េ ួ បឋម

េិងរេប់ ចេ ន េ ួ េរ់

នេមាេ

សរមាយបញ្ជាក់ មយើ ងនឹងរាយបញ្ជជក់ តា្វ ិារមោយកំមណើនតា្ ស្នមតថា

អចកោច់

។ មពល

មយើ ងទាញបានថា ស្ំ មណើព ិត។

។ មយើ ងមាន

លឹម សុ វណ្ណវិចរិ រ | 4.9 កូេររឹសីត បរអែមា

39


ចំ មរោះ

មយើ ងមាន

មានន័យថា

អតមោយ

ដូមចនោះ

។ ចំ មរោះ

ក៏មយើ ងមាន

អដរ។ ដូមចនោះ

ដូមចនោះ

អចកោច់ នឹង

។ ដូមចនោះស្ំ មណើព ិត៕៚

ាមរយៈត្ររឹសតីបរដសេះដយើ ងទាញបាសថា ដរ ដ េះមាសស័យថា

ដ្ក

។ ែូដ្នេះ ដបើ

ោ្់ ។ ែូដ្នេះ

ដ្ក

មិសោ្់

4.10 វិបាក តាង

ជាចនេ​េ ួ បឋមេិងសេមរថា

ឧទាហរណ្៍ : តាង

អចក

មិេ ច់ ន ោះ

។ ចូរេណ្ សនណ្ល់ នេ

អចកេឹង

ចនមលើយ តា្ររឹស្តីបរភែម៉ា មយើ ងមាន

។ មយើ ងមាន

រគប់ ចំ នន ួ គត់ វ ិជ្ជមាន

ចំ មរោះចំ នន ួ គត់

អចកនឹង

40

ស្ល់

មានន័យថា ៕៚

៤ - ភាពសមមូលកាុងសនណ្ុនចនេ​េ ួ េរ់ | លឹម សុ វណ្ណវិចរិ រ

ចំ មរោះ ោ្ួយ។ ដូមចនោះ


៥ - ផ្នែកគត់ 5.1 ផ្ផែកគរ់ និង ផ្ផែកទសភាគ ផ្ផែកគរ់ នន

តាងដោយ

ជាចំនន ួ គរ់ ធប ំ ផ ំ រ ុ ផ្ែលរូចជាងឬដសមើ ។

ឧទាហរណ្៍ ដយើ ងមាន

ឬក៏

ដគដរបើសញ្ញាសមា​ាល់

តាងឱ្យផ្ផែកទសភាគ។

ែូដចែេះ

5.2 រទឹសីតបទ ដគឱ្យ

ដ េះ

1) 2) 3)

សរមាយបញ្ញាក់ 1) តាង

។ ដូច្នេះ មានន័យថា

2)

លឹម សុ វណ្ណវិចរិ រ | 5.1 ផ្ផែកគរ់ និង ផ្ផែកទសភាគ

41


ចោយ

ជា្ំ នន ួ គត់ ច េះតាម 1) ចយើ ងទាញបាន

ចោយ

ដូច្នេះ

3) តាមវ ិសមភាព ចោយ

ចយើ ងទាញបាន ជា្ំ នន ួ គត់ ត្ ូ ជាងឬចសមើ

។ មានន័យថា

ឧទាហរណ្៍ : ចូរបង្ហាញថា ចំដ េះរគប់

ែំដ ចយើ ងមាន

និង

ដូច្នេះចយើ ងរតូវបង្ហាញថា

42

ច េះវារតូវតតតូ្ជាងឬចសមើ តផនកគត់ របស់

៥ - ផ្ផែកគរ់ | លឹម សុ វណ្ណវិចរិ រ

វិសមភាពខាងដរោមពិរ

េះរាយ ។ ដូច្នេះ


ចោយសារវ ិសមភាពគ្មមនលកខណៈព ិចសសរវាង ដឹងថា

និង

ច េះចយើ ងអា្សនមតថា

។ ចយើ ង

ដូច្នេះ

ព ិត៕៚

5.3 រទឹសីតបទ ដបើ

ជាចំនន ួ គរ់ ធមមជារិបឋមរវាងគ្នែ ចូរបង្ហាញថា

សរមាយបញ្ញាក់ ព ិនិតយ្តុចោណតកងមួយ តដលមានកំព ូលរតង់ មាន

។ ្តុចោណតកងចនេះ

្ំ ណុ្តដលមានកូអរចោចនជា្ំ នន ួ គត់ ។ ្តុចោណតកងចនេះត្កជាព ីរ

្ំ តណកចសមើ គ្មនចោយប ា ត់

ចយើ ងនិយាយថា គ្មមន្ំ ណុ្ណាមួយតដលឋិ តចៅចលើប ា ត់ ចនេះ មានកូអរចោចនជា្ំ នន ួ គត់ ចេ ចលើក តលងតត្ំ ណុ្​្​្ុងទាំងព ីរ។ ចររេះថាចបើសិនជាមាន្ំ ណុ្មានកូអរចោចនគត់ ឋិតចៅចលើប ា ត់ ចនេះ រតង់ ចៅជា

តដល ផាយ ុ ព ីសមមតក ិ មម តដល

ច េះ

។ ច េះ មានន័យថារបភាគ

អា្បរងួម

បឋមរវាងគ្មន ។ លឹម សុ វណ្ណវិចរិ រ | 5.3 រទឹសីត បទ

43


្ំ ណុ្

ជាបណា ា ្ំ ណុ្តដលឋិ តចៅចលើប ា ត់ ចនេះ។

្ំ នន ួ ្ំ ណុ្មានកូអរចោចនគត់ តដលឋិ តចៅចលើប ា ត់ ឈរ ច្ញព ី

ចសមើ នង ឹ

ចៅ

។ដូច្នេះ

ជា្ំ នន ួ ្ំ ណុ្មានកូអរចោចនគត់ តដលឋិ តចៅកនង ុ រក់ កណា ា លតផនកខាងចរោមរបស់ ្តុចោណតកង។ ដូ្គ្មនតដរ

ជា្ំ នន ួ ្ំ ណុ្មានកូអរចោចនគត់ តដលឋិ តចៅកនង ុ រក់ ក

ណា ា លតផនកខាងចលើរបស់ ្តុចោណតកង។ ចោយជាសរុបមាន

្ំ ណុ្មានកូអរចោ

ចនគត់ ច ើ យត្កចសមើ គ្មនតផនកខាងចលើនង ិ ខាងចរោម្តុចោណតកង

សំ ចណើខាងចលើព ិត៕៚

5.4 រទឹសីតបទឌីបូលញ ី ក់ (De Polignac) តាង ជាចំនន ួ បឋម។ ចំនន ួ គរ់ ធប ំ ផ ំ រ ុ

ផ្ែល

ផ្ចកោច់

កំណ្រ់ ដោយ

សរមាយបញ្ញាក់ កនង ុ ្ំ ចណាម្ំ នន ួ គត់ ព ី ដល់ ោ្់ នឹង ឧទា

មាន្ំ នន ួ

, ្ំ នន ួ គត់ តដលត្កោ្់ នឹង

,.....។ ដូច្នេះ

រណ៍

ត្កោ្់ នឹង

។ ព ី ដល់

។ ្ំ នន ួ ត្កោ្់ នឹង

មាន្ំ នន ួ

គឺ

មាន្ំ នន ួ សវ័ យគុណ

្ំ នន ួ ត្កោ្់ នឹង

។ ដូច្នេះ

, ្ំ នន ួ គត់ តដលត្ក ។ មាន្ំ នន ួ

ត្កោ្់ នឹង

គឺ

៕៚

ឧទាហរណ្៍ : ដរើមានដលខសូនយចំនន ួ បុ ម នននខាងចុង

ែំដ ចលខសូ នយខាង្ុង ថា

ចសមើ នង ឹ ្ំ នន ួ តដលជាសវ័ យគុណនន

ធំ បំផត ុ តដលត្កោ្់

បានចេ ចររេះ

44

េះរាយ មានន័យ

។ ចយើ ងមិនអា្យករូបមនា ឌ ីបូលញ ី ក់ មកចរបើចោយផ្ទាល់

មិនតមនជា្ំ នន ួ បឋម។ តត

៥ - ផ្ផែកគរ់ | លឹម សុ វណ្ណវិចរិ រ

ធំ បំផត ុ តដលត្កោ្់

។ ដូច្នេះ

ធំ បំផត ុ តដល

ត្កោ្់


ចសមើ នង ឹ តនមល ត្ ូ ជាងចគនន បំ ផត ុ តដល

ត្កោ្់

កនង ុ ្ំ ចណាម ( ធំ បំផត ុ តដល

) និង ( ធំ

)។ តាមរូបមនា ឌ ីបូលញ ី ក់

ធំ បំផត ុ តដល

ត្កោ្់

ចសមើ នង ឹ

ធំ បំផត ុ តដល

ត្កោ្់

ចសមើ នង ឹ

ដូច្នេះ

ត្កោ្់

មានចលខសូ នយចៅខាង្ុង្ំ នន ួ

ខាង់៕៚

លឹម សុ វណ្ណវិចរិ រ | 5.4 រទឹសីត បទឌីបូលញ ី ក់

45


46

៥ - ផ្ផែកគរ់ | លឹម សុ វណ្ណវិចរិ រ


៦ - អនុគមន៍ នព្វន ត 6.1 អនុគមន៍នពវន ត អនុគមន៍នពវនជា ត អនុគមន៍ ដែលមានដែនកំណ្រ់ ជាសំណ្ុំរងននចំនន ួ គរ់ វជ្ ិ មា ជ ន។ ខាងក្រោម ក្នេះជាអនុគមន៍នពវនស ត ំខាន់ ៗកនុងនពវន ត ជាចំនន ួ រួដចកវិជ្មា ជ នរបស់ ជាផលបូកននរួដចកវិជ្មា ជ នទំងអស់ របស់ ជាផលបូកសវ័ យគុណ្ ននរួដចកវិជ្មា ជ នទំងអស់ របស់ ជាផលគុណ្រួដចកវិជ្មា ជ នទំងអស់ របស់ ជាចំនន ួ ននចំនន ួ គរ់ វជ្ ិ មា ជ នដែលធំមិនក្លើសពី

និងបឋមនឹង

។ ក្គក្ៅ

ថា

អនុគមន៍អឺដល ជាចំនន ួ រួដចកបឋមខុសៗគ្ននរបស់ ជាចំនន ួ រួដចកបឋមរបស់ n រាប់ មិនបាច់ គិរថាខុសគ្ននឬអរ់ ក្េ(ក្សមើ នឹងផលបូកននសវ័ យគុណ្របស់ កត្តតបឋមរបស់ ែូក្ចនេះ

ឧទហរណ្៍

ក្យើ ងមាន ចំនន ួ ពីរែង ចំនន ួ មដង

)។

អនុគមន៍ខាងក្លើអាចត្តងជាសញ្ញាក្ោយ

លឹម សុ វណ្ណវិចរិ រ | 6.1 អនុគមន៍នពវន ត

47


(សញ្ញា

ក្ៅកនុង

ឧទាហរណ៍

មានន័យថា

ដរ

ជាតួចែករបស់

។ យ ើ ងមាន ិ មា ជាែំនន ួ គត់ វជ្ ជ នធំមិនយ ើ សពី

។ យោ យ ោះ

)

និងបឋមនឹង

មានន័ ថា ែំយ ោះ ើ ផ បូ កមួ ោែ់ យ ើ ងបយងក ន

ចែក

ដូយែនោះ

មានន័ ថា

ចរបរបួ ពី

ដ ់

យបើមានែំនន ួ គត់

ឯកតា។ ឧទាហរណ៍

មានតួចែកែំនន ួ បួ នគឺ

ណាមួ

យ ើ ងចរបរបួ

6.2 រេឹសីតបេ ក្បើផលគុណ្កត្តតបឋមរបស់

មានរាង

ក្ េះ ក្យើ ងមាន

សរមាយបញ្ញជក់ ដ ោះ តួចចករបស់

ដោយ ដោយយកតម្លល

48

ចរបរបួលព ី ដល់

៦ - អនុគមន៍នពវន ត | លឹម សុ វណ្ណវិចរិ រ

មានរាង

ដយើ ងផសំបានតួចចករបស់

ចដល មានចំ នន ួ


តួចចករបស់

មានដូចជា

ដយើ ងមាន

កនង ុ របមាណវ ិធីខាងដលើ ដំ បូងដយើ ងទប់ តម្លល របមាណវ ិធីបូកតាល

ដោយឱ្យ

ឱ្យដៅដថរសិ ន ប ា ប់ លកដយើ ងដធវ ើ

ចរបរបួលព ី ដល់

។ ដយើ ងដធវដ ើ ច ូ គ្ននចំ ដ ោះសវ័ យគុណដផសង

ដទៀត។ តួចចកវ ិជ្ជមានលួយរបស់ ចចកទំងអស់ របស់ ដយើ ងនឹងគណ

មានរាង

ចដល

មានរាង ។ ដយើ ងយកចំ នន ួ គត់

មានទំងអស់ ចំ នន ួ

។ ផលគុណរបស់ បណ្ត ា តួ

។ លួយចដល

។ តួចចករបស់

ចដល

។ ដពលដយើ ងគុណតួចចកទំងអស់ ដនោះ

បញ្ចូ លគ្នន ដយើ ងបាន

លឹម សុ វណ្ណវិចរិ រ | 6.2 រេឹសីត បេ

49


តាលរដបៀបដូចគ្នន ដយើ ងទញបាន

ឧទហរណ្៍ :

។ ដូដចនោះ

ចូរបង្ហាញថា

ចក្មលើយ រគប់ តួចចកវ ិជ្ជមាន

និលួយៗម្ន

។ ដោយ និង

ដយើ ងចតងចតអាចរកបានតួចចកលួយដទៀតរបស់

ដ ោះ រតូវចតមានលួយកនង ុ ចំ ដណ្តល ដ ោះ

តាង

និង

ចដល

ចដលដសមើ នង ឹ ដរ ោះដបើ

លិនព ិត។

ជាតួចចកវ ិជ្ជមានរបស់

ចដលធំ លិនដលើសព ី

។ តួចចករបស់

ដផសង

ដទៀតគឺ

ដោយ

ដ ោះ

។ ដយើ ងមាន

ឧទហរណ្៍ :

ចូរគណ្ រគប់ ចន ំ ន ួ គរ់

៕៚

ដែល

ចក្មលើយ ដយើ ងមាន

។ ដបើ

មានកតា​ាបឋល

ដផសងគ្ននដរចើនដលើសព ីព ីរ ដ ោះ

។ ដូដចនោះ n

រតូវចតមានកតា​ាបឋលខុសគ្ននចត២បុដណ្តណោះ គឺ

និង

ដូដចនោះ

ចដល

រតូវចតមានរាង

ឧទហរណ្៍ :

50

ឬក៏

ចូរបង្ហាញថា

៦ - អនុគមន៍នពវន ត | លឹម សុ វណ្ណវិចរិ រ

។ដូដចនោះ

និង

ជាចំ នន ួ បឋលខុសគ្នន៕៚


ចក្មលើយ ដយើ ងមាន

ចដល

ដបើ

និង

ដបើ ចចក

លិនោច់ ។

ដសមើ នង ឹ ចំ នន ួ ចំ នន ួ គត់ ចដលមានរាង

ដយើ ងមាន

ដរ ោះ

ជាផលចចកគត់ ចដលបានលកដោយចចកចំ នន ួ គត់

ចំ នន ួ ចំ នន ួ គត់ ព ី ដល់ សនលក ឹ ដបៀចំ នន ួ

ចដលមានរាង

សនលក ឹ ដល់ លនុសស

ដូចជា

នឹង

និង

។ ដបើដយើ ងព ិនិតយដលើរដបៀបចចក

ក់ មានន័យថា ចចកគ្ននលដងលួយសនលក ឹ ៗរហូ តដល់ រគប់ គ្នន រួច

ចចកលដងលួយសនលក ឹ ដទៀត ចចករហូ តដល់ សល់ ដបៀចចកចលងដកើត ដយើ ងទញបានផលចចកនិងចំ នន ួ ចំ នន ួ គត់ ចដលមានរាង អស់ ដបៀចំ នន ួ

ដសមើ គ្នន។ ឧទហរណ៍ ចចកដបៀ៩សនលក ឹ ដល់ លនុសស៤ ក់ ។ ចចកលួយសា សនលក ឹ បានមានក់ លួយសនលក ឹ ដៅសល់ ដបៀ

ដបៀសរុបគិតទំងដលើកលុនចំ នន ួ

បានមានក់ លួយសនលក ឹ ដទៀត ដហើ យសល់ ដបៀលួយសនលក ឹ ។

ដូដចនោះជាសរុបចចកបានព ីរសា (ដសមើ ចំនន ួ ចំ នន ួ ចដលមានរាង ចចក)។ ដូដចនោះ

សនលក ឹ ។ ចចកលួយសារដទៀតអស់ ) បានមានក់ ព ីរសនលក ឹ ចដរ (ផល

ព ិត។

ដយើ ងទញបាន

លឹម សុ វណ្ណវិចរិ រ | 6.2 រេឹសីត បេ

51


ឧទហរណ្៍ :

ចូរបង្ហាញថា ចំក្ េះរគប់ ចន ំ ន ួ គរ់ ធមមជារិ និង

ក្គមានចំនន ួ គរ់ ធមមជារិ

និង

ដែល

ចក្មលើយ តាង

។ ដយើ ងយក

។ ដយើ ងមាន

(សូ លអានដសចកដពី នយល់ កនង ុ រទឹសាីបទ 6.2) ។ ដូដចនោះ មានន័យថា មាន

ចដលដផាៀងផ្ទាត់ លកខខណឌ៕៚

6.3 រេឹសីតបេអឺដល ក្បើ

ក្ េះ

សរមាយបញ្ញជក់ ដយើ ងមាន ជាង

។ តាង

ដហើ យបឋលនឹង

សល់ សំ ណល់ ជា

ចដល

។ រគប់ ចំ នន ួ គត់ ទង ំ អស់ ចដលបឋលនឹង

លួយកនង ុ ចំ ដណ្តល

[*]។

ដ ោះ

ក៏បឋលនឹង

ដោយ

ដហើ យ ដបើ

ជាបណ្ត ា ចំ នន ួ គត់ វ ិជ្ជមាន តូច

ដ ោះ និង

និង

ដពលចចកនឹង

ចដរ។ ដូដចនោះ

ចដល

ដរ ោះដបើមាន

ដ ោះ ដរ ោះ

។ ដូដចនោះ 52

៦ - អនុគមន៍នពវន ត | លឹម សុ វណ្ណវិចរិ រ


ដោយ

ដ ោះ

សេចក្ដី ពន្យល់ បន្ន្ែ ម ត្រង់ចណ ំ ុ ច [*] ដែលនិយាយថា «ត្រប់ ចន ំ ន ួ ររ់ ទំងអស់ ដែលបឋមនឹង សំណល់ ជា

មួយកនុងចំពោម

ឧទហរណ៍ ចំព

ោះ

ចំនន ួ បឋមនឹង

ពេលដចកនឹង

សល់

»

ពយើ ងមាន

ជា

។ ែូពចនោះ

។ ពយើ ងមាន

។ ពយើ ងមាន

បឋមនឹង

បឋមនឹង

ពហើ យ

ពហើ យ

។ ពយើ ងអាចេនយល់ លកខណៈពនោះជាទូពៅពោយ តាងចំនន ួ ររ់ មយ ួ ពោយ ។ ពបើ

និង

និង

ោះ

នឹង

។ ពោយ

បឋមគ្នន ព

ោះ

ក៏ជារួដចករបស់ បឋមនឹង

ត្រូវដរបឋមនឹង ដែរ។ ែូពចនោះ

ពហើ យ

ដែរ។ ពត្

ឱ្ ំ យ ព

និង ោះ

ោះថា ពបើមាន

ជារួដចករ ួមរវាង

មិនបឋមគ្នន។ ែូពចនោះ

ត្រូវដរបឋម

ត្រូវដរជាចំនន ួ មួយកនុងចំពោម

6.4 រេឹសីតបេ ត្តង

ជាចំនន ួ គរ់ វជ្ ិ មា ជ នរូចបំផរ ុ ដែល

ដែល

បាន ក្បើ

។ ចំនន ួ គរ់ វជ្ ិ មា ជ ន

ជាពហុ គុណ្នន

និងរាសមកវិញ។

សរមាយបញ្ញជក់ ដបើ

ជាពហុ គុណម្ន

រាសលកវ ិញ សនមតថា

ដ ោះ

។ ដោយ ។ យក

ដ ោះ

ព ិត។ ។ ដូដចនោះ

ដយើ ងមាន លឹម សុ វណ្ណវិចរិ រ | 6.4 រេឹសីត បេ

53


ដបើ

ដ ោះ មាន

ដគ។ ដូដចនោះ

ចដល

។ ដូដចនោះផាយ ុ ព ីសលមតក ិ លម ចដលថា

។ ដូដចនោះ

ជាពហុ គុណម្ន

តូចជាង

៕៚

6.5 រេឹសីតបេ ត្តង

និង

ជាចំនន ួ គរ់ វជ្ ិ មា ជ នពីរដែលបឋមគ្នន ក្ េះ

សរមាយបញ្ញជក់ តាង

ជាចំ នន ួ គត់ ធលមជាតិលួយចដល

។ ដយើ ងតំដរៀបចំ នន ួ គត់ ចំនន ួ

គឺ

ដូចខាងដរោល

ចំ នន ួ គត់

លួយ បឋលនឹង

ដបើ វាបឋលនឹង

និងនឹង

កំណត់ ចំនន ួ ម្នចំ នន ួ គត់ កង នុ តារាងខាងដលើ ចដលបឋលនឹង

និងរាសលកវ ិញ។ ជាដំ បូងដយើ ងនឹង និងប ា ប់ លកគណ ឱ្យដឃើ ញថា ដតើ

មានចំ នន ួ បុ ម នកនង ុ ចំ ដណ្តលដ ោះចដលបឋលនឹង ។ មានចំ នន ួ គត់ ចំនន ួ ចដល

ដៅកនង ុ ប ា ត់ ដដកទី១ចដលបឋលនឹង

។ ចំ នន ួ គត់ នល ិ ួយៗឋិ តដៅកនង ុ ជ្ួរឈរលួយដនោះ មានរាង

។ ដោយ ជាលួយ

។ ឥលូ វដយើ ងដលើលប ា ត់ ឈរទី

ដ ោះ

មានតួចចកលួយជាលួយ

ចដរនិងរាសលកវ ិញ។ ដូដចនោះ ដបើ

ជ្ូរឈរជាលួយ

ក៏បឋលនឹង

បឋលនឹង

ដបើ

មានតួចចករួលលួយ

ដ ោះរគប់ ចំ នន ួ គត់ ដផសងដទៀតឋិ តដៅកនង ុ

ចដរ។ ដូដចនោះមានចំ នន ួ គត់ ចំនន ួ

ជ្ួរឈរចដលបឋលនឹង

ប ា ប់ លកដយើ ងកំនត់ ថាដតើមានចំ នន ួ គត់ ចំនន ួ បុ ម នកនង ុ ចំ ដណ្តលដនោះ ចដលបឋលនឹង ។ ដយើ ងនិយាយថា លិនអាចមានចំ នន ួ គត់ ព ីរកនង ុ ចំ ដណ្តល ទី

សលលូលគ្ននតាល

54

ដទ។ ដបើលិនអញ្ច ង ឹ ដទ ដបើ

៦ - អនុគមន៍នពវន ត | លឹម សុ វណ្ណវិចរិ រ

ដៅដលើប ា ត់ ឈរ ដ ោះ


។ ដោយ

ដ ោះ

។ ដោយ

ដ ោះ

។ ដូដចនោះដយើ ងទញបានថា បណ្ត ា ចំ នន ួ គត់ ឋិតដៅកនង ុ ជ្ួរឈរចតលួយ កនង ុ ចំ ដ លជ្ួរឈរនិលួ យៗម្នជ្ួរឈរទំងអស់ ចំ នន ួ

សលលូលដៅនឹង

កនង ុ ចំ ដ លដនោះបុដណ្តណោះចដលបឋលនឹង តារាងដនោះ ចដលបឋលនឹង

។ ចតមានចតចំ នន ួ គត់ ចំនន ួ

។ មានន័យថា មានចំ នន ួ គត់ ចត

បុដណ្តណោះ ដៅកនង ុ

៕៚

េម្គាល់ - យបើ

ជាែំនន ួ បឋម និង

ិ មា វជ្ ជ នចតមួ

ជាែំនន ួ គត់ ធមម ជាតិ យ ោះែំនន ួ គត់

ចបបគត់ ចដ

និងមានកតា​ារួមជាមួ ជាផ គុ ណកតា​ាបឋមរបស់

យបើ

ជាែំនន ួ គត់ ។ ដូយែនោះ

យ ោះ

6.6 រេឹសីតបេ ចំក្ េះចំនន ួ គរ់ វជ្ ិ មា ជ ន

ក្គមាន

សរមាយបញ្ញជក់ ដយើ ងព ិនិតយសំ ណំ ុ ម្នចំ នន ួ សនិទន

ដយើ ងដឹងថា សំ ណំ ុ ដនោះមានធាតុទង ំ អស់ ចំ នន ួ

។ ដរោយសរលួលរបភាគដនោះដហើ យ ដយើ ងនឹងទទួល

បានសំ ណំ ុ ថម ី ចដលភាគចបងនិងភាគយក បឋលនឹងគ្នន។ បណ្ត ា ភាគចបងកនង ុ សំ ណំ ុ ថមសុ ី ទចធ តជាតួចចក របស់

។ តាង

ជាភាគចបងលួយ។ ដ ោះ កនង ុ សំ ណំ ុ ថម ី មានធាតុចំនន ួ

ង។ ដូដចនោះ សំ ណំ ុ ថមមានធាតុ ី ទង ំ អស់ ចំ នន ួ

ចដលមាន

ជាភាគចប

។ ដោយសំ ណំ ុ ទំងព ីរមានចំ នន ួ តួដសមើ គ្នន ដ ោះ

ដយើ ងទញបានសំ ដណើព ិត៕៚

លឹម សុ វណ្ណវិចរិ រ | 6.6 រេឹសីត បេ

55


ឧទហរណ្៍ :

ត្តង

ជាចំនន ួ គរ់ វជ្ ិ មា ជ ន។

(1) គណ្ ផលបូកននរគប់ ចន ំ ន ួ គរ់ វជ្ ិ មា ជ ន រូចជាង

ក្ហើ យបឋមនឹង

(2) គណ្ ផលបូកននរគប់ ចន ំ ន ួ គរ់ វជ្ ិ មា ជ ន រូចជាង

ក្ហើ យបឋមនឹង

ចក្មលើយ តាង

តាង

ជាបណ្ត ា ចំ នន ួ គត់ វ ិជ្ជមានតូចជាង ទល់ ចត

ដូដចនោះ

បឋលនឹង

។ ដយើ ងមាន ចដរ ចតតូចជាងដគ ដូដចនោះ

លយងវ ិញដទៀត ដយើ ងមាន

ដូដចនោះ

ចូរឱ្យឧទហរណ្៍ចំនន ួ គរ់

ដែល

ចក្មលើយ 56

៦ - អនុគមន៍នពវន ត | លឹម សុ វណ្ណវិចរិ រ

។ ដយើ ងមាន

ធំ ជាងដគ ដហើ យបឋលនឹង រតូវចតជា

ដូដចនោះ

ឧទហរណ្៍ :

ដហើ យបឋលនឹង

។ ដយើ ងទញបាន


យក

ដ ោះ

ឧទហរណ្៍ :

ក្បើ

៕៚

និង

ជាចំនន ួ បឋម និង

ចូរបង្ហាញថា

ចក្មលើយ សដងេតដឃើ ញថា ដបើ

ដ ោះ

អាចមានរាង

លួយចតបុដណ្តណោះ ចដល

និង

។ មានចតករណី អាចជាចំ នន ួ បឋលទំងព ីរបាន។ ដូដចនោះ

6.7 រេឹសីតបេឡឺសង់ ត្តង

ជាចំនន ួ បឋម និងត្តង

ជាព ល ររបស់

កនុងក្គ្នល

។ ចំនន ួ គរ់

ធំបផ ំ រ ុ ដែល

ដចកោច់

កំណ្រ់

ក្ោយ

សរមាយបញ្ញជក់ តាលរទឹសាីបទឌីប៉ាល ៉ូ ីញ ៉ា ក់

ដយើ ងមាន

លឹម សុ វណ្ណវិចរិ រ | 6.7 រេឹសីត បេឡឺសង់

57


ដូដចនោះ

6.8 រេឹសីតបេខាំម័រ (Kummer) ត្តង

ជាចំនន ួ បឋម។ ចំនន ួ គរ់

ធំបផ ំ រ ុ ដែល

ក្សមើ នឹងផលបូកចំនន ួ រត្តេុកកនុងរបមាណ្វិធប ី ូករបស់

ដចកោច់ ក្មគុណ្ក្េវធា និង

សរក្សរកនុងក្គ្នល ។

សរមាយបញ្ញជក់ តាង

និង ។ តាង

យក

ចដល

58

និង

៦ - អនុគមន៍នពវន ត | លឹម សុ វណ្ណវិចរិ រ

កំណត់ ដោយ


ផលបូកម្នចំ នន ួ រតាទុកកំណត់ ដោយ

។ គុណសលភាពទំងអស់ ដនោះបនាប ា ប់ គ្នននឹង

រួចបូកអងគនង ិ អងគ ដយើ ងទញបាន

ដោយបូកសលភាព(*)បញ្ចូ លគ្នន ដយើ ងទញបាន

តាលរទឹសាីបទឡស ួ គត់ ឺ ង់ ចំ នន

ធំ បំផត ុ ចដល

ចចកោច់

កំណត់ ដោយ

6.9 រេឹសីតបេ រគប់ ចន ំ ន ួ គរ់ ធមមជារិ បានទំងអស់ ដែល

សុ េធដរអាចសរក្សរជារាង ។

លឹម សុ វណ្ណវិចរិ រ | 6.9 រេឹសីត បេ

59


ឧទហរណ្៍

60

៦ - អនុគមន៍នពវន ត | លឹម សុ វណ្ណវិចរិ រ


ឡូស៊ិ ច

តាង

ិ មា ជាចំនន ួ គត់ វជ ជ ន។ ចូរបង្ហាញថា រគប់ អងក ត់

និចច។ (ចំនន ួ ស្វ័យគុ ណគោល២ ជាចំនន ួ ដដលមានរាង តាង

ិ មា ជាស្ំ ណុំមិនទគទ ននចំនន ួ គត់ វជ ជ ន ដដល " គបើ

ក៏ជាធាតុរបស្់

ដដរ "។ ចូរបង្ហាញថា

តាងឱ្យដផន កគត់ នន

) ជាធាតុរបស្់

ចំនន ួ ពីរកនង ុ ចំគោមចំនន ួ ទំងគនោះ កំណត់ គោយ មួ យមានធាតុចំនន ួ

និង

ដដល

គផេងោន។ ធាតុទំង

ើ គរ ើស្គោយគស្រ ីពីកង គគគរជស្ នុ ចំគោមចំនន ួ គត់ ចាប់ ពី ដល់ បាន ស្ំ ណុំរងមិនទគទខុស្ោនពីរនន ើ គរ ើស្ គគគរជស្

និង

គគអាចរកបាន ។

គនោះជាចំនន ួ គត់ ធមម ជាតិ ដដល ។ ចូរបង្ហាញថា គគអាចរក

ដដលមានផលបូ កធាតុរបស្់ វាគស្មើោន។

ចំនន ួ គត់ គោយគស្រ ីគចញពី

ពីរកនង ុ ចំគោមចំនន ួ ទំង

គ ោះ

ជាស្ំ ណុំននរគប់ ចំនន ួ គត់ ធមម ជាតិ។ កនង ុ គនោះ

ិ មា ចំនន ួ គត់ វជ ជ នដដលធំមិនគលើ ស្ពី

ចូរបង្ហាញថា កនុងចំគោមបោ ា គគឱ្យស្ំ ណុំ

មានចំនន ួ ស្វ័យគុ ណគោល២ ជា

។ ចូរបង្ហាញថា មានចំនន ួ គត់

ដដលមានផលស្ងគស្មើ

(អាមេរច ិ ១៩៩៤)

គគបិទស្លលកគលខថាស្ចំនន ួ មួ យ គោយគលខ១, ថាស្ចំនន ួ គោយគលខ២, ថាស្ចំនន ួ គោយគលខ៣,…, ថាស្ចំនន ួ

គោយគលខ៥០។ គគោក់ ថាស្ដដលបិទស្លលកទំង

គនោះ គៅកនង ុ របអប់ មួ យ។ គគទញយកថាស្មា ងមួ យៗ គចញពីរបអប់ ិ ។ គតើថាស្តិចបំ ផុតចំនន គនោះ គោយនចដន់ គោយមិនោក់ ចូលវញ ួ បុ ម ន ដដលរតូវទញគចញ ើ បីឱ្យរបាកដថា មានយងតិចថាស្ចំនន មកគដម ួ

មានស្លលកដូចោន?។

លឹម សុ វណ្ណវិចត្ិ រ| ឡូសិុ ច

61


(អន្តរជាតិ ១៩៦៤) មនុស្េ

ក់ ទក់ ទងោនតាមស្ំ បុរត គោយមនុស្េមានក់ ៗ ស្រគស្រគៅកាន់ មនុស្េ

ក់

គទៀតដដលគៅស្ល់ ។ពួ កគគ ទក់ ទងោន គលើ របធានបទចំនន ួ ដតបុ គោ ណ ោះ។ រាល់ បោ ា មនុស្េ ិ គៅមក គលើ របធានបទដតមួ យ បុ គោ ពីរ ក់ ស្រគស្រគៅោនគៅវញ ណ ោះ។ ចូរបង្ហាញថា មាន យងតិចមនុស្េចំនន ួ ៣ ក់ ដដលស្រគស្រស្ំ បុរតគៅកាន់ អនកដនទពីរបធានបទដតមួ យដូច ោន។ ើ គោយគរជស្ ើ គរ ើស្ចំនន ស្ុខ និង គៅគលងដលបងដូចតគៅ។ ពួ កវាចាប់ គផា ម ួ គត់ ើ គរ ើស្យកចំនន ប ា ប់ មកគទៀត ស្ុខគរជស្ ួ គត់

មួ យជាស្មា​ាត់ ដដល

ើ បីឱ្យគៅអាចទយបាន គៅរបាប់ គលខ គៅជាអន កទយគលខស្មា​ាត់ គនោះ។ គដម ិ ថា គ ើ យស្ុខគឆលយ ើ មកវញ

មួ យ ។ ។ មួ យគៅស្ុខ

ជាចំនន ួ បឋមឬអត់ ។ ចូរបង្ហាញថា គៅអាចកំណត់ គលខ

ស្មា​ាត់ របស្់ ស្ុខបានគោយរបាប់ ស្ុខយងគរចើន

ដង។

ចូរបង្ហាញថា រគប់ ឆ្នំទំងអស្់ មាននថាស្ុរកទី១៣ យងគោចោស្់ មា ង។ ើ គរ ើស្យក គគគរជស្

ើ គរ ើស្ ។ ចូរបង្ហាញថា គបើគទោះជាគគគរជស្

ចំនន ួ គត់ គចញពី

យងគមចក៏គោយ ក៏គគគង់ទទួ លបានចំនន ួ ពីរដដលបឋមរវាងោន ដដរ។ (អាមេរច ិ AHSME 1999) ទូ ោក់ អីវាន់ គៅផារមួ យរតូវបានគគបិទស្លលកគលខចាប់ ពីគលខ១គៅ។ តួគលខបាលស្ាិចដដលគរបើ ើ បីបិទ ស្រមាប់ គធវជា ើ ស្លលកគលខបិទគលើ ទូទំងគ ោះ មានតនមល ពរី គស្នកនង ុ មួ យគលខ។ ដូគចនោះគដម ើ បីបិទគលខ១០ គគរតូវចំោយអស្់ បួ នគស្ន។ គលខ៩ គគរតូវចំោយអស្់ ពីរគស្ន គ ើ យគដម គបើស្ិនជាគគរតូវចំោយអស្់

ើ បីបិទស្លលកគលខ គតើមានទូ ទំងអស្់ ចំនន គដម ួ បុ ម ន?។

(អាមេរច ិ AMC12 2001) ិ មា គតើមានចំនន ួ គត់ វជ ជ នចំនន ួ បុ ម នដដលមិនគលើ ស្ពី មិនដមនព ុ គុណនន

62

ឡូសិុ ច |លឹម សុ វណ្ណវិចត្ិ រ

គ ើ យជាព ុ គុណនន ឬ

ដត


(អាមេរច ិ AHSME 1983) តាង ដដលគលខកនង ុ ខា ង់និមួយៗបានមកពីចុោះគលខគត់ ពី ដល់

តាមលំ ោប់ ។ ចូរកំណត់ ខាង់ទី

គរកាយគកបៀស្។ ិ តុមូលមួ យ។ ចូរបង្ហាញថា គទោះជាអងគុ យ គកមងរបុ ស្២៥ ក់ នង ិ គកមងស្ស្ី២៥ ក់ អងគុ យជុំវញ ដបបោ ក៏គង់មានគកមងមានក់ ដដលអន កគៅស្ង្ហាងវាជាគកមងស្ស្ីទំងពីរ ក់ ដដរ។ (អាមេរច ិ AMC12 2001) ើ មួ យនិងដស្បកគជង ើ មួ យស្រមាប់ គជង ើ នីមួយៗ ននគជង ើ ទំងរបាប ពីងពាងមានគរស្លមគជង ំ ីរបស្់ ើ និងដស្បកគជង ើ ទល់ ដតរគប់ គជង ើ តាមលំ ោប់ លំ គោយ វា។ គតើពង ី ពាងអាចពាក់ គរស្លមគជង ឹ ថា រាល់ គជង ើ នីមួយៗរបស្់ វា វារតូវពាក់ គរស្លមគជង ើ មុ ន បានបុ ម នរគបៀបខុស្ៗោន គបើដង ើ ? ដស្បកគជង (អាមេរច ិ AHSME 1986) ិ កនង ើ រក ម ថតតុឋត ុ បនាប់ងងិតមួ យមានគរស្លមគជង ើ គខៀវ គរស្លមគជង

ើ គមម គូ និងគរស្លមគជង

ើ នបតង គូ គរស្លមគជង

គូ

ើ មួ យមា ង គូ ។ យុ វជនមានក់ យកគរស្លមគជង

ើ ទំងគ ោះបានគទ។ គតើយុវជនគ ោះ គចញពីថតតុគនោះ ដតវាមិនអាចស្លគល់ ពណ៌ របស្់ គរស្លមគជង ើ គចញយងតិចចំនន ើ បីឱ្យវាទទួ លបានគរស្លមគជង ើ យងគោច រតូវយកគរស្លមគជង ួ បុ ម នគដម ោស្់

ឹ ថា គរស្លមគជង ើ មួ យគូ ជាគរស្លមគជង ើ ពីរដដលមានពណ៌ ដូចោន។ គូ ? គោយដង

ើ មួ យមិនអាចឱ្យរាប់ គៅកនង ើ គលើ ស្ពីមួយបានគទ។) គរស្លមគជង ុ គូ គរស្លមគជង (អាមេរច ិ ១៩៩១) គគឱ្យចំនន ួ ស្និទនមួ យ គគស្រគស្រវាជាទរមង់បរងួ មរួច ប ា ប់ មកគគគណ ផលគុ ណនន ិ គៅចគ ល ោះ និង ភាគយកនឹងភាគដបង។ គតើមានចំនន ួ ស្និទនចំនន ួ បុ ម នដដលឋត មានផលគុ ណគស្មើ

ដដល

លឹម សុ វណ្ណវិចត្ិ រ| ឡូសិុ ច

63


(អាមេរច ិ ១៩៩៨) ិ មា ននចំនន ួ គត់ វជ ជ នគស្ស្ ដដល

ចូរកំណត់ ចំនន ួ ចតុធាតុ (អាមេរច ិ AIME 1992)

ិ គៅកនង គតើមានគូ ននចំនន ួ គត់ បនាប ា ប់ ោន ចំនន ួ បុ ម ន ដដលឋត ុ ស្ំ ណុំ ិ ប គ ើ យដដលមិនរតូវការរតាទុ ក គពលគធវរើ បមាណវធ ី ូ កននចំនន ួ គត់ ទំងពីរ? (អាមេរច ិ AHSME 1994) គៅអីរបាប ុ ជួរមួ យ ស្ំ រាប់ ទុ កគអាយស្ិស្េ៦ ក់ និងស្លស្ត្ស្លាចារយអាល់ ផា ដបតា និងកា ំ ួ នកនង ើ គរ ើស្គៅអី មា អងគុ យ។ ស្លស្ត្ស្លាចារយទំងបីមកដល់ មុ នស្ិស្េទំង៦ ក់ គ ើ យស្ំ គរចចិតគា រជស្ យងោគអាយស្លស្ត្ស្លាចារយមានក់ ៗស្ថិតគៅចគ ល ោះស្ិស្េ២ ក់ ។ គតើស្លស្ត្ស្លាចារយអាល់ ផា ដបតា ើ គរ ើស្គៅអីខលួនបានបុ ម នរគបៀប? និងកាមា អាចគរជស្ ិ មា ចូរបង្ហាញថាកនង ុ ចំគោមចំនន ួ គត់ វជ ជ ន១៦គផេងោន មានតនមល មិនគលើ ស្ពី១០០ មានចំនន ួ គត់ ៤គផេងោន

ដដល

(អាមេរច ិ ១៩៨៩) កុ មារមានក់ មានឥដឋមួយស្ំ ណុំមាន៩៦ដុំគផេងៗោន។ ដុំនម ី ួ យៗអាចគធវព ើ បា ី ល ស្ាិចឬគឈើ អាចមាន ទំ ំ តូច មធយមឬធំ អាចមានពណ៌ គខៀវ នបតង រក ម ឬគលឿង និងអាចមានរជុងជារងវ ង់ ឆគកាណ កាគរ ឬរតីគកាណ។ គតើមានដុំឥដឋបុ ម នដុំ ដដលមានលកាណៈខុស្ពីឥដឋបាលស្ាិច ទំ ំ មធយមពណ៌ រក មរាងរងវ ង់ រតង់ពីរចំណុចគត់ ? (ឥដឋគឈើ ទំ ំ មធយមពណ៌ រក មរាងកាគរ ជាឧទ រណ៍ មួ យ)។ (អាមេរច ិ ១៩៩៨) គគគៅគលខទូ រស្ពា ៧ខា ង់ ដូចោននឹង

64

ឡូសិុ ច |លឹម សុ វណ្ណវិចត្ិ រ

ថាជាគលខពិគស្ស្ គបើលំោប់ លំ គោយ ឬ

(ដូចទំងពីរក៏បាន)។


ឧទ រណ៍ តនមល ជាគលខ

ស្ុទធដតជាគលខពិគស្ស្។ ស្នមតថា

នីមួយៗអាចមាន

។ ចូរកំណត់ ចំនន ួ គលខពិគស្ស្ទំងអស្់ ដដលអាចមាន។

(អាមេរច ិ ១៩៩៦) រកឡាពីរននរកឡាទំង

របស្់ កា​ារអុ កមួ យ រតូវបានគគលាបពណ៌ គលឿង គ ើ យ

រកឡាគៅស្ល់ រតូវលាបពណ៌ នបតង។ កា​ារអុ កពណ៌ ពីរ ដដលលាបពណ៌ តាមរគបៀបគនោះ ស្ម មូ លោន គបើកា​ារពណ៌ មួ យអាចទទួ លបានគោយបងវ ិលកា​ារពណ៌ មួ យគទៀតកនង ុ បល ង់កា​ារអុ ក។ គតើ អាចមានកា​ារអុ កពណ៌ មិនស្មមូ លោនចំនន ួ បុ ម ន?។ ិ គី រៀបគលខ គតើគគមានវធ

និង

ើ បីឱ្យផលបូ ក ចំនន ួ បុ ម នរគបៀប គដម

គលខ៤គៅជាប់ ោនដចកោច់ នឹង៣?។ (អាមេរច ិ ១៩៩៣) តាង

ើ គរ ើស្ស្ំ ណុំរង ជាស្ំ ណុំមួ យមានធាតុ៦។ គតើគគមានបុ ម នរគបៀបគផេងោនកនង ុ ការគរជស្ ដដលមិនចាំបាច់ ខុស្ោនក៏បាន គ ើ យដដលរបជុំននស្ំ ណុំរងទំងពីរជា

ពីរនន

ើ គរ ើស្មិនស្ំ ខាន់ គទ ឧទ រណ៍ គូ ស្ំណុំរង ការគរជស្ ើ គរ ើស្គូ គរជស្

? លំ ោប់ នន

ដូចោននឹងការ

ដដរ។

(ឥណ្ឌ ា ១៩៩៨) ិ មា គគឱ្យចំនន ួ គត់ វជ ជ ន

ចូរបង្ហាញថា ចំគពាោះរគប់

។ តាង

បោ ា ផលគុ ណ

ស្ុទធដតមានតនមល ខស្ ុ ោនទំងអស្់ ។

ដលបងគបើកគបៀកនង ុ កុ ព ំ យូទ័រមួ យគគគលងដតមានក់ ឯង ដូចតគៅ។ គរកាយគលងជាប់ មួ យគលើ កៗ គៅតាមលទធ ផលដដលទទួ លបាន អន កគលងទទួ លបាន

ពិនាុ (

) គ ើយ

ឹ ថា មានពិនច ពិនរាុ បស្់ ោត់ គកើនគ ើ ងគោយបូ កបដនថមនឹងពិនគាុ លើ កមុ ន។ គគដង ាុ ំនន ួ របគភទដដលគគមិនអាចទទួ លបាន គ ើ យកនង ុ ចំគោមគ ោះ មានមួ យគស្មើ និង

។ ចូរគណ

។ លឹម សុ វណ្ណវិចត្ិ រ| ឡូសិុ ច

65


(អាមេរច ិ ១៩៩៤) គគមានឥដឋចំនន ួ

ដុំ។ មួ យដុំៗមានរង្ហវស្់

បគង្ហគលមួ យមានកមព ស្់

គគយកមកតំគរៀបពីគលើ ោនជា

ដុំឥដឋ។ ឥដឋនម ី ួ យៗអាចតំគរៀបគោយយករជុង

ស្ស្បនឹងកមព ស្់របស្់ បគង្ហគល ។ គតើមានកមព ស្់បគង្ហគលស្រុបបុ ម នរបគភទដដលគគអាចគធវបា ើ ន គោយគរបើឥដឋទំង

ដុំគនោះ?។

(អន្តរជាតិ ១៩៧០)

ិ មា ចូរកំណត់ រគប់ ចំនន ួ គត់ វជ ជ ន

ដដលគធវឱ្ ើ យស្ំ ណុំ

អាចជាបំ ដបកជាស្ំ ណុំរងពីរ ដដលផលគុ ណននបោ ា ចំនន ួ ទំងអស្់ គៅកនុងស្ំ ណុំមួ យ គស្មន ើ ង ឹ ផលគុ ណននបោ ា ចំនន ួ ទំងអស្់ គៅកនង ុ ស្ំ ណុំមួ យគទៀត។ គតើគគរតូវដចកបូ លចំនន ួ

គៅកនង ុ របអប់ ចំនន ួ

ើ បីឱ្យ ផលបូ កននចំនន យងគមច គដម ួ បនេំបូ

លពីរៗកនង ុ របអប់ នីមួយៗ បូ កចូលោននឹងរបអប់ ទំងអស្់ គផេងគទៀត មានចំនន ួ តិចបំ ផុត?

66

ឡូសិុ ច |លឹម សុ វណ្ណវិចត្ិ រ


ប្រព័ ន្រធ បារ់ .ចំ ន្ន្ ួ គត់ .ភាព ចចកដាច់ ត្រព័ន្រធ បារ់ 32.

(អន្តរជាត១ ិ ៩៧៥) ពពលពេសរពសរ

នីមួយៗរបស់ វា ពសមើ

ពៅក្នង ុ របព័នពធ ោល

។ តាង

បូ ក្តួពលខននខទ ង់នីមួយៗរបស់

, ពេទទួ លបានផលបូ ក្តួពលខននខទ ង់

ជាផលបូ ក្តួពលខននខទ ង់នីមួយៗរបស់

។ ចូរេណនា ផល

។(

)។

សរពសរពៅក្នង ុ របព័នពធ ោល

33.

ចូរេណនា ពលខ២ខទ ង់ចុងពេរបស់

34.

ចូរេណនា ពលខ២ខទ ង់ចុងពេរបស់

35.

ចំនន ួ េត់ មួ យថយចុ​ុះជាេត់ ដង ពពលពេលុ បតួពលខខាងចុងពេពោល។ ចូរេណនាចំនន ួ បបបពនុះ។

36.

ើ ពោយពលខ ចូរក្ំណត់ រេប់ ចំនន ួ េត់ ធមម ជាតិបដលពផត ម

និង ថយចុ​ុះ

ដង ពបើពលខ

ពនុះ

រតូវបានលុ បពោល។ 37.

(អន្តរជាតិ ១៩៦៨)

ចូរក្ំណត់ រេប់ ចំនន ួ េត់ ធមម ជាតិ

បដលពផទ ៀងផ្ទទត់ ផលេុ ណននតួពលខខទ ង់នីមួយៗរបស់ វា (ពៅ

ក្នង ុ ពោលដប់ ) ពសមើនង ឹ 38.

(អាមេរច ិ ១៩៩២)

តាង S ជាសំ ណុំននរេប់ ចំនន ួ សនិទាន

បដល

ព ើ យបដលមានតួពលខជាខួបរាង

លឹម សុ វណ្ណវិចត្ិ រ| ត្រព័ន្រធ បារ់

67


បដលតួពលខ

មិនោំខស ុ ោនក្៏បាន។ ពបើរបភាេនីមួយៗបដលជាធាតុរបស់

សរមួ លរួច

ព ើ យ ពតើមានភាេយក្ខុសៗោនចំនន ួ បុ នាមន?។ ិ មា ចូរបង្ហាញថា រេប់ ចំនន ួ េត់ វជ្ ជ ន បតងមានព ុ េុណរបស់ វាមួ យ បដលពលខរបស់ វាក្នង ុ

39.

របព័នទ ធ សក្ៈ (របព័នពធ ោល

) មានរេប់ តួពលខពី

ដល់

ពលខ

40.

ទទួ លបានពោយពរៀបចំនន ួ េត់ ជាអនុេមន៍បដលខទ ង់ទី

តពរៀងោនបនតបនាទប់ ោន។ ពយើ ងតាង

ិ ពៅចំចំណុចបដលពេបបនថមពលខមាន ឱ្យ ឋត សថិតពៅចំពលខ 41.

ចូរសរពសរ

42.

ចូរសរពសរចំនន ួ ទសភាេ

43.

(អាមេរច ិ ១៩៩៤)

បដលមាន

ពៅក្នង ុ ពោល

ខទ ង់ ។ ឧទា រណ៍ ខទ ង់។ ចូរេណនា

ពៅក្នង ុ ពោល

តាង

។ ។

ជាចំនន ួ េត់ ធមម ជាតិ បដល

ខទ ង់ចុងពេរបស់

)។ តាង

(អន្តរជាតិ ១៩៧៨)

44.

ពររុះ ខទ ង់ទី

ជាផលេុ ណននតួពលខខុសពីសូនយរបស់

មានបតមួ យខទ ង់ ពនាុះ

ចូរក្ំណត់ ក្តាតបឋមធំបំផុតរបស់

របស់ ពលខបដល

ិ មា ពេឱ្យចំនន ួ េត់ វជ្ ជ ន , តាង (ពបើ

។ ពៅក្នង ុ របព័នរធ បាប់ ពោល

ពសមើនង ឹ ពលខបីខទង់ចុងពេរបស់

។ ចូរេណនា

ពលខបី ពពល

មានតនមល តច ូ បំ ផុត។ ចូរបង្ហាញថា ពលខមួ យរន់ ខទង់ដំបូងពរោយពក្បៀសរបស់

45.

ទាំងអស់ ។

ចំន្ន្ ួ គរ់ ចូរបង្ហាញថា

46.

68

ត្រព័ន្រធ បារ់ .ចំន្ន្ ួ គរ់ .ភាពចចកដាច់ |លឹម សុ វណ្ណវិចត្ិ រ

សុទធបតជាពលខ


ជាចំនន ួ េត់ ចំពរុះរេប់ ចំនន ួ េត់ ធមម ជាតិ 47.

តាង

ិ មា ជាចំនន ួ េត់ មិនអវជ្ ជ ន។ ចូរបង្ហាញថា

បដល

ជាចំនន ួ េត់ ។ 48.

ចូរបង្ហាញថា ផលបូ ក្

មិនអាចជាចំនន ួ េត់ ពទ។ 49.

(អាមេរច ិ ១៩៩៣) ពតើមាន

រជ្ុង មានមុំ ក្ង នុ ជាចំនន ួ េត់ េត ិ ជាដ ឺ

ចំនន ួ បុ នាមន បដលព ុ ពោណនិយ័តមួ យ មាន

ពរក្?។

ភាពចចកដាច់ 50.

ចូរបង្ហាញថា ពបើ ដូពចនុះ ពបើ

ពសស ពនាុះ

ពសស ពនាុះ

បចក្ោច់

51.

ចូរបង្ហាញថា

52.

ចូរបង្ហាញថា ចំពរុះចំនន ួ េត់ ធមម ជាតិ

បចក្ោច់ មួ យ ពេមាន ចំនន ួ េត់ ធមម ជាតិ

នីមួយៗរបស់ សវុីត 53.

ិ មា េណនារេប់ ចំនន ួ េត់ វជ្ ជ ន

54.

ពបើ

55.

ចូរបង្ហាញថា ពបើ

បចក្ោច់

បចក្ោច់ នឹង បដល

ចូរបង្ហាញថា

ិ មា េណនារេប់ ចំនន ួ េត់ វជ្ ជ ន

57.

ិ មា ចូរក្ំណត់ ចំនន ួ េត់ វជ្ ជ ន

58.

ចូរបង្ហាញថា

បចក្ោច់ នឹង

បចក្ោច់

ពនាុះ

បដល បដល

មួ យពទៀត បដលតួ

បចក្ោច់

ជាព ុ េុណនន

ិ ។ រោសមក្វញ 56.

ជាព ុ េុណនន បចក្ោច់

បចក្ោច់

បដរ និង

។ ។

ចំពរុះចំនន ួ េត់

ពរចើនរាប់ មិនអស់ ។

លឹម សុ វណ្ណវិចត្ិ រ| ភាពចចកដាច់

69


59.

ចូរបង្ហាញថា ពបើ

ជាចំនន ួ បឋម ពនាុះ

60.

ចូរបង្ហាញថា ចំពរុះរេប់

61.

ចូរបង្ហាញថា ផលេុ ណនន

62.

ចូរបង្ហាញថា

63.

តាង

បចក្ោច់

បចក្ោច់

ចំនន ួ េត់ បនតបនាទប់ ោន បចក្ោច់ នឹង

បចក្ោច់

ចំពរុះរេប់ ចំនន ួ េត់

ជាចំនន ួ បឋម។ ចូរបង្ហាញថា

ជាព ុ េុណនន

ើ ពរ ើសយក្ពីរ តាងពោយ ចូរបង្ហាញថា ពីក្ង នុ ចំពោមចំនន ួ េត់ បីោក្៏ពោយ ពេអាចពរជ្ស

64.

បដលពផទ ៀងផ្ទទត់ 65.

ចូរបង្ហាញថា ពបើ

66.

តាង

បចក្ោច់ នឹង បចក្ោច់

ជាចំនន ួ េត់ ។ បង្ហាញថា

ពនាុះ

បចក្ោច់

បចក្មិនោច់ នឹង

ផងនិង

ពបើ

ផង ។

បចក្ោច់ នឹង

និងរោស

ិ ។ មក្វញ (ហុ ងគ្រី ១៨៩៩)

67.

ចូរបង្ហាញថា ធមម ជាតិ

បចក្ោច់ នឹង

68.

ចូរបង្ហាញថា

69.

ចូរបង្ហាញថា ពបើ

ជាចំនន ួ េត់ ធមម ជាតិពសស ពនាុះ

ចំនន ួ េត់ ធមម ជាតិ

ជាព ុ េុណ នន

(រុស្ុ ី ១៩៩៥)

70.

ចូរបង្ហាញថា មានចំនន ួ េត់ ព ុ េុណ តាង

71.

ដូច

ិ មា ជាចំនន ួ េត់ វជ្ ជ ន និង

បចក្ោច់

ពរចើនរាប់ មិនអស់ បដល

ចំពរុះរេប់

បចក្ោច់

ជាចំនន ួ មួ យបដលសរពសរពោយពរបើតពួ លខនិងចំនន ួ ខទ ង់ ពនាុះ

(អាមេរច ិ ១៩៩២)

បោ ត ពលខមានពីរខទ ង់ ោប់ ពី

ពៅ

ើ បី រតូវបានពេយក្ពៅសរពសរបនតបនាទប់ ោន ពដម

បពងក ើតបានជាចំនន ួ េត់

70

ចំពរុះ រេប់ ចំនន ួ េត់ ធមម ជាតិ

បតតាមលំ ោប់ លំ ពោយមួ យពផេងពទៀត។ ចូរបង្ហាញថា ពបើ

បចក្ោច់ នឹង 72.

ចំពរុះ រេប់ ចំនន ួ េត់

ត្រព័ន្រធ បារ់ .ចំន្ន្ ួ គរ់ .ភាពចចកដាច់ |លឹម សុ វណ្ណវិចត្ិ រ


ក្ំណត់

ធំបំផុត បដល

73.

បង្ហាញថា ពបើ

74.

ចូរបង្ហាញថា ពបើ

ពនាុះ 75.

បចក្

ជាចំនន ួ បឋម ពនាុះ

បចក្ោច់

ជាចំនន ួ ពសសបឋម ព ើ យពបើ

ោច់ ។

ចូរក្ំណត់ បោ ត ចំនន ួ េត់ បចក្ោច់

76.

បចក្ោច់ ចំនន ួ ពនុះ។

បដល

បចក្ោច់

បចក្ោច់

និង

(អន្តរជាតិ ១៩៩៤)

ិ មា ចូរក្ំណត់ រេប់ ចំនន ួ េត់ វជ្ ជ នោច់ ខាត

បដល

ជាចំនន ួ េត់ ។ 77.

ចូរក្ំណត់ រេប់ ចំនន ួ េត់

78.

(បាល់កង់ ១៩៩៦) ពេឱ្យចំនន ួ បឋម

បដល

បចក្ោច់

។ ពេក្ំណត់

ចូរបង្ហាញថា មាន

និង

ជាធាតុរបស់

បដល

ព ើយ

79.

បង្ហាញថា

ជាចំនន ួ េត់ ធប ំ ំ ផុតបដលបចក្ោច់ នឹងរេប់ ចំនន ួ េត់

80.

ិ មា ចំនន ួ េត់ វជ្ ជ ន

មានលក្ខណៈ

<<ចំពរុះរេប់

, ពេមាន

ិ មា ចូរក្ំណត់ រេប់ ចំនន ួ េត់ វជ្ ជ ន

82.

(អន្តរជាតិ ១៩៩២) ចូរក្ំណត់ រេប់ ចំនន ួ េត់ បចក្ោច់

បដល បដល

បដល បចក្មិនោច់ នឹង >>

ចូរបង្ហាញថា លក្ខណៈពនុះ អាចពក្ើតមានបតក្នង ុ ក្រណី 81.

បចក្ោច់

ជាសវ័យេុ ណនន

បចក្ោច់

មួ យបតបុ ពោ ណ ុះ។

។ និង

។ លឹម សុ វណ្ណវិចត្ិ រ| ភាពចចកដាច់

71


83.

ិ មា ចូរបង្ហាញថា ចំពរុះរេប់ ចំនន ួ េត់ េូវជ្ ជ ន

84.

(អន្តរជាតិ ១៩៨៤)

ិ មា ចូរឱ្យឧទា រណ៍ េូ ចំនន ួ េត់ វជ្ ជ ន (i)

ពយើ ងមាន

បចក្ោច់

មួ យ បដលពផទ ៀងផ្ទទត់ លក្ខខណឌទាំងពីរខាងពរោម

បចក្មិនោច់ នឹង

(ii)

បចក្ោច់ នឹង

ចូរពផទ ៀងផ្ទទត់ ចពមល ើយទទួ លបាន។ 85.

តាង

ជាចំនន ួ បឋម។ ចូរបង្ហាញថា

86.

ពេឱ្យចំនន ួ បឋម ចំនន ួ បចក្ោច់ នឹង

បចក្ោច់

។ ចូរបង្ហាញថា

ចំពរុះរេប់ ចំនន ួ េត់ , បដលមានពលខ

ចំនន ួ

និង

ដង, ជា

87.

បង្ហាញថា

88.

ពេឱ្យចំនន ួ បឋមពសស

ពេមាន

បចក្ោច់ នឹង

។ ចូរបង្ហាញថា មានចំនន ួ េត់

ជានិចច។

ពរចើនរាប់ មិនអស់ បដល

បចក្ោច់

។ 89.

ចូរបង្ហាញថា ោមនចំនន ួ េត់

90.

ពតើ

បចក្ោច់

បដល

បចក្ោច់

ពទ។

ឬពទ?។

ើ ពរ ើសយក្ចំនន ពេពរជ្ស ួ េត់ ចំនន ួ

91.

ើ ពរ ើសចំនន ពរជ្ស ួ េត់ ទាំង

ពីក្ង នុ ចំពោម

។ ចូរបង្ហាញថា ពទាុះពេ

ពនាុះយងពមចក្៏ពោយ ក្៏ពេ​េង់ទទួ លបាន ចំនន ួ មួ យក្នង ុ

ចំពោមពនាុះ បចក្ោច់ ចំនន ួ មួ យពទៀតបដរ ។

សំណ្ល់. ភាពសមមូល តាង

92.

ជាចំនន ួ បឋមមួ យ។ ចូរបង្ហាញថា

i) ii) 93.

តាង

94.

(អាមេរច ិ ១៩៩៤) 72

ជាចំនន ួ បឋម បដល

។ ចូរបង្ហាញថា

ត្រព័ន្រធ បារ់ .ចំន្ន្ ួ គរ់ .ភាពចចកដាច់ |លឹម សុ វណ្ណវិចត្ិ រ


តាង

ិ បី ចក្ ជាសំ ណល់ ននវធ

និង

នឹង

។ េណនា

ចំន្ន្ ួ ការរ 95.

ចូរបង្ហាញថា ចំនន ួ ោពរមានរាង

96.

ចូរបង្ហាញថា ោមនចំនន ួ េត់ ពៅក្នង ុ សវុីត

97.

ចូរបង្ហាញថា ពេមានចំនន ួ េត់ ធមម ជាតិ

98.

ចូរបង្ហាញថា ក្នង ុ រេប់ របព័នរធ បាប់ ទាំងអស់

99.

ចូរក្ំណត់ ចំនន ួ េត់

100.

ចូរក្ំណត់ រេប់ ចំនន ួ េត់

101.

(អន្តរជាតិ ១៩៨៨) ពបើ

102.

បដល

ពេឱ្យ

។ ជាចំនន ួ ោពរពទ។ មួ យេត់ បដល

ជាចំនន ួ ោពរ។

សុទធបតជាោពរននចំនន ួ សនិទាន ។

ជាចំនន ួ ោពរ។

បដល

ិ មា ជាចំនន ួ េត់ វជ្ ជ ន បដល

ចូរបង្ហាញថា 103.

ជាចំនន ួ ោពរ។

ជាចំនន ួ េត់ ពនាុះ

ជាចំនន ួ េត់ បដល

ជាចំនន ួ ោពរ។

និង

ជាចំនន ួ ោពរ។

ក្) ចូរបង្ហាញថា ផលេុ ណននចំនន ួ េត់ ពីរតោនមិនអាចជាចំនន ួ េត់ ោពរពទ។ ខ) ចូរបង្ហាញថា ផលេុ ណននចំនន ួ េត់ បីតោនមិនអាចជាចំនន ួ ោពរពទ។ េ) ចូរបង្ហាញថា ផលេុ ណននចំនន ួ េត់ បួ នតោនមិនអាចជាចំនន ួ ោពរពទ។

104.

ចូរបង្ហាញថា ផលេុ ណននចំនន ួ េត់ បីតពរៀងោន មិនអាចជាចំនន ួ សវ័យេុ ណពទ (មានន័យថា មិន អាចជាចំនន ួ ោពរ, ចំនន ួ េូ ប។ល។)

105.

ចូរក្ំណត់ ចំនន ួ មិនោពរទី ជាចំនន ួ មិនោពរទី

106.

។ឧទា រណ៍

ជាចំនន ួ មិនោពរទី ,

ជាចំនន ួ មិនោពរទី ,

។ល។

តាង

ិ មា ។ ចូរបង្ហាញថា ចំពរុះរេប់ ចំនន ួ េត់ វជ្ ជ ន

, សវុីត

មានចំនន ួ ោពរយងពោចោស់ មួ យ។ 107.

ចូរក្ំណត់ រេប់ ចំនន ួ េត់

បដល

ជាចំនន ួ ោពរ។

លឹម សុ វណ្ណវិចត្ិ រ| ចំន្ន្ ួ ការរ

73


108.

ចូរបង្ហាញថា ពៅក្នង ុ រេប់ សំ ណុំទាំងអស់ បដលមាន

ធាតុ, បដលធាតុនម ី ួ យៗ ជាចំនន ួ េត់ ខុ

សៗោន ព ើ យចំនន ួ េត់ នម ី ួ យៗមានក្តាតបឋមជាធាតុរបស់ សំ ណុំ

; ពេមាន

ធាតុពរី បដលផលេុ ណរបស់ វាជាចំនន ួ ោពរ។ 109.

(អន្តរជាតិ ១៩៨៥) ពេឱ្យសំ ណុំ ធំជាង

ិ មា ធាតុជាចំនន ួ េត់ វជ្ ជ នខុសៗោន។ ោមនធាតុោមួ យមានក្តាតបឋម

មាន

ពទ។ ចូរបង្ហាញថា M មានសំ ណុំរងមួ យ បដលមាន

ជាចំនន ួ សវ័យេុ ណទី

ធាតុខស ុ ៗោន បដលផលេុ ណ

ចំន្ន្ ួ ចែលមាន្រាងណាមួយ 110.

ចូរបង្ហាញថា រេប់ ចំនន ួ េត់ ធមម ជាតិ ជាង

111.

សុទធបតអាចសរពសរជាផលបូ ក្ននចំនន ួ េត់ ពរី ធំ

និង បឋមរវាងោន ។

(អាមេរច ិ ២០០១) ិ មា ពតើមានចំនន ួ េត់ វជ្ ជ នបដលជាព ុ េុណនន

ចំនន ួ បុ នាមន បដលអាចសរពសរជារាង

បាន បដល និង ជាចំនន ួ េត់ ព ើ យ 112.

?។

( អាមេរច ិ ១៩៨៦) សវុីតពក្ើន

ិ មា មានរេប់ ចំនន ួ េត់ វជ្ ជ ន បដលជាសវ័យេុ ណនន ចូរេណនា តួទី 113.

ននសវុីតពនុះ។

(អាមេរច ិ ១៩៧៨) ចំនន ួ េត់

បាន ពោយ 74

ឬជាផលបូ ក្ននសវ័យេុ ណនន ពផេងោន។

ពេពៅវាថាជាពលខពិពសស ពបើពេអាចសរពសរជា ិ មា ជាចំនន ួ េត់ វជ្ ជ ន(មិនោំបាច់ ខុសោនក្៏បាន) បដល

ត្រព័ន្រធ បារ់ .ចំន្ន្ ួ គរ់ .ភាពចចកដាច់ |លឹម សុ វណ្ណវិចត្ិ រ


ឹ ថា ចំនន ពោយដង ួ េត់ ោប់ ពី ចំនន ួ េត់ ទាំងអស់ បដល 114. (អន្តរជាតិ ១៩៨៣) ិ មា ពេឱ្យចំនន ួ េត់ វជ្ ជ ន

ដល់

សុទធបតជាពលខពិពសសទាំងអស់ ចូរបង្ហាញថា រេប់

សុទធបតជា ពលខពិពសស។

និង

បឋមនឹងោនពីរៗ។ ចូរបង្ហាញថា

ជាចំនន ួ េត់ ធប ំ ំ ផុត បដលមិនអាចសរពសរជារាង

បាន បដលក្នង ុ ពនាុះ

ិ មា ជាចំនន ួ េត់ វជ្ ជ នឬសូនយ ។ 115.

ិ មា ចូរបង្ហាញថា រេប់ ចំនន ួ សនិទានវជ្ ជ ន សុទធបតអាចសរពសរជារាង

បានទាំងអស់ ចំពរុះចំនន ួ េត់

ិ មា វជ្ ជ ន។

លឹម សុ វណ្ណវិចត្ិ រ| ចំន្ន្ ួ ចែលមាន្រាងណាមួយ

75


76

ត្រព័ន្រធ បារ់ .ចំន្ន្ ួ គរ់ .ភាពចចកដាច់ |លឹម សុ វណ្ណវិចត្ិ រ


ពហុ គុណរួមតូ ចបំ ផុត. តួ ចចករួមធំ បំផុត ចំនន ួ បឋម 116.

(ចំនន ួ មែរមែន) ចូរបង្ហាញថា បបើ

117.

ជាចំនន ួ បឋម ប ោះ

ចូរបង្ហាញថា មានគូ ប ើយ

ក៏ជាចំនន ួ បឋមដែរ។

បរចើនរាប់ មិនអស់ ដែល

និង

និង

មានកត្តាបឋមរួម

មានកត្តាបឋមរួមគ្ននដែរ។

118.

ចូរកំណត់ រគប់ ចំនន ួ បឋមដែលមានរាង

119.

ចូរបង្ហាញថា

120.

ចូរកំណត់ រគប់ ចំនន ួ គត់

121.

ឹ ថា បគែង

122.

ចូរគណ ចំនន ួ ននចំនន ួ បឋមដែល

123.

(រម ៉ា៉ូ ៉ា នី ២០០៣)

ចំប ោះចំនន ួ គត់

ជាចំនន ួ បឋម ដតកនង ុ ករណី ដែល

ប៉ុ ប្ ណ ោះ ចំប ោះ

ជាចំនន ួ បឋម។

មានកត្តាបឋម

បគឱ្យចំនន ួ បឋម

។ គណ

កត្តាបឋមបនោះ។

។ ។ ចូរបង្ហាញថា បបើ

ដចកដាច់

ប ោះកនង ុ ចំប្មចំនន ួ ទំងប ោះបគអាចរកបានចំនន ួ បឋមបីតបរៀងគ្នន។

ចំនន ួ ពហុ គណ្ ុ 124.

ចំប ោះរគប់ ចំនន ួ គត់

បគអាចរកបានចំនន ួ គត់

មួ យ ដែលប្ ា ចំនន ួ

ស៉ុទធដតជាចំនន ួ ព ៉ុ គ៉ុណ។ លឹម សុ វណ្ណវិចត្ិ រ| ចំនន ួ បឋម

77


ពហុ គណ្ ុ រួមរូចបំផរ ុ រួចចករួមធំបផ ំ រ ុ 125.

ត្តង

។ ចូរបង្ហាញថា

126.

ត្តង

127.

(អាមែរច ិ ១៩៨៥)

ិ មា ជាចំនន ួ គត់ វជ្ ជ ន។ ចូរបង្ហាញថា

ប្ ា ចំនន ួ បៅកនង ុ សវុីត

មានរាង

ចំប ោះ

។ គណ

នីមួយៗ ត្តង

ចូរបង្ហាញថា បបើ

129.

ចូរបង្ហាញថា មានសវុីតនពវ នដា វងមួ យ ដែលតួរបស់ វានីមួយៗបឋមរវាងគ្ននពីរៗ។

130

ិ មា បតើមានចំនន ួ គត់ វជ្ ជ ន

131.

ត្តង

132.

បគកំណត់ ចំនន ួ ដភមាទី

និង

ជាចំនន ួ បសស ប ោះ

ចំនន ួ ប៉ុ ម ន ដែលបឋមនឹង

ជាចំនន ួ បឋមនឹងគ្នន។ ចូរបង្ហាញថា

ចូរបង្ហាញថា ចំនន ួ 133.

ជាចំនន ួ គត់ ធមម ជាតិ ប ើ យ

128.

.

និង

និង

បដាយរូបមនា

?។ ក៏បឋមនឹងគ្ននដែរ។

ទំងអស់ បឋមនឹងគ្ននពីរៗ។

(អនតរជាតិ ២០០៥) បគឱ្យសវុីត ដែល

កំណត់ បដាយ

ិ មា ិ មា ជាចំនន ួ គត់ វជ្ ជ ន។ ចូរកំណត់ រគប់ ចំនន ួ គត់ វជ្ ជ ន ដែលបឋមនឹងរគប់ តួទំងអស់ នន

សវុីត។

ិ មា ននចំនន ួ គត់ វជ្ ជ នចំនន ួ ប៉ុ ម ន ដែល

134.

គណ គូ

135.

(ក៉ូមរ ៉ា ១៩៩៨)

ិ មា ចូរកំណត់ រគប់ ចំនន ួ គត់ វជ្ ជ ន

បឋមនឹងគ្ននពីរៗ ដែល

ជាចំនន ួ គត់ ។ 136.

ិ មា បគឱ្យចំនន ួ គត់ វជ្ ជ ន មានតនមល តច ូ ជាង

78

។ ចូរបង្ហាញថា ។ កនង ុ បនោះ

និង

របស់ ប្ ា ផលគ៉ុ ណ ជាអបថរ និង

ពហុ គណ្ ុ រួមរូចបំផរ ុ រួចចករួមធំបផ ំ រ ុ |លឹម សុ វណ្ណវិចត្ិ រ

ិ មា ជាចំនន ួ គត់ វជ្ ជ នបផទ ៀងផ្ទទត់


137.

(អនតរជាតិ ១៩៥៩) ចូរបង្ហាញថា របភាគ

ជារបភាគមិនអាចសរមួ លបានចំប ោះរគប់ ចំនន ួ គត់ ធមម ជាតិ 138.

(អាមែរច ិ ១៩៧២) ចូរបង្ហាញថា

139.

ចូរគណ ចំនន ួ ននចត៉ុធាត៉ុ

ដែល

បំចបកជាកត្តាបឋម 140.

ិ មា គណ ផលគ៉ុ ណននរគប់ ប្ ា តួដចកវជ្ ជ នរបស់

141.

ិ មា គណ ផលបូ កននតួដចកវជ្ ជ នគូ របស់

142.

(អាមែរច ិ ១៩៨៨)

។ ។

ិ មា ើ បរ ើសបដាយនចែន់ នវូ តួដចកវជ្ ចូរគណ របូ បាបី លីបត កនង ុ ការបរជ្ស ជ នរបស់ ចំនន ួ ព ៉ុ គ៉ុណនន 143.

បានជា

បយើ ងបៅថាចំនន ួ គត់ ធមម ជាតិមួយជាែ​ែបុណ្ណមេខ បបើចំនន ួ ប ោះបសមន ើ ង ឹ ផលបូ កតួដចក ិ មា វជ្ ជ នរបស់ វា បរៅពីខ្លួនវា។ ឧទ រណ៍ ។

ជាសមប៉ុណណ បលខ្ បរ ោះ

ចូរបង្ហាញថា ចំនន ួ គូ មួយជាសមប៉ុណណ បលខ្ ល៉ុ ោះរត្តដត វាមានរាង

ដែល

និង

ិ ។ ជាចំនន ួ បឋម, និងរាសមកវញ

លឹម សុ វណ្ណវិចត្ិ រ| បំចបកជាកត្តាបឋម

79


80

បំចបកជាកត្តាបឋម |លឹម សុ វណ្ណវិចត្ិ រ


ផ្នែកគត់

144.

ចូរបង្ហាញថា ចំព ោះ

ពគមាន

145.

គណនាផ្ផែ កគត់ របស់

146.

ចូរបង្ហាញថា តួរបស់ សវវ៊ីតននចំនន ួ គត់

ផ្ែល

ិ មា អវជ្ ជ ន ជាចំនន ួ គូ នង ិ ពសសឆ្លលស់ គ្នែ។ 147.

(អាមេរច ិ ១៩៨៧) ិ មា ចូរកំណត់ ចំនន ួ គត់ វជ្ ជ ន

148.

ើ ប៊ីឱ្យពគមានចំនន ធំបំផុត ពែម ួ គត់

(អាមេរច ិ ១៩៩៧) សនមតថា

149.

ជាចំនន ួ គត់ មិន

ិ មា ជាចំនន ួ វជ្ ជ ន ផ្ែល

(អាមេរច ិ ២០០៣) ិ មា គណនាចំនន ួ គត់ វជ្ ជ ន

150.

កំណត់ ពហុ ធាមិនសូនយ

151.

(អន្តរជាតិ ១៩៦៨)

និង

។ ចូរគណនា

ិ ពៅជ្ត

ផ្ែល

មួ យគត់ ផ្ែល

បំ ផុត។

មួ យ ផ្ែល

ចំព ោះរគប់ ចំនន ួ ពិត

ចូរគណនា

152.

(សេភាពហ ចូរបង្ហាញថា ពបើ

៊ែរេត ី ) ជាចំនន ួ ពិត និង

ជាចំនន ួ គត់ ធមម ជាតិ ពនាោះ លឹម សុ វណ្ណវិចត្ិ រ| ផ្នែកគរ់

81


(អាមេរច ិ ១៩៨៥)

153.

ិ មា ចំនន ួ គត់ វជ្ ជ នែំបូង ពតើមានបុ នាមនផ្ែលអាចសរពសរជារាង

ពតើកង ែវ ចំពោម

បាន? (អាមេរច ិ ១៩៩១)

154.

សនមតថា

ចូរគណនា

ជាចំនន ួ ពិត ផ្ែល

155.

ចូរកំណត់ ចំនន ួ តួផ្ែលមានតនមល ខស ុ គ្នែកែង វ សវវ៊ីតខាងពរោម

156.

តាង ព៊ីរៗមានតនមល ធជា ំ ង

82

ផ្នែកគរ់ |លឹម សុវណ្ណវិចត្ិ រ

ជាចំនន ួ គត់ ផ្ែលពហុ គុណរួមតូចបំ ផុតននរាល់ បោ ា ។ ចូរបង្ហាញថា


សមី ការដ្យូផង់ 157.

ចូរគណនាចំនន ួ គត់

ដែលផផទ ៀងផ្ទទត់ សមីការ

158.

គណនាចផមល ើយសមីការែយូផង់ :

159.

ផតើមានចំនន ួ គត់

ដែល

160.

ចូររកចផមល ើយជាចំនន ួ គត់ ននសមីការ

161.

ចូរបង្ហាញថា សមីការ

162.

សនាតថា

163.

គណនាចំនន ួ គត់

164.

ចូរបង្ហាញថា សមភាព

ិ មា គ្មានឫសជាចំនន ួ គត់ វជ្ ជ នផទ។

ជាចំនន ួ គត់ ដែល

ករណី ដែល 165.

ឬផទ?។

ដែល

ិ មា ផតើមានចំនន ួ គត់ វជ្ ជ ន

167.

(អាមេរច ិ ១៩៧៩)

។ អាចពិតចំផ ោះបណ្ត ា ចំនន ួ គត់

ដត

បុ ផណ្ត ណ ោះ។

ចូរបង្ហាញថា គ្មានចំនន ួ គត់ ដែល

166.

។ ចូរបង្ហាញថា

ផទ។

ដែល

ិ មា ចូរកំណត់ ចំនន ួ គត់ មិនអវជ្ ជ ន

ឬផទ? ដែលផផទ ៀងផ្ទទត់ សមីការខាងផរកាម

168.

ផ ោះរាយសមីការកនង ុ សំ ណុំចំនន ួ គត់

169.

ផ ោះរាយសមីការកនង ុ សំ ណុំចំនន ួ គត់

170.

ិ មា ចូរកំណត់ ចំនន ួ គត់ វជ្ ជ ន

171.

(ហុ ងគ្រី ១៩៩៧)

ដែល

ចូរកំណត់ ចផមល ើយជាចំនន ួ គត់ ននសមីការ

លឹម សុ វណ្ណវិចត្ិ រ| សមីការដ្យូផង់

83


172.

ចូរកំណត់ ចំនន ួ គត់

ដែល

173.

ិ មា ក) ចូរកំណត់ រគប់ ចំនន ួ គត់ វជ្ ជ ន ិ មា ខ) ចូរកំណត់ រគប់ ចំនន ួ គត់ វជ្ ជ ន ិ មា គ) ចូរកំណត់ រគប់ ចំនន ួ គត់ វជ្ ជ ន

174. 175.

ចូរកំណត់ ចំនន ួ គត់

ដែល

និង

ដែល

និង

ដែល

ិ មា ចូរបង្ហាញថា ផគមានរតីធាតុសនិទានមិនគត់ វជ្ ជ ន ផសាើ

ដែល

ជាចំនន ួ គត់ ធប ំ ំ ផុតដែលតូចជាងឬ

(ហុ ងគ្រី ១៩៩៨)

177.

ិ មា ចូរកំណត់ រគប់ ចំនន ួ គត់ វជ្ ជ ន

178.

(លថ ួ ី ១៩៩៤) ី ន

ដែល ដែល

ិ មា ចូរកំណត់ រគប់ ចំនន ួ គត់ វជ្ ជ ន (អុ តា ី លី ១៩៩៤)

ចូរកំណត់ រគប់ ចំនន ួ គត់

ដែល

តាង

181.

(មអៀរឡង់ ១៩៩៥) ចូរកំណត់ ចំនន ួ គត់

និង ។

និង

ដែល

ិ មា ជាចំនន ួ គត់ វជ្ ជ ន ដែល

180.

។ ចូរបង្ហាញថា

ើ បីឱ្យសមីការ ផែម

មានចផមល ើយផរចើនរាប់ មិនអស់ ផៅកនង ុ

?។

182.

ចូរកំណត់ រគប់ ចំនន ួ គត់ ធមា ជាតិ

ដែល

183.

ក) ចូរកំណត់ ចំនន ួ គត់ ធមា ជាតិ

ដែល

84

ផរចើនរាប់ មិនអស់ ដែលផផទ ៀង

និង

ិ មា ចូរកំណត់ រគប់ ចំនន ួ គត់ វជ្ ជ ន

179.

ដែល

ផ្ទទត់

176.

និង

សមីការដ្យូផង់ | លឹម សុ វណ្ណវិចត្ិ រ


ើ បីឱ្យផគអាចរកបាន ផែម

ខ) ចូរកំណត់ រគប់ ចំនន ួ គត់ ធមា ជាតិ

ចំនន ួ គត់ ធមា ជាតិ

ដែល

184.

ិ មា ចូរកំណត់ រគប់ ចំនន ួ គត់ វជ្ ជ នឬសូនយ

185.

(អនតរជាតិ សតលីស ២០០៣) ចូរកំណត់ ចំនន ួ គត់

186.

ដែល

តូចបំ ផុត ដែល ផគមានចំនន ួ គត់

ផផទ ៀងផ្ទទត់

(តតវ៉ាន់ ១៩៩៨) ផតើសមីការខាងផរកាម ដែល

187.

ជាចំនន ួ គត់ ធជា ំ ង

(ឥណ្ឌ ា ១៩៩៨) ិ មា ចូរកំណត់ រគប់ ចំនន ួ គត់ វជ្ ជ ន

188.

ដែល

(អនតរជាតិ ១៩៩៧) ចូរកំណត់ រគប់ គូ

189.

មានឫសឬផទ?។

ននចំនន ួ គត់ ដែល

និង

ផផទ ៀងផ្ទទត់ សមីការ

ផយើ ងពិនត ិ យសមីការ ក) ចំផ ោះចំនន ួ គត់ ណ្តខល ោះនន

ដែលសមីការ (*) មានចផមល ើយ

គត់ ផផទ ៀងផ្ទទត់

។ ខ) ផ ោះរាយសមីការ 190.

ចំផ ោះ

ិ មា ចូរកំណត់ រគប់ ចំនន ួ គត់ វជ្ ជ ន

។ ដែល

ដែល លឹម សុ វណ្ណវិចត្ិ រ| សមីការដ្យូផង់

។ 85


191.

គ្ទឹសីប ត ទគ្តីធាតុ ពីតាករ តាង

ជាចំនន ួ គត់ ធមា ជាតិ ដែលមានតួដចករួមធំបំផុតផសាើ

ចូរបង្ហាញថា

ឬក៏មិនអុ ីចង ឹ ផទ

មានចំនន ួ គត់

192.

និង

ជាចំនន ួ គូ ។ កនង ុ ករណី ដែល

ផផទ ៀងផ្ទទត់ ជាចំនន ួ គូ ចូរបង្ហាញថា ផគ

ដែលបឋមនឹងគ្មន ផ ើ យមានភាពគូ ផសសផទុ យគ្មន ដែល

ក) ចូរកំណត់ រតីផកាណដកងដែលមានរង្ហវស់ រជ្ុងជាចំនន ួ គត់ ផ ើ យមានរកឡានផទ ជាចំនន ួ កាផរ។ ិ មា ខ) ចូរកំណត់ ចំនន ួ គត់ វជ្ ជ ន

ដែល

193.

ិ មា ចូរបង្ហាញថា គ្មានរតីធាតុននចំនន ួ គត់ វជ្ ជ ន

194.

គណនាឫសសនិទាន

ដែល

195.

ចូរកំណត់ ចំនន ួ សនិទាន

និង

196.

តាង

។ ផតើសមីការ

ជាដផន កគត់ នន

។ ដែល

ផទ។

។ ដែលផផទ ៀងផ្ទទត់

មានឫសឬផទ?។ 197.

ផ ោះរាយសមីការ

198.

ផ ោះរាយសមីការ

199.

(អូស្ត្រ ត លី ១៩៩៩)

ចូរផ ោះរាយរបព័នស ធ មីការ

200.

ចូរបង្ហាញថា

201.

តាង ចូរបង្ហាញថា

86

មិនអាចផសាើ ជាចំនន ួ គត់ ដែល មានចផមល ើយ។

សមីការដ្យូផង់ | លឹម សុ វណ្ណវិចត្ិ រ

ផទ។


202.

(អនតរជាតិ ១៩៨៤) តាង

និង

ិ មា ជាចំនន ួ គត់ វជ្ ជ នផសស ដែល ដែល

203.

(មអៀរឡង់ ១៩៩៦) តាង

និង

ជាចំនន ួ គត់ ពរី ។ ចូរបង្ហាញថា

ិ មា ជាចំនន ួ គត់ វជ្ ជ ន។ ចូរបង្ហាញថា ផបើ

ជាចំនន ួ បឋម និង

ផនាោះ

។ 204.

(អនតរជាតិ ១៩៨១) គណនាតនមល ធប ំ ំ ផុតរបស់ និង

205.

ិ មា ចូរកំណត់ តនមល វជ្ ជ នតូចបំ ផុត នន

206.

ចូរកំណត់ រគប់ ចំនន ួ បឋម

207.

តាង

និង

208.

និង

ដែល

ជាសវុីតននចំនន ួ គត់ ពរី ដែលផផទ ៀងផ្ទទត់ សវុីត មួ យដែលចំផ ោះរគប់

និង

ជា

ផយើ ងមាន

តាង

ជាព ុ ធាែផឺ រកទី

ផ ើ យមានផមគុ ណជាចំនន ួ គត់ ។ ផបើ

ចូរបង្ហាញថា 209.

ិ មា ចំផ ោះរគប់ ចំនន ួ គត់ វជ្ ជ ន

និង ចំនន ួ គត់

សវុីតនពវ ន។ា ចូរបង្ហាញថា មានចំនន ួ ផថរ ឬ

ិ មា ជាចំនន ួ គត់ វជ្ ជ ន ដែល

ដែលកនង ុ ផនោះ

គ្មានឫសជាចំនន ួ សនិទានផទ។

ចូរបង្ហាញថា ផបើព ុ ធា

ដែលមានផមគុ ណ

ជាចំនន ួ គត់ និង មានតនមល ផសាើ ុ ធាផនោះ មានតំនលផសាើ

សុទធដតជាចំនន ួ ផសស

រតង់

ចំនន ួ គត់ ផផេងគ្មននន

ផនាោះ គ្មានតនមល

ណ្ត ដែលព

ផទ។

លឹម សុ វណ្ណវិចត្ិ រ| សមីការដ្យូផង់

87


88

សមីការដ្យូផង់ | លឹម សុ វណ្ណវិចត្ិ រ


ឡូស៊ិ ច

ជាស្វ័ យគុណបោល២ ប ោះស្ំ បណើព ិត បោយមិនចំបាច់ រាយបញ្ជជក់ ។ បបើ

បបើ

មិនមមនជាស្វ័

យគុណបោល២ ប ោះវារតូវបៅចប ល ោះ ស្វ័ យគុណបោល២ ព ីរតបរៀងោន មានន័យថា ឱ្ ំ យ

ឱ្ ំ យ

(បស្មើ នង ឹ

មានន័យថាមានចំ នន ួ ស្វ័ យគុណបោល២ បៅចប ល ោះ

)។

បយើ ងនឹងរាយបញ្ជជក់ តាមវ ិចរបោយកំបណើន។ បោយារ

ជាស្ំ ណំ ុ មិនទបទននចំ នន ួ គត់ វ ិជជមាន ប ោះ

តាមស្មមតក ិ មម

ជាធាតុរបស្់

។ មានន័យថា បោយ

មត

ជាធាតុមួយរបស្់

ជាធាតុមួយរបស្់

ប ោះ

មានធាតុមដលតូចបំ ផត ុ តាងបោយ មត

ធាតុរបស្់

ជាធាតុតច ូ ជាងបគ ដូបចនោះ ទាល់ មត

ក៏ជាធាតុរបស្់

មដរ ប ើ យដូចោនមដរ

ដូបចនោះ រគប់ ចំ នន ួ ទាំងអស្់ មដលមានរាង ក៏ជាធាតុរបស្់

ជាធាតុរបស្់

។ល។ ។ ប ទ ប់ មកបទៀត

មដរ មានន័យថា រគប់ ចំ នន ួ ស្វ័ យគុណបោល២ទាំងអស្់ ស្ុ ទមធ តជា

ឥលូ វបយើ ងស្នមតថា មាន បោល២ បទ។ បោយ ធាតុរបស្់

បទ បររោះ

(បររោះ បបើ

ប ោះ

ដូចោនមដរ ចំ បរោះ

មួយមដលមិនមមនជាធាតុរបស្់

។ ដូបចនោះ

មិនមមនជាស្វ័ យគុណ

ប ោះ ោមនចំ នន ួ គត់ ណាមួយ បៅចប ល ោះ បគមាន ក៏ជាធាតុរបស្់

មដល

មដលជា ដូបចនោះ

មដរ) ។

ោមនចំ នន ួ គត់

ណាមួយជាធាតុរបស្់

បទ បររោះ បបើ ។

បោយ

ប ោះ

មិនមមនជាធាតុរបស្់

មិនមមនជាធាតុរបស្់

មដរ។

លឹម សុ វណ្ណវិចត្ិ រ| ឡូសិុ ច

89


តាមវ ិចរបោយកំបណើន បយើ ងទាញបានថា ោមនធាតុណាមួយ មដលឋិ តបៅចប ល ោះ ជាធាតុរបស្់

បទ។

ប ទ ប់ មកបទៀត បយើ ងនឹងបង្ហាញថា ចប ល ោះបនោះធំ ណាស្់ មដលរតូវមតមានចំ នន ួ ស្វ័ យគុណបោល២ បៅ កនង ុ ប ោះ។ បបើដបូ ចនោះមមន ប ោះវាផទយ ុ ព ីការស្នមតមដលថា ោមនធាតុណាមួយរបស្់

មដលជាធាតុរបស្់

ប ោះបទ។ អនុគមន៍

ជាអនុគមន៍បកើន ចំ បរោះ

បោយអនុគមន៍

ជាអនុគមន៍ចុោះបលើ

។ ដូបចនោះ

ប ោះ បគមានចំ នន ួ គត់ វ ិជជមានធំ រគប់ រោន់

មួយ

មដល ។ មតរគប់ ចំ នន ួ គត់ វ ិជជមាន

ចប ល ោះ

ស្ុ ទមធ តមានចំ នន ួ

ស្វ័ យគុណបោល២ ជានិច។ ច ដូបចនោះ រតូវមតមានចំ នន ួ ស្វ័ យគុណបោលព ីរមដលជាធាតុរបស្់ ។ បោយចំ នន ួ ស្វ័ យគុណបោលព ីរជាធាតុរបស្់

ប ោះ បយើ ងទាញបានភាព

ផទយ ុ នឹងការឧបមាមដលបយើ ងចង់ បាន។ ដូបចនោះស្ំ បណើព ិត៕ ជាស្ំ ណំ ុ របាំ មួយ

. មចក

និង

សំណំ ទំងប្រំមួយន េះមា លក្ខណៈពិ នសសប្រង់ ន ច ើ ំ ួ គរ់ ពីរឋិ រក្នុងសំណំ តរមួយន ន្ទៀងផ្ទទរ់

។ ឧទហរណ៍ នយើ ងមា

។ សំណំ ខ្លេះដូចជា មា

នយើ ងទញរ

នយើ ងមា ។ ឧទហរណ៍

គ្មា លក្ខណៈពិ នសសត

េះវា

។ ឧទហរណ៍ នយើ ងមា ន េះនេ នប្រេះន នើ យើ ងយក្ចំ ួ គរ់ ពីរ

បយើ ងរតូវបរជើស្បរ ើស្យកចំ នន ួ គត់ របាំ ព ីរបរស្ចមតចិតត បចញព ីស្ំ ណំ ុ ទាំងរបាំ មួយខាងបលើ។ តាមបោល ការណ៍រទុងររប រតូវមតមានចំ នន ួ គត់ ព ីរកនង ុ ចំ បណាមចំ នន ួ គត់ ទាង ំ របាំ ព ីរប ោះ មដលឋិ តបៅកនង ុ ស្ំ ណំ ុ មតមួយកនង ុ ចំ បណាមស្ំ ណំ ុ ទាំងរបាំ មួយខាងបលើ។ ដូបចនោះ ចំ នន ួ គត់ ព ីរបនោះបផទៀងផ្ទទត់ 90

ឡូសិុ ច |លឹម សុ វណ្ណវិចត្ិ រ


ឧទហរណ៍ នយើ ងនប្រើសនរើសនប្សចតរចិរត ឧទហរណ៍ សំណំ មួយនយើ ងយក្ចំ ួ គរ់ មយ ួ ចំ ួ ប្រំមួយដំ ូងមា

នហើ យចំ ួ មួយនេៀរនដើមបី ង្គងរ ់ ប្រំពីរឧទហរណ៍ យក្នលខ្

។ នយើ ងទញរ

ិង

ឋិ រក្នុងសំណំ តរមួយ នហើ យវាន្ទៀងផ្ទទរ់

.  ព ីស្ំ ណំ ុ មួយមដលមាន១០ធាតុ បគអាចបបងកត ើ បាននូវស្ំ ណំ ុ រងមិនទបទខុស្ៗោនចំ នន ួ

(តាមរូបមនត

) ស្ំ ណំ ុ រងមដលមានផលបូកធាតុមានតនមលធំបំផត ុ គឺ បយើ ងមានស្ំ ណំ ុ រងទាំងអស្់ ចំ នន ួ ចប់ ព ី ដល់

ប ើ យមានផលបូកបស្មើ

។ ផលបូកធាតុរបស្់ ស្ំ ណំ ុ រងនិមួយៗទាំងប ោះរតូវមានតនមល

។ ដូបចនោះវារតូវមតមានស្ំ ណំ ុ រងព ីរមដលមានផលបូកបស្មើ ោន។

. បយើ ងស្បងកតបឃើ ញថា បបើបយើ ងបរជើស្បរ ើស្

ចំ នន ួ គត់ បចញព ី

ចំ នន ួ គត់ បនតប ទ ប់ ោន

ប ោះបគរតូវមតទទួលបានចំ នន ួ ព ីរ មដលមានផលស្ង បស្មើ

។ បររោះ បយើ ងអាចចប់ ផគូ

បនតប ទ ប់ ោន

ស្ំ ណំ ុ

បៅជា

ដូបចនោះ បបើបយើ ងរតូវបរជើស្បរ ើស្ យក

ចំ នន ួ គត់ បចញព ី

មដលឋិ តបៅកនង ុ ស្ំ ណំ ុ មតមួយ ប ើ យមានផលស្ងបស្មើ ឥលូ វ បយើ ងបបងកត ើ ស្ំ ណំ ុ បយើ ងរតូវបរជើស្បរ ើស្យក

មដលមាន

ស្ំ ណំ ុ ប ោះ វារតូវមានចំ នន ួ គត់ ព ីរ

ធាតុ ដូចខាងបរកាម

ចំ នន ួ គត់ បចញព ី ស្ំ ណំ ុ

ឋិ តកនង ុ ស្ំ ណំ ុ មតមួយ។ ស្ំ ណំ ុ និមួយៗមានធាតុចំនន ួ ចំ នន ួ គត់ ព ីរមដលមានផលស្ងបស្មើ

ចំ នន ួ គត់

ខាងបលើ ដូបចនោះរតូវមានចំ នន ួ គត់ យ៉ាងតិច ជាចំ នន ួ គត់ បនតប ទ ប់ ោន។ ដូបចនោះរតូវមតមាន

លឹម សុ វណ្ណវិចត្ិ រ| ឡូសិុ ច

91


. ឧបមាថា បយើ ងទាញថាស្បចញមួយចំ នន ួ ដំ បូងមានបលខខុស្ោនទាំងអស្់ បោយមិនមានថាស្ណា មានបលខដូចោនដល់ បៅ

ដងបទ។ ដូបចនោះ ស្នមតថាបយើ ងទាញបចញថាស្មានបលខ១ដល់ បលខ៩ ទាំង

អស្់ ដូបចនោះ មានទាំងអស្់

ថាស្។ ដូបចនោះ ថាស្បៅស្ល់ មានមត បលខ១០ដល់

បលខ៥០។ ប ទ ប់ មកបទៀត ស្នមតថា បយើ ងទាញយកថាស្ព ីកនង ុ ចំ បណាម ថាស្បលខ ១០ដល់ បលខ៥០ មួយមុខ ចំ នន ួ

(បដើមបីកំឱ្ ុ យទាន់ មានថាស្ចំ នន ួ

មានាលកដូចោន)។ ដូបចនោះ បយើ ងយកថាស្បចញបាន ចំ នន ួ

ប ើ យ។ ថាស្មដលទាញបចញបលើកទី

នឹងបធវឱ្ ើ យមានថាស្ចំ នន ួ

មានបលខដូចោន។

. ស្នមតថាស្ុ ខជាមនុស្សមានក់ បៅកនង ុ មនុស្សទាំង១៧ ក់ ប ោះ។ ោត់ ស្របស្រស្ំ បុរតបៅមនុស្ស១៦ ក់ បទៀត។ តាមបោលការណ៍រទុងររប មានរបធានបទមួយ កនង ុ ចំ បណាមរបធានបទទាំង៣ មដល ស្ុ ខស្របស្របៅកាន់ មនុស្សយ៉ាងតិច៦ ក់ បផសងោនមដរ ។ បៅរបធានបទប ោះថា របធានបទបលខ១។ បបើស្ិនជាមាន២ ក់ កង នុ ចំ បណាម៦ ក់ បនោះ ស្របស្របៅកាន់ ោនបៅវ ិញបៅមកនិងបៅស្ុ ខ ព ីរបធាន បទបលខ១ មដរ ប ោះ មានន័យថា មានយ៉ាងតិចមនុស្សចំ នន ួ ៣ ក់ មដលស្របស្រស្ំ បុរតបៅកាន់ អនក ដនទព ីរបធានបទមតមួយដូចោន។ ផទយ ុ បៅវ ិញ ស្នមតថា អន កទាំង៦ ក់ បនោះ ស្របស្របៅកាន់ ោនបៅវ ិញបៅមក មតបលើរបធានបទបលខ២ ឬបលខ៣ ប៉ា ុបណាណោះ។ ស្នមតថា បៅបៅកនង ុ ចំ បណាមអន កទាំង៦ ក់ បនោះ។ តាមបោលការណ៍រទុងររប មានរបធានបទមួយ កនង ុ ចំ បណាមរបធានបទ២ ឬ៣ មដលបៅស្របស្របៅកាន់ មនុស្សយ៉ាងតិច៣ ក់ បផសងោនមដរ កនង ុ ចំ បណាមមនុស្សទាំង៥ ក់ បទៀត។ បៅរបធានបទបនោះ ថា របធានបទបលខ២។ បបើ ស្ិ នជាមាន២ ក់ កនង ុ ចំ បណាម៣ ក់ បនោះ ស្របស្របៅកាន់ ោនបៅវ ិញបៅមក និងបៅបៅ ព ីរបធាន បទ បលខ២បនោះ ប ោះ មានន័យថា មានយ៉ាងតិចមនុស្សចំ នន ួ ៣ ក់ មដលស្របស្រស្ំ បុរតបៅកាន់ អនក ដនទព ីរបធានបទមតមួយដូចោន។ ផទយ ុ បៅវ ិញ បបើអនកទាំង៣ ក់ បនោះ ស្របស្របៅកាន់ ោនបៅវ ិញបៅមក មតព ីរបធានបលខ៣ ប ោះក៏ មានន័យថា មានយ៉ាងតិចមនុស្សចំ នន ួ ៣ ក់ មដលស្របស្រស្ំ បុរតបៅកាន់ អនកដនទព ីរបធានបទមត មួយដូចោន។

92

ឡូសិុ ច |លឹម សុ វណ្ណវិចត្ិ រ


. បយើ ងដឹងថា ចំ បរោះ

បយើ ងមាន

ណ។ ដូបចនោះ បយើ ងអាចរកបានចំ នន ួ បឋម

ស្ុ ទមធ តជាចំ នន ួ ព ុ គុ

មួយមដល

ប ើ យបផទៀងផ្ទទត់

ស្ុ ទធ

មតជាចំ នន ួ ព ុ គុណ។ បៅរតូវគណ ចំ នន ួ បឋម បៅរបាប់ បលខ

បនោះស្ិ ន។ បៅស្ុ ខ។ បបើ

ប ោះ

ប ោះ ដូបចនោះបបើស្ុខបឆលយ ើ ថា

ជាចំ នន ួ បឋម។ មតបបើ

មានន័យថា

មិនមមនជាចំ នន ួ បឋម។

ជាចំ នន ួ បឋម ប ោះមានន័យថា

។ មតបបើស្ុខបឆលយ ើ ថា

មិនមមនជាចំ នន ួ បឋមបទ ប ោះបៅដឹងថា

ប ើ យរតូវឱ្យបលខបផសងបទៀតបៅស្ុ ខ។

បពលបនោះបៅឱ្យបលខ

ប ោះ

។ បបើ

ប ោះ ដូបចនោះបបើស្ុខបឆលយ ើ ថា

ជាចំ នន ួ បឋម។ មតបបើ មានន័យថា

ជាចំ នន ួ បឋម ប ោះមានន័យថា

មិនមមនជាចំ នន ួ បឋមបទ ប ោះបៅដឹងថា

មិនមមនជាចំ នន ួ បឋម។ ។ មតបបើស្ុខបឆលយ ើ ថា

ប ើ យរតូវឱ្យបលខបផសងបទៀតបៅស្ុ ខ។

បបើស្ិនជាខុស្រ ូ ត បៅរតូវាកតាមវ ិធីបនោះរ ូ តដល់ ករណី ថាបតើ បៅឱ្យបលខ

។ បបើ ប ោះ

បឆលយ ើ ថា

ប ោះ មានន័យថា

ជាចំ នន ួ បឋម ប ោះមានន័យថា

ចំ នន ួ បឋមបទ ប ោះបៅដឹងថា

ឬអត់ ? បពលបនោះ

ជាចំ នន ួ បឋម។ មតបបើ មិនមមនជាចំ នន ួ បឋម។ ដូបចនោះបបើស្ុខ ។ មតបបើស្ុខបឆលយ ើ ថា

មានន័យថា

ដូបចនោះបយើ ងស្ននោ ិ ា នបានថា បៅរតូវស្ួ រយ៉ាងបរចើន

បស្មើ

មិនមមនជា

។ ស្ំ នួរ។

. ស្ំ នួរខាងបលើស្មមូលនឹងស្ំ បណើ “បបើឆ្នណា ំ មួយមាននថៃស្ុរកទី១៣ ប ោះរតូវមាននថៃអាទិតយទី១” ។ តារាងខាងបរកាមគណ នថៃទ១ ី ននមខនីមួយៗ បតើរតូវនឹងនថៃអវី។ កនង ុ តារាងបនោះ - ជួរបដកទី 1ជួរឈរទី 2 : បលខ 1 ជានថៃទម ី ួយ ននឆ្នថ ំ ។ ម ី ជួរឈរទី 3 ស្ំ ណល់ នន

មចកនឹង 7

- ជួរបដកទី 2 ជួរឈរទី 2 : មខមករាមាន 31 នថៃ ដូបចនោះនថៃទ ី 1 ននកុមភៈរតូវនឹងនថៃទ ី 32 គិតព ីបដើមឆ្នំ មក។ ជួរឈរទី 3 ស្ំ ណល់ នន

មចកនឹង 7

លឹម សុ វណ្ណវិចត្ិ រ| ឡូសិុ ច

93


- ជួរបដកទី 3 ជួរឈរទី 2 : មខមករាមាន 31 នថៃ មខកុមភៈអាច 28 ឬ 29 នថៃ ដូបចនោះនថៃទ ី 1 ននមី រតូវនឹងនថៃទ ី 60 ឬ 61 គិតព ីបដើមឆ្នម ំ ក។ ជួរឈរទី 3 ស្ំ ណល់ នន

មចកនឹង 7

…… តាមរយៈតនមល កង នុ ជួរឈរទី

បយើ ងបឃើ ញថា បោយមចកលំ ោប់ នថៃននបដើមមខនីមួយៗ គិតព ីបដើមឆ្នំ

មក បយើ ងទទួលបានស្ំ ណល់ មានតាំងព ីបលខ ធាលក់ ចំនថៃណាមួយ រគប់ នថៃទាង ំ មដលមាននថៃស្ុរកទី

ដល់ បលខ

។ មានន័យថា នថៃទម ី ួយននមខនីមួយៗ

។ ដូបចនោះវារតូវមតមាននថៃអាទិតយទី

កនង ុ មខណាមួយ។ មខប ោះជាមខ

។ 1

2

3

មខ

លំ ោប់ នថៃបដើមមខ នីមួយៗគិតព ីបដើមឆ្នម ំ ក

មករា កុមភៈ

(= ១ កុមភៈ)

មី

បមា

94

(=១មី )

ឬ ឬ

ឧស្ភា

មិថ ុ

កកកោ

ស្ី ហា

កញ្ជា

តុលា

វ ិចឆកា ិ

ធនូ

ឡូសិុ ច |លឹម សុ វណ្ណវិចត្ិ រ


. បយើ ងតំបរៀប

ចំ នន ួ គត់ ជា

ស្ំ ណំ ុ ដូចខាងបរកាម

ស្ំ ណំ ុ និមួយៗមានធាតុរបស្់ វាបឋមនឹងោន។ បោយបគរតូវបរជើស្បរ ើស្

ចំ នន ួ គត់ ប ោះបគរតូវមត

បានចំ នន ួ គត់ ព ីរមដលបៅកនង ុ ស្ំ ណំ ុ មតមួយ។ មានន័យថា រតូវមតទទួលបានចំ នន ួ ព ីរមដលបឋមរវាង ោន។ . បគរតូវការបរបើបលខចំ នន ួ

។ ទូព ីបលខ១ ដល់ បលខ៩ រតូវបរបើបលខ

ចំ នន ួ ៩បលខ។ ព ីបលខ១០ដល់ បលខ៩៩ រតូវបរបើបលខ បលខ៩៩៩ រតូវបរបើបលខចំ នន ួ

បលខ ប ើ យទូរព ីបលខ១០០ដល់

។ ទូបៅស្ល់ រតូវការតួបលខចំ នន ួ

។ ដូបចនោះរតូវមានទូចំនន ួ

បទៀត មដលទូនម ិ ួយៗបរបើបលខ៤ខទង់។

ដូបចនោះជាស្រុបមានទូចំនន ួ

. ក្នុងចំ បណាមចំ នន ួ គត់ មដលធំ មិនបលើស្ព ី

, ចំ នន ួ មដលជាព ុ គុណនន

; ចំ នន ួ មដលជាព ុ គុណនន មានចំ នន ួ គុណនន

មានចំ នន ួ

ចំ នន ួ

មានចំ នន ួ

; ចំ នន ួ មដលជាព ុ

។ ដូបចនោះចំ នន ួ មដលជាព ុ គុណនន (ដក

មាន

បចញ បររោះរាប់ រចំ មដលប៉ា ុណនឹងដង)។

ប ទ ប់ មកបយើ ងរតូវដកចំ នន ួ មដលជាព ុ គុណនន៥ មដលព ុ គុណនឹង៣ ឬនឹង៤ បចញ។ ចំ នន ួ ព ុ គុណនឹង៥ផងនឹង៣ផងមានចំ នន ួ

។ ចំ នន ួ ព ុ គុណនឹង៥ផងនឹង៤ផងមានចំ នន ួ

។ ចំ នន ួ មដលជាព ុ គុណនន៣ ៤និង៥ មានចំ នន ួ

។ ដូបចនោះ

ចំ នន ួ មដលជាព ុ គុណនន៥ មដលព ុ គុណនឹង៣ ឬនឹង៤ មានចំ នន ួ

ដូបចនោះចំ នន ួ មដលជាព ុ គុណនន៣ឬនន៤ មតមិនមមនជាព ុ គុណនន៥មានចំ នន ួ ។ . តាង ជាបលខខទង់ទី១៩៨៣។ បយើ ងកាត់ តបួ លខបរកាយបកបៀស្របស្់

រតឹមខទង់ទី១៩៨៣ជា

លឹម សុ វណ្ណវិចត្ិ រ| ឡូសិុ ច

95


មានបលខ៩ខទង់។

មានបលខ

ខទង់។ ដូបចនោះ រតូវមានបលខ

ខទង់។ បយើ ងមាន

។ ដូបចនោះ

មានបលខ៣ខទង់ដំ បូងចំ នន ួ

ខទង់ទីមួយបស្មើ ១០០ ទីព ីរបស្មើ ១០១....ទី៥៩៨បស្មើ

។ បលខ៣

។ ដូបចនោះខទង់ទី១៩៨៣បស្មើ

. ស្នមតផទយ ុ បៅវ ិញថា មានរបបៀបអងគយ ុ មដលោមនបកមងរស្ី បៅអមស្ង្ហាងបកមងរបុស្។ បយើ ងកំណត់ បលុកមួយបោយ បកមងរស្ី មានក់ ឬព ីរ ក់ អងគយ ុ ជាប់ ោន ប ើ យមានបកមងរបុស្អងគយ ុ ស្ង្ហាង ...របុស្.រស្ី .(រស្ី ).របុស្.... បដើមបីកំឱ្ ុ យមានបកមងមានក់ មដលអន កបៅស្ង្ហាងវា ជាបកមងរស្ី ប ោះទាល់ មតបលុកនីមួយៗមានបកមងរស្ី បៅក ណា ត លយ៉ាងបរចើនព ីរ ក់ ប ើ យរតូវមានបកមងរបុស្អងគយ ុ អមយ៉ាងតិចព ីរ ក់ ។ ដូបចនោះបលុកមបបបនោះ មានយ៉ាងបហាចណាស្់ ចំ នន ួ

ប ើ យរតូវការបកមងរបុស្យ៉ាងបហាចណាស្់ ចំ នន ួ

ក់ មដរបដើមបីអងគយ ុ បៅរវាងបលុកនិមួយៗ។ ប៉ា ុមនតបយើ ងមានបកមងរបុស្មត២៥ ក់ មត ប៉ា ុបណាណោះ។ ដូបចនោះការស្នមតរបស្់ បយើ ងមិនអាចបៅរួចបទ។ . តាងបលខបជើងនីមួយៗរបស្់ ព ីងរងបោយបលខ១ដល់ បលខ៨។ តាង និងមស្បកបជើង មដលស្ថិ តបៅបជើងទី នន

មដល

ចំ នន ួ

ប ើយ

មដល

បៅមុខ

ចំ លាស្់ មដលមាន

បៅមុខ

ជាបរាមបជើង

។ តំបរៀបមដលអាចននបរាមបជើងនិងមស្បកបជើង ជាចំ លាស្់

រតូវបៅមុខ

ចំ បរោះ

ចំ លាស្់ មានចំ នន ួ រក់ កណា ត លគឺ

មានចំ នន ួ រក់ កណា ត លននចំ នន ួ បរកាយបនោះគឺ បៅមុខ

និង

មដល

មានចំ នន ួ

។មានចំ លាស្់ ននចំ នន ួ ទាំង១៦បនោះ ។ ដូចោនមដរ ចំ នន ួ ចំ លាស្់ ។ ដូបចនោះបយើ ងទាញបានចំ នន ួ ។

. ស្នមតថាយុ វជនប ោះយកបរាមបជើង៤ដំ បូងមានពណ៌ខុស្ោនទាំងអស្់ ។ ដូបចនោះវាចប់ មិនទាន់ បានមួយគូបទ។ វាយកបរាមបជើងមួយបទៀត។ បទាោះជាបរាមបជើងបនោះពណ៌អី កប៏ ោយ ក៏គង់ ទទួល បានបរាមបជើងមានពណ៌ដូចោនមួយគូមដរ។ ស្នមតថាពណ៌បខៀវ។ ដូបចនោះយុ វជនបនោះចប់ បានបរាម បជើងពណ៌បខៀវ២ រក

96

ម១ នបតង១ និងបមម ១។

ឡូសិុ ច |លឹម សុ វណ្ណវិចត្ិ រ


វាចប់ យកមថមមួយបទៀត។ ករណីអន់ បំផត ុ គឺវាយកបានបរាមបជើងមួយបទៀត មដលមានពណ៌បខៀវ បររោះមិនអាចផសំបានបរាមបជើងមួយគូមថមបទៀតបទ។ ដូបចនោះបរាមបជើងមដលយកបចញប ើ យមាន បខៀវ៣ រក

ម១ នបតង១ និងបមម ១។

វាយកបរាមបជើងមួយបមនថមបទៀត។ បពលបនោះបទាោះជាយកពណ៌អី កគ ៏ ង់ ផសំបានបរាមបជើងមួយគូ បទៀតមដរ។ ស្នមតថាវាចប់ ចំ ពណ៌បខៀវដមដល។ ដូបចនោះវាទាញបរាមបជើងបចញមកបរៅបាន ពណ៌ បខៀវ៤ រក

ម១ នបតង១ និងបមម ១។

តាមរបបៀបដូចោន វាយកបរាមបជើងបចញបាន ពណ៌បខៀវចំ នន ួ ២០ រក

ម១ នបតង១ និងបមម ១។

មានន័យថាវារតូវយកបរាមបជើងបចញចំ នន ួ ២៣។ . របភាគមួយមដលបគបរងួមរួច មានភាគមបងនិងភាគយកបឋមនឹងោន។ ដូបចនោះកតាតបឋមមួយ មិនអាចស្ថិ តបៅកនង ុ ភាគយកផងភាគមបងផងបានបទ។ កតាតបឋមរបស្់ និង

មានចំ នន ួ ៨ គឺ

។ កតាតបឋមនិមួយៗកនង ុ ចំ បណាមបនោះ រតូវឋិ តបៅបលើភាគយកឬបៅ

បលើភាគមបងបានមតមតង មតមិនអាចឋិ តបៅបលើភាគយកផង ភាគមបងផងបានបទ។ ដូបចនោះបគមាន របបៀប។ មតចំ នន ួ ទាំង

មដលទទួលបានប ោះរតូវបធវឱ្ ើ យរបភាគមានតនមលតច ូ ជាង១។

កនង ុ ចំ ប មរបភាគទាំង២៥៦ប ោះ បយើ ងអាចមចកជារបភាគចំ នន ួ ១២៨គូ មដលគូនម ី ួយៗគុណោនបស្មើ ១ ឬកនង ុ គូនម ី ួយៗមានរបភាគមួយមានតនមល តច ូ ជាង១។ ដូបចនោះចំ នន ួ ស្និទានមដលមានលកាណៈចង់ បាន មានចំ នន ួ ១២៨។ . ចំ នន ួ គត់ បស្ស្

ដូបចនោះ

នីមួយៗអាចជំនស្ ួ បោយ

។ ចតុធាតុ

មដល

មានដូចជា

ជាចំ នន ួ គត់ វ ិជជមាន។ បោយ

ដូចោននឹងបលខ១ចំ នន ួ

១៧ ខណឌបោយបលខ០ ប ទ ប់ មកបលខ១ចំ នន ួ ៥ ខណឌបោយបលខ០ ប ទ ប់ មកបលខ១ចំ នន ួ ១១ ខណឌ បោយបលខ០ ប ទ ប់ មកបលខ១ចំ នន ួ ១៨

លឹម សុ វណ្ណវិចត្ិ រ| ឡូសិុ ច

97


ដូបចនោះចតុធាតុបនោះដូចោននឹងបលខ១ចំ នន ួ ៥១ រតូវមចកជាបួនរកុមខណឌបោយបលខ០។ បគមាន របបៀបកនង ុ ការស្កបលខ០ចូលកនង ុ ចប ល ោះទាំង៥០រវាងបលខ១ទាំងអស្់ ៥១។ . ស្នមតថា

មានទរមង់ ទស្ភាគជា

ប ោះនឹងមានរតាទុក បពល

បូកនឹង

ប ោះក៏នង ឹ មានរតាទុកមដរ បពល បបើ

។ បបើមួយកនង ុ ចំ បណាម ។ បបើ

បូកនឹង

និង

ជាបលខ

ឬ បបើ

និង

ឬ ឬ

ជាចំ នន ួ គត់ មដលមានលកាណៈខាងបលើ ប ោះវារតូវមានរាងមួយខាងបរកាម

មដល តនមល

ទាំងបនោះមិនរតូវការរតាទុកបទ បពលបយើ ងបូក

មបបបនោះ មានទាំងអស្់ ចំ នន ួ

ទាំងអស្់

នឹង

។ ដូបចនោះមានចំ នន ួ គត់ បនតប ទ ប់ ោន

គូ មដលផលបូកគូនម ី ួយៗមិនរតូវការរតាទុក។

. បៅអី ខាងចុងទាំងព ីររតូវមតស្រមាប់ ស្ិ ស្សអន កអងគយ ុ ដូបចនោះាស្ត្ាតចរយមានបៅអី កង នុ ចំ នន ួ ៧ ស្រមាប់ បរជើស្បរ ើស្ បោយមិនឱ្យអងគយ ុ បៅជាប់ ោន។ បបើបៅអី ទាង ំ បនោះរតូវតាងបោយបលខ២ដល់ បលខ ៨ ប ោះបៅអី ទាង ំ បីស្រមាប់ ាស្ត្ាតចរយមាន កនង ុ រតីធាតុនម ិ ួយៗ ាស្ត្ាតចរយអាចផ្ទលស្់ បតូរកមនលងោន បាន

របបៀប។ ដូបចនោះជាស្រុបមាន

។ ជាចំ នន ួ គត់ ទាង ំ ១៦ប ោះ។ បយើ ងព ិនិតយគូបផសងៗោនននចំ នន ួ គត់ ទាង ំ

. តាង

បនោះ។ បគអាចផសំបានគូមបបបនោះចំ នន ួ តាង

ជាគូមួយ មដល

គូ។ ។ បបើបយើ ងមានគូព ីរបផសងោន

និង

មដល

ប ោះបយើ ងទាញបានចតុធាតុមដលចង់ បាន បលើកមលងមតករណី បយើ ងថា

មិនបានការ ចំ បរោះគូព ីរ គឺ

និង

)។ បយើ ងស្មាគល់ បឃើ ញថា ស្ំ បនើព ិត បបើ

98

ឡូសិុ ច |លឹម សុ វណ្ណវិចត្ិ រ

បបើ

(ឬ

មិនបានការចំ បរោះគូព ីរននគូចំនន ួ គត់ បររោះ


ថា បបើ មិនបានការចំ បរោះ

និង

ប ោះ

។ ជាចុងបរកាយបយើ ងស្នមតថា

នីមួយៗ មិនបានការ បរចើនបំ ផត ុ បៅកនង ុ គូមួយននគូចំនន ួ គត់ ។ ចំ បរោះគូ

ននគូចំនន ួ គត់ នម ី ួយៗទាំងបនោះ បយើ ងដកគូននចំ នន ួ គត់ មួយបចញ។ ដូបចនោះោមនចំ នន ួ មិនបានការបទៀតបទ។ ដូបចនោះបយើ ងបៅស្ល់ យ៉ាងបហាចណាស្់

គូននចំ នន ួ គត់ មដលបៅស្ល់ ។ ផលស្ងនន

ចំ នន ួ បៅកនង ុ គូបៅស្ល់ នីមួយៗ មានតនមល ព ី១ដល់ ៩៩។ តាមបោលការណ៍រទុងររប មានផលស្ងខលោះ មានតនមលដច ូ ោន។ ស្នមតថា

ដូបចនោះ

បផទៀងផ្ទទត់ លកាខណឌស្ំ

នួរ។ . បដើមបីឱ្យឥដាមួយដុំមានលកាណៈខុស្ព ីឥដាមួយដុំបទៀត បគមានជបរមើស្មួយបៅបលើស្មាភរៈ ជបរមើស្ព ីរបៅបលើទំ ំ ជបរមើស្៣បៅបលើពណ៌ និងជបរមើស្៣បៅបលើរទង់ រទាយ។ មាន របបៀបមដលឥដាមួយដុំខុស្ព ីឥដាមួយដុំបទៀត រតង់ ព ីរចំ ណុចគត់ គឺ ១) ស្មាភរៈនិងទំ ំ ៖ មានឥដា ចំ នន ួ

ដុំបផសងោន

២) ស្មាភរៈនិងពណ៌៖ មានឥដា ចំ នន ួ

ដុំបផសងោន

៣) ស្មាភរៈនិងរទង់ រទាយ៖ មានឥដា ចំ នន ួ ៤) ទំ ំ និងពណ៌៖ មានឥដា ចំ នន ួ

ដុំបផសងោន ដុំបផសងោន

៥) ទំ ំ និងរទង់ រទាយ៖ មានឥដា ចំ នន ួ

ដុំបផសងោន

៦) ពណ៌និងរទង់ រទាយ៖ មានឥដា ចំ នន ួ

ដុំបផសងោន

ដូបចនោះជាស្រុបមាន ឥដាចំនន ួ

ដុំ មដលមានលកាណៈខុស្ព ីដុំមួយដុំមដល

ឱ្យរតង់ ព ីរចំ ណុចគត់ ។ . ចំ នន ួ បលខមដល អាចបបងកត ើ បានបោយបរបើបលខ០ដល់ ៩ ជារាង ។ ដូបចនោះបយើ ងមានករណី ករណី និង ករណី ករណី ប៉ា ុបណាណោះ(ស្ំ ណំ ុ

មានចំ នន ួ

(ស្ំ ណំ ុ )ចំ នន ួ (ស្ំ ណំ ុ )ចំ នន ួ

ករណី។

អាចមតបពល )។ ករណីមបបបនោះមាន

មត

ករណី។ ដូបចនោះ លឹម សុ វណ្ណវិចត្ិ រ| ឡូសិុ ច

99


របបៀបកនង ុ ការបរជើស្បរ ើស្ទីតាង ំ ននរកឡាពណ៌បលឿង។ បោយារបគ

. បគមាន

អាចបងវល ិ កាតរអុ ក៩០ដឺបរក ប ោះកាតរពណ៌មិនស្មមូលោនមានតិចជាង នឹ ង។ កាតរពណ៌មដលមានពណ៌ បលឿងមិនឋិ តបៅឈមោនជាអងកត់ផត ចិ មានទំ រង់ ស្មមូល៤មបប។ កាតរពណ៌មដលរកឡាពណ៌បលឿងព ីរ ឈមោនជាអងកត់ផត ចិ មានទរមង់ ស្មមូលព ីរ ប ើ យបយើ ងមានគូរកឡាពណ៌បលឿងមបបបនោះចំ នន ួ ។ ដូបចនោះចំ នន ួ កាតរអុ កពណ៌មិនស្មមូលោនមានចំ នន ួ

Y1 Y2 x Y2 Y1 រកឡាY1 ឈមោនជាអងកត់ផត ចិ ។ រកឡាY2 ឈមោនជាអងកត់ផត ចិ ។ . បោយបយើ ងចង់ ស្ិ កាស្ំ នួរបនោះ តាមរយៈស្មមូលតាម៣ ដូបចនោះបយើ ងស្របស្របលខ និង

ជា

។ ស្នមតថា

ជាតំ បរៀបមដល

បយើ ងចង់ បាន។ បយើ ងបឃើ ញថា ។ ដូបចនោះ រតូវមតជាតំបរៀបនន

បររោះ

។ បោយ

ប ោះ

តាមរបបៀបដូចោន បយើ ងទាញបានថាលំ ោប់ របស្់ 100

ឡូសិុ ច |លឹម សុ វណ្ណវិចត្ិ រ

កំណត់ បោយ

។ មតមួយគត់ ។


ដូបចនោះបយើ ងមាន

របបៀប (បដើមបីបរជើស្បរ ើស្

បយើ ងមាន របបៀប។ ស្នមតថា

ដំ បូងបរជើស្បរ ើស្យក

ដូបចនោះ បដើមបីបរជើស្បរ ើស្

បយើ ងមាន របបៀប បដើមបីបរជើស្បរ ើស្ របបៀប ដូបចនោះបយើ ងមាន

បយើ ងមាន របបៀប បដើមបីបរជើស្បរ ើស្

របបៀបកនង ុ ការបរជើស្បរ ើស្

បយើ ងមាន

។ ប ទ ប់ មកចំ លាស្់ របស្់ វាមាន

)

។ . បដើមបីឱ្យ

ប ោះចំ បរោះធាតុនម ី ួយៗ

របស្់

បគមានស្ំ បណើមួយគត់ កង នុ ចំ បណាម

ស្ំ បណើទាំងបីខាងបរកាមមដលព ិត ដូបចនោះបបើ

មាន

ធាតុ ប ោះមាន

បយើ ងរាប់ ជាប់ ោនព ីរដង បររោះឧទា

កនង ុ ការបរជើស្បរ ើស្ស្ំ ណំ ុ រណ៍ គូស្ំណំ ុ រង

មដលបស្មើ

បពល

ប ោះ

ទាល់ មត

បស្មើ នង ឹ

. ស្នមតថាផទយ ុ មកវ ិញថា មានចំ នន ួ គត់ បន ួ បផសងោន មដល

។ បរៅព ីករណី និងគូ

រតូវរាប់ មតមតង។ បោយ ដូបចនោះចំ នន ួ គូននស្ំ ណំ ុ រងមដលមានរបជុំរបស្់ វាជា

និង

មដល

។ តាង

ជាចំ នន ួ គត់ ។

ដូចោន បយើ ងទាញបាន

បយើ ងមាន បបើ

ប ោះ

មតបយើ ងរតូវស្ិ កាករណី

ដូបចនោះបយើ ងមិនផ្ទត់ បចលករណីបនោះ។ ដូបចនោះ

។ បយើ ងទាញបាន

លឹម សុ វណ្ណវិចត្ិ រ| ឡូសិុ ច

101


បោយ

បររោះ

ករណី

។ ដូបចនោះ

ស្ុ ទមធ តជាចំ នន ួ គត់ ។

ជាឬស្ននស្មីការ

ស្មីការបនោះមានឬស្

មដល

បយើ ងមាន

ទាល់ មត

និង

មតមិនអាចបររោះ បបើ

ប ោះ

បបើ

ប ោះ

កនង ុ ករណីទាំងព ីរ បយើ ងមាន

បររោះ ។ ដូបចនោះ

ឬ មានតនមលោច់ ខាតធំ ជាង

ដូចោនចំ បរោះករណី

។ ដូបចនោះ

មដលករណីបនោះផទយ ុ ព ីស្មមតក ិ មម។

. ព ិនទមុ ដលបគអាចបធវបា ើ ន ជាចំ នន ួ គត់ មិនអវ ិជជមានមដលមានរាង Bachet-Bézout បបើ ប ោះមានចំ នន ួ គត់

ប ោះ

រតូវមតមចក

បរចើនរាប់ មិនអស្់ មដលមចកមិនោច់ នឹង

។ តាមរទឹស្តីបទ ោច់ ។ បបើ ។ ដូបចនោះមានព ិនទបុ រចើនរាប់

មិនអស្់ មដលបគមិនអាចទទួលបាន ករណីបនោះផទយ ុ ព ីស្មមតក ិ មម មដលមានមតព ិនទច ុ ំ នន ួ មដលបគមិនអាចទទួលបាន។ ដូបចនោះ ទទួលបានមាន ចំ នន ួ

102

ឡូសិុ ច |លឹម សុ វណ្ណវិចត្ិ រ

ប៉ា ុបណាណោះ

។ តាមរទឹស្តីបទរបូបិន ចំ នន ួ ព ិនទមុ ដលមិនអាច ។ ដូបចនោះ


លកាខណឌ

និង

ឱ្ ំ យបយើ ងទាញបានព ីរករណី

និង

។ បោយ ប ើយ

ជាព ិនទមុ ដលបគមិនអាចទទួលបាន ដូបចនោះករណីទី១មិនរតឹមរតូវ។ ប ទ ត់ កាត់ តាមចំ ណុច

និង

។ ប ទ ត់ បនោះមិនកាត់ តាមចំ ណុចគត់

ណាមួយបៅកនង ុ ការដង់ ទី១បទ។ ដូបចនោះមិនមានចំ នន ួ គត់ វ ិជជមាន

ដូបចនោះចបមលើយមានមតមួយគត់ គ ឺ . តាង បូក

មដលបផទៀងផ្ទទត់ ស្មីការបនោះបទ។

ជាចំ នន ួ ដុំឥដាមដលតំបរៀបជាកមពស្់

និង

បរៀងោន។ បយើ ងចង់ ដឹងថាបតើផល

មានប៉ា ុ ម នរបបភទខុស្ោន បោយឱ្យលកាខណឌ ។

បយើ ងមាន

។ បោយយក

បយើ ងរាប់ ចំ នន ួ

ចបមលើយមិនអវ ិជជមានរបស្់ វ ិស្មភាព

តាមរទឹស្តីបទរបូបិន រគប់ ចំ នន ួ គត់ បាន ប ើ យតាមរទឹស្តីបទរបូបិន ចំ នន ួ នន ចំ នន ួ

។ បយើ ងមានករណី

ស្ុ ទមធ តអាចស្របស្រជារាង មដលមិនអាចស្របស្រជារាង និង

បាន មាន

មដលមិនអាចស្របស្រជារាង

លឹម សុ វណ្ណវិចត្ិ រ| ឡូសិុ ច

103


បាន។ ដូបចនោះ កនង ុ ចំ បណាម មាន

ចំ នន ួ គត់ មិនអវ ិជជមាន មដល

ចំ នន ួ គត់ មដលអាចស្របស្រជារាង

យក

បយើ ងបឃើ ញថា

បាន។

បយើ ងទាញបាន ។ តាមរទឹស្តីបទរបូបិន បណា ត

រាង

បាន មានចំ នន ួ

ទាំងប ោះរតូវមត

មដលមិនអាចស្របស្រជា

ករណី ។ ប ើ យតាមរទឹស្តីបទរបូបិន បណា ត មដលកនង ុ ប ោះមាន

។ ដូបចនោះរគប់ ចំ នន ួ គត់

ដូបចនោះរតូវនឹង បលើកមលងមតករណី

បចញ អាចស្របស្រជារាងខាងបលើបាន ដូបចនោះចំ នន ួ ផលបូកខុស្ៗោនមានចំ នន ួ ។ . បយើ ងនឹងបង្ហាញថា បគមិនអាចបំ មបកស្ំ ណំ ុ ខាងបលើជាព ីរដូចបរៀបរាប់ បទ។ ស្នមតផទយ ុ បៅវ ិញ ថាបគអាចបំ មបកវាជាព ីរបាន បោយស្នមតថា ផលគុណននបណា ត ធាតុរបស្់ ស្ំ ណំ ុ មួយបស្មើ ផលគុណននបណា ត ធាតុរបស្់ ស្ំ ណំ ុ មួយបទៀតបស្មើ

។ កនង ុ ចំ បណាមចំ នន ួ គត់ ចំនន ួ

បនតប ទ ប់ ោន បយើ ង

បយើ ងអាចមាន២ករណីគឺ ករណីមានចំ នន ួ មួយគត់ កង នុ ចំ បណាមប ោះមដលមចកោច់ នឹង មួយបទៀត ោមនចំ នន ួ ណាមួយមចកោច់ នឹង

មដលមចកោច់ នឹង

។ កនង ុ ករណីបនោះមានមត A ឬ B មួយប៉ា ុបណាណោះមដលមចកោច់ នឹង ។ ឧទា មចកមិនោច់ នឹង

និងករណី

បទ។

ករណីទី១៖ មានមតមួយគត់ កង នុ ចំ បណាម ប ោះ

ប ើ យនិង

រណ៍ បបើ

មចកោច់ នឹង

បទ ដូបចនោះ A មិនអាចបស្មើ B បទ។

ករណីទី២៖ រគប់ ធាតុទាង ំ អស្់ ស្ុ ទមធ តបឋមនឹង ។ កនង ុ ករណីបនោះ បយើ ងមាន មតបបើ

ប ោះ ស្មមូលខាងបលើបៅជា ដូបចនោះ

. ឧបមាថា

មិនអាច។ ជាចំ នន ួ បល ូ កនង ុ របអប់ បលខ

។ ចំ នន ួ បនសំគប ូ ល ូ កនង ុ របអប់

104

ឡូសិុ ច |លឹម សុ វណ្ណវិចត្ិ រ

។ ចំ នន ួ កាបរស្មមូលនឹង

មានចំ នន ួ

។ បយើ ងមាន ។ ដូបចនោះ ជាស្រុបបស្មើ នង ឹ


មដលបយើ ងចង់ ឱ្យវាតិចបំ ផត ុ ។ បយើ ងរំមលកបល ូ តាមវ ិធីដច ូ តបៅបនោះបដើមបីចំនន ួ ប នសំគប ូ ល ូ ថយចុោះរ ូ តដល់ តិចបំ ផត ុ ។ បបើមាន

មដល

ប ោះ បោយដកបល ូ មួយបចញព ីរបអប់ បលខ ប ើ យយកបៅោក់ កង នុ

របអប់ បលខ បយើ ងទាញបាន ចំ នន ួ គូបល ូ ថយចុោះ

បនតប ទ ប់ មកបទៀត បយើ ងបនថយចំ នន ួ គូបូលតាមរបបៀបមបបបនោះ បោយបធវយ ើ ៉ាងណាឱ្យចំ នន ួ បល ូ កនង ុ របអប់ ព ីរបផសងោន មានចំ នន ួ បល ូ បរចើនជាងោនមិនបលើស្ព ីមួយបទ។ ដូបចនោះ ជាបញ្ច ប់ របអប់ នីមួយមាន បល ូ ចំ នន ួ

រឺ

បូល។ បៅខណៈប ោះ ចំ នន ួ បនសំគប ូ ល ូ បៅតិចបំ ផត ុ ។

លឹម សុ វណ្ណវិចត្ិ រ| ឡូសិុ ច

105


106

ឡូសិុ ច |លឹម សុ វណ្ណវិចត្ិ រ


ប្រព័ ន្រធ បារ់ .ចំ ន្ន្ ួ គត់ .ភាព ចចកដាច់ ត្រព័ន្រធ បារ់ 32. យ ើ ងមាន

តាង

ជាផលបូកយលខខទង់នីមួ ៗរបស់

។ តាមរទឹសតីបទននភាពចចកដាច់ នឹង

យ ើ ងទាញបាន

យ ើ ងមាន មានន័ ថា

មានយ៉ាងយរចើន យ៉ាងយរចើនយសមើ

ជាតិទាង ំ អស់ ចដល ដូយចនេះ

ដូយចនេះ

យ៉ាងយរចើនយសមើ

។ កនង ុ ចំ យោមចំ នន ួ គត់ ធមម

, ចំ នន ួ ចដលមានផលបូកតួយលខននខទង់នីមួ ៗរបស់ វាធំ បំផត ុ គឺ ។ កនង ុ ចំ យោមចំ នន ួ គត់ ធមមជាតិទាង ំ អស់ ចដល

បូកតួយលខននខទង់នីមួ ៗរបស់ វាធំ បំផត ុ គឺ របស់

ខទង់។ ដូយចនេះ ផលបូក តួយលខខទង់នីមួ ៗរបស់

។ ចតយដា

ចដលផលបូកតួយលខយសមើ យ េះមានន័ ថា

, ចំ នន ួ ចដលមានផល ។ ដូយចនេះ ផលបូកតួយលខ ។

33.  យ ើ ងមាន

លឹម សុ វណ្ណវិចត្ិ រ| ត្រព័ន្រធ បារ់

107


ឬតាមរយបៀបមួ ចបបយទៀត យដា

យ េះ តាម

រទឹសតីបទអលឺ ែ យ ើ ងទាញបាន

ដូយចនេះ យ ើ ងទាញបានយលខព ីរខទង់ចុងយគគឺ

ត្រឹសីតររអឺលល បរើ

ប ោះ

លែល

ជាចំន្ន្ ួ នន្ចំន្ន្ ួ គរ់ វជ្ ិ មា ជ ន្លែលធំមិន្បលើសពី

ន្ិងរឋមន្ឹង

ត្រឹសីតររ 6.5 បរើ

ជាផលគុណ្កត្តតរឋមររស់

ប ោះ

។ ដូយចនេះ តាមរទឹសតីបទអឺលែ

34. យ ើ ងមាន ។ យ ើ ងមាន

ដូយចនេះ

។ យ ើ ងមាន មានន័ ថា

។ ដូយចនេះ

ចំ យ េះចំ នន ួ គត់

មានន័ ថា យលខ២ខទង់ចុងយគគឺ

មួ ។ យ ើ ងទាញបាន

35. តាងចំ នន ួ យ េះមុនលុ បយលខខាងចុងយោលយដា មានយលខោប់ ព ីមួ ខទង់យ ង ើ យៅ និង

ជាចំ នន ួ គត់ ។ ដូយចនេះ

យបើ

យ េះ រគប់ ចំ នន ួ គត់

យបើ

យ េះ

យ ើ ងទាញបាន ចំ នន ួ យ េះគឺ

យបើ

យ េះ

108

។ ។

យ ើ ងទាញបានចំ នន ួ យ េះគឺ

ត្រព័ន្រធ បារ់ |លឹម សុ វណ្ណវិចត្ិ រ

ជាចំ នន ួ គត់ ធមមជាតិ យ ើ

។ តាមសមមតក ិ មម យ ើ ងមាន

ចដល m ជាចំ នន ួ គត់ ធមមជាតិ។ ដូយចនេះ ចចកដាច់

ចដល

និង

អាច


តាមរយបៀបដូចយនេះ យ ើ ងទាញបាន ចំ នន ួ ចដលរតូវរកមាន៖ និងយលខ ជាព ុ គុណនន

36. សនមតថា ចំ នន ួ ទាំងយ េះមាន ជាយលខមាន

ខទង់។ ដូយចនេះចំ នន ួ យនេះអាចសរយសរជា

ខទង់ (វាអាចយផតម ើ យដា យលខសូ នយមួ ឬយរចើនជាងយនេះ)។ តាមសមមតក ិ មម យ ើ ងមាន

ចំ យ េះ

យ ើ ងទាញបាន

ចំ យ េះ

យ ើ ងទាញបាន

មិនចមនជាចំ នន ួ គត់ ។ មានន័ ថា

មានរាង

)។ យ ើ ងទាញបាន ចំ នន ួ ចដលរតូវរកមានរាង 37. សនមតថា តាង

ចដល

(មានយលខសូ នយចំ នន ួ

មានរាង

ជាផលគុណតួយលខខទង់នីមួ ៗរបស់

ដូយចនេះ

យ ើ ងមាន

ដូយចនេះ

យដា អងគទាង ំ ព ីរយសមើ គ្ននយពល

មានយលខមួ ខទង់

។ ដូយចនេះ

។ យ ើ ងមាន ចំ យ េះ ដូយចនេះ

38. យ ើ ងយឃើ ញថា

និង

។ យបើ

យ េះរបភាគយនេះជារបភាគសរមួលរួចយ ើ ចំ នន ួ ចដលជាព ុ គុណនន ចំ នន ួ ចដលជាព ុ គុណនន ចំ នន ួ ចដលជាព ុ គុណនន

យ ើ

ផងនិង

ដូយចនេះ ចំ នន ួ ចដលជាព ុ គុណនន

យ ើ

និងនឹង

តូចជាងឬយសមើ

មានចំ នន ួ

ចចកមិនដាច់ នឹង

មានចំ នន ួ

។ ផង មានចំ នន ួ មានចំ នន ួ

។ ។

លឹម សុ វណ្ណវិចត្ិ រ| ត្រព័ន្រធ បារ់

109


ចំ នន ួ ចដលមិនចមនជាព ុ គុណនន របភាគមានរាង

ចដល

និងនន

និង

មានចំ នន ួ

ឋិ តយៅកនង ុ សំ ណំ ុ

។ យគមានរបភាគចបបយនេះចំ នន ួ

ចប

ប។ គួរសមាគល់ ថា យ ើ ងមិន ករបភាគមានរាង ចដលមិនចមនជាធាតុរបស់

ចដល

និង

យទ យរ េះរបភាគអស់ យនេះ ធំ ជាង

ដូយចនេះចំ នន ួ ភាគ កខុសៗគ្ននសរុប យៅកនង ុ សំ ណំ ុ យនេះយរោ សរមួលរួច មានចំ នន ួ 39. តាង

ជាចំ នន ួ គត់ វ ិជជមាន ចដលមាន ខទង់។ តាង

ចំ នន ួ គត់ តគ្នន

។ ។ ដូយចនេះរគប់

សុ ទចធ តោប់ យផតម ើ យដា យលខ

កនង ុ ចំ យោមយ េះចចកដាច់ នឹង

។ ដូយចនេះ ចំ នន ួ យ េះជាព ុ គុណនន

យ ើ

និងមានមួ មានរគប់ តួយលខព ី ដល់

។ 40. យៅកនង ុ យលខចដលឱ្យមកយនេះ យគមានចំ នន ួ គត់ វ ិជជមានមាន ខទង់ចំ នន ួ របស់ យលខយនេះយពលយគ ករតឹមយលខមាន

យដា

។ ចំ នន ួ ខទង់សរុប

ខទង់យ៉ាងយរចើន មកយរៀប កំណត់ យដា

យ ើ ងទាញបាន

។ ដូយចនេះ និង

។ ដូយចនេះ យរបើរតឹមយលខមាន មានយរចើនជាង យរបើ ដូយចនេះ 41. យ ើ ងយឃើ ញថា

មានន័ ថា ចំ នន ួ ខទង់របស់ យលខយនេះបយងកត ើ យដា ខទង់, មានតិចជាង

ខទង់ យ ើ

ខទង់។ ដូយចនេះ ទីតាង ំ ខទង់ទី

ឋិ តយៅរតឹម កយលខមាន

ខទង់, ខទង់មក

។ ។ ដូយចនេះ យ ើ ងគណ ។ យ ើ ងមាន

110

យដា យរបើរតឹមយលខមាន

ត្រព័ន្រធ បារ់ |លឹម សុ វណ្ណវិចត្ិ រ

ចដល


ដូយចនេះ

42.  យ ើ ងរតូវសរយសរ

យ ើ ងមាន

យ ើ ងទាញបាន

43.  យ ើ ងដឹងថា

ចដល

ដូយចនេះ ចំ នន ួ ចដលរតូវរកគឺ

។ យ ើ ងមាន

44. យ ើ ងមាន

។ ដូយចនេះ

តូចបំ ផត ុ យពល

និង

តូចបំ ផ ុ

ត។ យ ើ ងមាន

ចចកដាច់ នឹង

បីខង ទ ់ ខាងចុងរបស់ រតូវចតចចកដាច់ ចំ នន ួ គត់ វ ិជជមានតូចបំ ផត ុ

និង

មានតនមលដច ូ គ្នន។ យដា កតាតទីព ីរជាចំ នន ួ យសស យ េះ

ដូយចនេះ មួ

។ ដូចគ្នន

រតូវចតចចកដាច់

។ ដូយចនេះ មាន

ចដល

យដា

យ ើ ។ ដូយចនេះ

យរ េះយលខ

តាមរទឹសតីបទអឺលែ

។ លឹម សុ វណ្ណវិចត្ិ រ| ត្រព័ន្រធ បារ់

111


យដា នន

ជាព ុ គុណ ។ យដា

ករណី

យ េះ ឱ្ ំ យ

។ យ ើ ងព ិនិតយករណីនីមួ ៗ។

យ ើ ងមាន

មិនយផទៀងផ្ទទត់ មានន័ ថា មានន័ ថា

។ ដូចគ្ននចដរ

។ ដូយចនេះមានចត

។ យដា

យ េះ យ ើ ង ក

ជាចំ នន ួ គត់ វ ិជជមានតូចបំ ផត ុ ចដល និង

។ យ ើ ងទាញបាន

មានន័ ថា

45. យ ើ ងដឹងថា

ជាចំ នន ួ គត់ គ។ ូ យ ើ ងមាន (យរ េះយបើ

)

ដង

ខទង់ដំ បូងយរោ យកបៀសរបស់

សុ ទចធ តជាយលខ

ចំន្ន្ ួ គរ់ 46.  យ ើ ងមាន

យដា

និង

ជាចំ នន ួ បឋមរវាងគ្នន យ ើ

ចំ នន ួ គត់ យ េះ វារតូវចត

ចចកដាច់

47. យ ើ ងដឹងថា យបើ

យ េះ

យ ើ ងទាញបាន

112

ចំន្ន្ ួ គរ់ |លឹម សុវណ្ណវិចត្ិ រ

យដា ចផនកខាងស្តំរបស់ សមភាពខាងយលើជា

។ ។ ដូយចនេះតាមកំយណើន


តាមរទឹសតីបទឌ ីប៉ា ូលញ ី ៉ាក់ ចំ យ េះចំ នន ួ បឋម

មួ

សវ័ គុណ

ធំ បំផត ុ នន

ចដល

ចចកដាច់

កំណត់ យដា

សវ័ គុណ

ធំ បំផត ុ នន

ចដល

ចចកដាច់

កំណត់ យដា

យដា

យ េះ សវ័ គុណធំ បំផត ុ

ននចំ នន ួ បឋម

ោមួ

មានតនមលតច ូ ជាងឬយសមើ សវ័ គុណធំ បំផត ុ

ចដល

ចចកដាច់ ភាគចបងរបស់ របភាគ

ននចំ នន ួ បឋម

ោមួ

ចដល

ចចកដាច់ ភាគ ក

របស់ របភាគ។ មានន័ ថារបភាគយនេះជាចំ នន ួ គត់ ។ 48.  យ ើ ងកំណត់

មួ ។ តាង

ននរគប់ ចំ នន ួ គត់ យសសធំ មិនយលើសព ី

ជាចំ នន ួ គត់ ធំបំផត ុ ចដល

និង តាង

។ យ ើ ងមាន

ជាផលបូក ចដលតួនម ី ួ ៗជាចំ នន ួ គត់ ទាង ំ អស់ យលើកចលងចត ជាចំ នន ួ គត់ យទ

ជាផលគុណ

។ ដូយចនេះ

មិនអាច

មិនអាចជាចំ នន ួ គត់ យទ។

ត្រឹសីតររ បរើផលគុណ្កត្តតរឋមររស់ លែល

មាន្រាង

ជាចំន្ន្ ួ រួលចកវិជ្មា ជ ន្ររស់

ប ោះ បយើ ងមាន្ ។

49. រង្វវស់ មុំកនង ុ របស់ ព ុ យោណនិ ័ តមួ ចដលមាន រតូវចតចចកដាច់

។ យដា

រជុង យសមើ នង ឹ យ េះ យបើ

បឋមនឹង

។ ដូយចនេះមានន័ ថា យ េះ

លឹម សុ វណ្ណវិចត្ិ រ| ចំន្ន្ ួ គរ់

បឋម

113


នឹង

ចដរ។ យ ើ ងមាន

ចយមលើ គឺ

យរ េះ

មានតួចចកចំ នន ួ

ដូយចនេះយ ើ ងមិនគិតតួចចកយលខ

យ េះ និងយលខ ។

ភាពលចកដាច់ 50.  យ ើ ងមាន ជំនស ួ

យៅកនង ុ

យៅកនង ុ រូបមនត

យ ើ

យដា ដឹងថា

យរ េះ

យសស

យ ើ ងទាញបានរូបមនតចដលរតូវរស្ បញ្ជជក់ ។ 51. យ ើ ងមាន ចចកដាច់ នឹង ចចកដាច់ នឹង …………………………………………………. ចចកដាច់ នឹង ដូយចនេះ ផលបូកចចកដាច់ នឹង

52.  ក

យសស ។ យ ើ ងទាញបាន

ដូយចនេះ

ចចកដាច់ នឹង ចចកដាច់ នឹង … ដូយចនេះសំ យណើព ិត។ 53. យ ើ ងមាន យដា

យដា

ជាចំ នន ួ គត់ វ ិជជមាន ដូយចនេះ

54.  យ ើ ងមាន

114

ភាពលចកដាច់ |លឹម សុ វណ្ណវិចត្ិ រ

។ ដូយចនេះ

។ ចត


ដូយចនេះ មានន័ ថា យបើ

យ េះ

55.  យ ើ ងមាន ដូយចនេះ យបើ

ចចកដាច់ នឹង

យ េះ

ចចកដាច់ នឹង

ចដរ។

រោសមកវ ិញ យដា យ េះ យបើ

ចចកដាច់ នឹង

យ េះ

ចចកដាច់ នឹង

ចដរ។

56. យ ើ ងមាន

យដា ជាង

យ េះ

ជាចំ នន ួ គត់ ។ ចំ នន ួ គត់ ចដលធំ ជាង

ឋិ តយៅចយ ល េះ

និង

យ ើ

តូច

គឺ ។ ដូយចនេះ

57. យ ើ ងដឹងថា ផលយធៀប ចតយសមើ

ដាច់ ខាត យដា វាជាចំ នន ួ គត់ យ េះវារតូវ

។ ដូយចនេះ យ ើ ងរតូវយដាេះរស្ សមីោរ

យ ើ ងទាញបាន

ចំ នន ួ យនេះមិនចមនជាចំ នន ួ គត់ វ ិជជមានយទ ដូយចនេះ គ្នមនចំ នន ួ គត់

យទ ចដលយផទៀងផ្ទទត់ យទ។

58.  យ ើ ងមាន ក

យ ើ ងទាញបាន

59. ចំ នន ួ គត់ ទាង ំ អស់ អាចមានរាងមួ យបើ យបើ

ចចកដាច់ នឹង

កនង ុ ចំ យោមរាងទាំង

ជាចំ នន ួ បឋម យ េះវារតូវចតមានរាង ជាចំ នន ួ គូ យ េះ

(យរ េះយផេងយទៀត ចចកដាច់ នឹង

។ ឬនឹង )។

។ លឹម សុ វណ្ណវិចត្ិ រ| ភាពលចកដាច់

115


យបើ

ជាចំ នន ួ យសស យ េះ

ជាចំ នន ួ គូ

យ េះសំ យណើព ិត។ សនមតថា

60. យបើ

រគប់ តួទាង ំ អស់ របស់ ផលបូកយនេះ ចចកដាច់ នឹង 61. ជាដំ បូង យ ើ ងសនមតថា បោ ត

។ យ ើ ងមាន

។ ដូយចនេះផលបូកចចកដាច់ នឹង

ចំ នន ួ គត់ បនតប ទ ប់ គ្នន

។ ជាចំ នន ួ គត់

វ ិជជមាន។ យដា ដឹងថា យមគុណយទវធា ជាចំ នន ួ គត់

ដូយចនេះ មានន័ ថា ផលគុណ

ចចកដាច់ នឹង

យបើមានមួ កនង ុ ចំ យោមបោ ត ចំ នន ួ គត់ បនតប ទ ប់ គ្ននទាំងអស់ យនេះ យសមើ ចចកដាច់ នឹង

យ េះផលគុណយសមើ

យ ើ

យបើបោ ត ចំ នន ួ គត់ បនតប ទ ប់ គ្ននទាំងអស់ យនេះ សុ ទចធ តជាចំ នន ួ អវ ិជជមាន យ េះ ផលគុណយ េះយសមើ នង ឹ ផល គុណករណីវ ិជជមាន គុណនឹង

។ ដូយចនេះ វាក៏ចចកដាច់ នឹង

ចដរ។

62.  យ ើ ងមាន មានន័ ថា

ជាផលគុណននចំ នន ួ គត់ បីតយរៀងគ្នន។ កនង ុ ចំ យោមចំ នន ួ ទាំងយ េះ ដាច់ ខាតរតូវចត

មាន 

មួ ជាព ុ គុណនន

យរ េះចំ នន ួ គត់ បីតយរៀងគ្ននរតូវឋិ តយៅកនង ុ ចំ យោមចំ នន ួ ចដលមានរាង ។ យបើយផដម ើ យដា

យ េះយ ើ ងមានចំ នន ួ គូ

យដា យលខយសស យ ើ ងមានចំ នន ួ គូ 

និងមួ យទៀតជាព ុ គុណនន មានរាង គុណនន មាន

116

។ ដូយចនេះយ៉ាងោក៏រតូវចតចំ នន ួ គូមួ ចដរ។

យរ េះចំ នន ួ គត់ បីតយរៀងគ្នន រតូវឋិ តយៅកនង ុ ចំ យោមចំ នន ួ ចដល ។ យបើយផដម ើ យដា

មាន ។

ភាពលចកដាច់ |លឹម សុ វណ្ណវិចត្ិ រ

។ យបើយផដម ើ

។ យបើយផដម ើ យដា

យ េះយ ើ ងមានចំ នន ួ ព ុ

យ េះយ ើ ងមានចំ នន ួ ព ុ គុណនន


ដូយចនេះ

ជាព ុ គុណនន

នន

ផង និងនន

ផង។ យដា

បឋមនឹង

ដូយចនេះ

ជាព ុ គុណ

63.  យ ើ ងមាន យដា

ជាចំ នន ួ បឋម យ េះ

ជាចំ នន ួ យសស។ ដូយចនេះ

និង

គូទាង ំ ព ីរ ដូយចនេះផល

គុណរបស់ វាជាព ុ គុណនន ។ យដា

ដូយចនេះ

យបើ

មិនអាចជាព ុ គុណនន

យ េះ

។ យបើ

មានមួ កនង ុ ចំ យោម ដូយចនេះ

ជាព ុ គុណនន

គុណនន

យទ។ ដូយចនេះ

យ េះ

ជាព ុ គុណនន ផងនិង នន ផង យ ើ

។ ដូយចនេះរតូវចត

។ ដូយចនេះ យដា

ជាព ុ គុណនន បឋមនឹង

យ េះ

។ ជាព ុ

64.  យ ើ ងមាន និង

ជាចំ នន ួ គូ ជានិចយច រ េះ កនង ុ ចំ យោម

រតូវចតមានមួ ជាចំ នន ួ គូ យរ េះយបើមានមួ កនង ុ ចំ យោម

យសសទាំងព ីរយ េះ

យ េះ យបើយ ើ ង ក

ជាចំ នន ួ គូ យ េះវារតូវមានរាង យបើចំនន ួ គត់ ទាង ំ បីសុទចធ តចចកមិនដាច់ នឹង ឬ

គូ, យបើ និង យដា

ដូយចនេះចចកដាច់ នឹង

យ េះ ព ួកវាឋិ តយៅកនង ុ ចំ យោមរកុមចំ នន ួ មានរាង

។ យដា យគមានចំ នន ួ គត់ បី យ េះយគរតូវចតមានចំ នន ួ គត់ ព ីរចដលសថិ តយៅកនង ុ រកុម

(រកុម ចដលមានរាង

រតូវចតចចកដាច់ នឹង ។ យដា ដាច់ នឹង

គូ យ េះ

គូ។

យបើមានមួ កនង ុ ចំ យោមចំ នន ួ គត់ ទាង ំ បី មានរាង

ចតមួ

រតូវមានរាង

)។ ផលបូករ ឺយបើមិនអុី ចង ឹ យទ ផលសងននចំ នន ួ ព ីរយ េះ ជាចំ នន ួ គូ យ េះវារតូវមានរាង

ដូយចនេះចចក

65.  យបើ

យ េះ ឬ

យទ។ ដូយចនេះ មានចត

និង

មិនអាចចចកដាច់ នឹង យទ។ យបើ យ េះ

ចចកដាច់ នឹង

ទាំងព ីរ យទើប

មិនអាចចចកដាច់ នឹង ។

លឹម សុ វណ្ណវិចត្ិ រ| ភាពលចកដាច់

117


66. ជាដំ បូងយ ើ ងបង្វាញថា យបើ

ចចកដាច់ នឹង

ចចកដាច់

។ យដា

ដាច់ នឹង

។ កនង ុ ករណីទាំងព ីរ

ចចកមិនដាច់ នឹង

។ ដូយចនេះ យបើ

។ ឧបមាថា

រតូវចតចចក

ចចកដាច់ នឹង

យ េះ

។ ចចកមិនដាច់ នឹង

យ េះ

ចចកដាច់ នឹង ។ ដូយចនេះ យបើ

ចចកមិនដាច់ នឹង

ជាចំ នន ួ បឋម យ េះ

ចចកមិនដាច់ នឹង

រោសមកវ ិញ យ ើ ងបង្វាញថា យបើ ចចកមិនដាច់ នឹង

យ េះ

ចចកដាច់ នឹង

។ យបើ

។ យបើ

ចចកមិនដាច់ នឹង

យ េះ

។ យដា

យ េះ

យ េះ យ េះ

ចចកដាច់ នឹង

ចចកដាច់ នឹង

67. យ ើ ងមាន ចចកដាច់ នឹង ចចកដាច់ នឹង មានន័ ថា

ចចកដាច់ នឹង

ដូចគ្នន ចដរ ចចកដាច់ នឹង ចចកដាច់ នឹង មានន័ ថា កយនោមខាងយលើចចកដាច់ នឹង យដា

និង

មិនមានកតាតបឋមរួមគ្នន យ េះ កយនោមខាងយលើចចកដាច់ នឹង

68. ករណី

យ ើ ងមាន

សនមតថាវាព ិត រ ូ តដល់

ជាព ុ គុណនន មានន័ ថា

។ ព ិត។ ចំ យ េះចំ នន ួ គត់

ោមួ ។ យ ើ ងមាន

ព ុ គុណនន ដូយចនេះ 118

ព ិត។ ជាព ុ គុណនន

ភាពលចកដាច់ |លឹម សុ វណ្ណវិចត្ិ រ

ចំ យ េះរគប់ ចំ នន ួ គត់ ធមមជាតិ


69. ករណី យដា យបើ

ជាចំ នន ួ យសស យ េះ គូ យ េះ

ចដល

ចចកដាច់ នឹង

។ យបើ

យ ើ

ចចកដាច់ នឹង

យសស តាង

យ េះ

ចចកដាច់ នឹង ករណី

ជាចំ នន ួ គត់ ធមមជាតិ។ ដូយចនេះ ។ ដូយចនេះ

ចចកដាច់ នឹង

ចចកដាច់ នឹង

ដូយចនេះ

យ ើ

ចចកដាច់ នឹង

។ ដូយចនេះ

សំ យណើព ិត។

ឧបមាថា សំ យណើព ិតដល់

មានន័ ថា

ចចកដាច់ នឹង យដា

ចចកដាច់

។ យ ើ ងរតូវបង្វាញថា

។ យ ើ ងមាន

ចចកដាច់ នឹង

យសស យ េះ

យ េះ យ ើ ងនឹងបង្វាញថា

គូ ដូយចនេះចចកដាច់ នឹង

ចចកដាច់ នឹង

។ យដា

។ សំ យណើព ិត។

70. យ ើ ងមាន តាមសមភាពយនេះ យ ើ ងទាញបានថា i) យបើ

ចចកដាច់

យ េះ

ii) យបើ

ជាចំ នន ួ ព ុ គុណ យ េះ

យ ើ ងនឹងបង្វាញថា មាន ដូយចនេះយ ើ ងទាញបាន

ចចកដាច់ ក៏ជាចំ នន ួ ព ុ គុណចដរ។

យរចើនរាប់ មិនអស់ ចដលចចកដាច់ ចចកដាច់

។ យ ើ ងរតូវបង្វាញថា

។ យ ើង ក ចចកដាច់

យទៀត។ យ ើ ងនឹង

បង្វាញតាមវ ិោរយដា កំយណើន។ យបើ

យ េះ

យ ើ ងមាន ចចកដាច់ នឹង ចចកដាច់

ចចកដាច់ ចចកដាច់ នឹង

ព ិត។ សនមតថា ព ិតចំ យ េះចំ នន ួ គត់ តាមកំយនើន យ ើ

មួ ។ យ ើ ងមាន

ជាចំ នន ួ គូ ដូយចនេះ

ដូយចនេះសំ យណើព ិត។ ដូយចនេះ យ ើ ងបានបង្វាញថា មាន

យរចើនរាប់ មិនអស់ ចដល

។ លឹម សុ វណ្ណវិចត្ិ រ| ភាពលចកដាច់

119


យដា

ជាចំ នន ួ ព ុ គុណ យ េះ

យដា

ចចកដាច់

ក៏ជាចំ នន ួ ព ុ គុណចដរ(តាម ii))។

យ េះ

ចចកដាច់

(តាម i))។

ដូយចនេះជាសរុប យ ើ ងបានបង្វាញថា មាន ចដល

ចដល

ចចកដាច់

71. យ ើ ងយឃើ ញថា

និង

។ រតូវចតមានយលខ

។ ដូយចនេះ

កនង ុ ផលបូកយលើ យបើ យបើ

យរចើនរាប់ មិនអស់

ខទង់។ តាង

និង

លុ េះរតាចត មាន មួ

ចដល

មានន័ ថា គ្នមនតួ

យ េះ មានចត

យ ើ

ចដល

យទ។

យដា បូក ផលបូកទាំងយនេះបញ្ចូ លគ្នន យ ើ ងទាញបាន យដា

ជាចមាលស់ របស់

យ េះ យ ើ ងទាញបាន

ដូយចនេះ រតូវចតជាចំ នន ួ យសស ចចកដាច់ នឹង

។ ដូយចនេះ

72.  តាមរទឹសតីបទននភាពចចកដាច់ នឹង ចចកដាច់ នឹង

និង រោសមកវ ិញ។ ចតចំ នន ួ យនេះ ចចកដាច់ នឹង

ធំ បំផត ុ ចដល

ចចកដាច់ ចំ នន ួ យនេះគឺ

73.  យ ើ ងមាន

ចំ យ េះ 120

ចំ នន ួ យនេះ ចចកដាច់ នឹង

យ ើ ងមាន ភាពលចកដាច់ |លឹម សុ វណ្ណវិចត្ិ រ

យបើ

ចតចចកមិនដាច់ នឹង

យទ។ ដូយចនេះ


ដូយចនេះ

ឱ្ ំ យ

យ ើ ងទាញបាន

។ យដា

យ េះ

ចចកមិនដាច់

ចចកដាច់ រគប់ តួបក ូ ទាំងអស់ កនង ុ (*) ដូយចនេះ

។ ដូយចនេះរតូវចត

រតូវចចកដាច់

74. យ ើ ងយរៀបផលបូកជា

យ ើ ងមាន

ដូយចនេះ ផលបូករបស់ របភាគយនេះ មានកតាតរួមរបស់ ភាគ កយសមើ ភាគចបង ចដលមានកតាតផលគុណដូចជា

សុ ទចធ តតូចជាង

យ េះ គ្នមនកតាតោមួ របស់ ភាគចបងអាចសរមួល

បានយទ។ ដូយចនេះ

ដចដល យរោ បូករបភាគទាំងអស់ បញ្ចូ លគ្ននមក។ ដូយចនេះ 75. 

ចចកដាច់

យ េះ មាន

ជាតួចចករួមធំ បំផត ុ រវាង

និង

។ តួនម ី ួ ៗននផលបូកខាងយលើមាន

ចចក

ចដល

យ េះ

។ យដា

ជាចំ នន ួ បឋម

យៅកនង ុ ភាគ កយៅរកោ

ដាច់ ។ ។ ដូយចនេះ

ចដល

បឋមនឹង

។ យបើ ។ ដូយចនេះ

។ អងគខាងយឆវងននសមភាពយនេះ ជាចំ នន ួ គត់ ដូយចនេះមានចត ។ ដូយចនេះ

និង

ជាចំ នន ួ បឋមនឹងគ្នន។ តាមរយបៀបដូចគ្ននយ ើ ងទាញបាន

បឋមនឹងគ្នន

ព ីរៗ។

លឹម សុ វណ្ណវិចត្ិ រ| ភាពលចកដាច់

121


យបើ

ចចកដាច់

យ េះ វាចចកដាច់ ។ យដា

។ ដូចគ្នន

និង

បឋមនឹងគ្ននព ីរៗ យ េះយ ើ ងទាញបាន

។ យ ើ ងមាន

យដា

ជាចំ នន ួ គត់ យ េះ

ជាដំ បូងយ ើ ងសនមតថា

យដា

។ ដូយចនេះ

។ យ ើ ងទាញបាន

។ សមីោរ (*)យៅជា

យដា

យបើ

យ េះ

សមីោរគ្នមនចំ យលើ ។ យបើ

យ េះ

យ ើ ងយផទៀងផ្ទទត់ យឃើ ញថា

ជាចយមលើ របស់ សមីោរ។

ដូយចនេះជាសរុបសមីោរមានចយមលើ

និង ចមាលស់ របស់ វា ចដលមាន ។

122

ភាពលចកដាច់ |លឹម សុ វណ្ណវិចត្ិ រ

រតូវចតចចកដាច់ ចចកដាច់


76. សនមត

ជាចំ នន ួ គត់

ជាគូចយមលើ ចដលរតូវរក ចដល

ជាចំ នន ួ គត់ ។ ដូយចនេះ

ក៏ជាចំ នន ួ គត់ ចដរ។ មានន័ ថា យបើ

ចយមលើ ចដរ។ ដូយចនេះជាដំ បូងយ ើ ងយរៀប តាង

ជាចយមលើ

យ េះ

ក៏ជា

សិ ន។

។ យ ើ ងទាញបាន

ចចក

ដាច់ ។ ចំ យ េះចំ នន ួ គត់

ោមួ ។

ចំ យ េះ

យបើ

យ េះ

យរ េះ

ចចក យបើ

ដូយចនេះ មានចត

យ េះ

យ ើង ក យបើ

ដាច់ ។ យដា

(យរ េះយ ើ ងបានសនមតថា

)។

យ េះ លឹម សុ វណ្ណវិចត្ិ រ| ភាពលចកដាច់

123


យ ើ ងទាញបាន

ដូយចនេះជាសរុបចយមលើ គឺ

និង ចមាលស់ របស់ វា ។ ។

77. តាង យបើ

យ េះ

មិនព ិត។ យបើ

យ េះ

ដូយចនេះ មានចត

មិនព ិត។ ។ ដូយចនេះ

រោសមកវ ិញ យបើ

ដូយចនេះ ចំ យ េះ

យ េះ

78. ដំ បូងយ ើ ងព ិនិតយករណី ។ យដា

យដា ដាច់

យរ េះ

យ េះ 124

ចចកដាច់

។ ដូយចនេះ មានចំ នន ួ គត់

យសស យ េះ

គូ។ តាង

និង

យ េះ

ចដល

។ យដា ។

យ ើ

យផទៀងផ្ទទត់

យ ើ

គូ។

មតងយនេះយ ើ ងសនមតថា

ចតយដា

យ េះ

ចចកដាច់ នឹង

យ េះ

ចដល

យ េះ

ជាចំ នន ួ បឋម យ េះ

។ យដា ។ តាង

ភាពលចកដាច់ |លឹម សុ វណ្ណវិចត្ិ រ

។ យ ើ ងមាន

ចចក


យរ េះ

ជាចំ នន ួ បឋម និង

តាង

។ យ ើ ងមាន យរ េះ

79.  សនមតថា បឋម

។ ដូយចនេះ

ចចក

ចចកដាច់ នឹងរគប់ ចំ នន ួ គត់

យ ើ

តាង

។ យ ើ ងមាន

និង

ដាច់ ។ ។ តាង

ជាបោ ត ចំ នន ួ

ជាចំ នន ួ គត់ មួ ចដល

។ ដូយចនេះ

តាង

តាមសមមតក ិ មម

រតូវចចកដាច់ នឹងរគប់ ចំ នន ួ គត់

។ ដូយចនេះ មានន័ ថា ចំ នន ួ បឋមចដល យ ើ ងយឃើ ញថា

ចដល

ដូយចនេះ

រតូវចចកដាច់ នឹង

។ ដូយចនេះ

។ យដា

ជាបោ ដ

។ យ ើ ងព ិនិតយចំ នន ួ គត់ មដងមួ ៗ ព ី ដល់

យផទៀងផ្ទទត់ ចំយ េះ

។ ដូយចនេះ

ជាចំ នន ួ ធំ បំផត ុ ចដលចង់

បាន។ 80.  យ ើ ងមាន កនង ុ ចំ យោម

និង

។ តាង យបើ

ចដល

យ េះ

ចចកមិនដាច់ នឹង ចតយបើ

យ េះ

រតូវមានមួ គូ និងមួ យទៀតយសស។ កតាតចដលគូ មានតនមល ជាចំ នន ួ យសស។

ចដលមានកតាតយលខគូមានតនមល តច ូ ជាង

មិនអាចចចកដាច់ នឹង

យទ។ ដូយចនេះ

។ ចចកដាច់ នឹង

ចំ យ េះ

ចដល

លឹម សុ វណ្ណវិចត្ិ រ| ភាពលចកដាច់

125


ដូយចនេះកនង ុ ករណីយនេះ មានតនមល

ចដល

យ ើ

ចចកដាច់ នឹង

ចដលផទុ ព ីលកខ

ខណឌ។ 81. យ ើ ងមាន យបើ

ជាចំ នន ួ យសស យ េះ

ជាផលបូកននចំ នន ួ យសសចំ នន ួ

យនេះជាចំ នន ួ យសសចដរ។ ដូយចនេះ យបើ

ជាចំ នន ួ គូ តាង

ចំ នន ួ ទាំងព ីរគឺ របស់

មិនអាចជាព ុ គុណនន ។ លកខខណឌយៅជា

និង

ដូយចនេះផលបូក

បានយទ។ ដូយចនេះមានចត

ចចកដាច់

មានផលសងយសមើ

ដូយចនេះ

របស់ ចំ នន ួ ព ីរយនេះ ជាតួចចក

។ ម៉ា ាងវ ិញយទៀតចំ នន ួ ទាំងយនេះជាចំ នន ួ គូ ដូយចនេះ

បានថា យបើមិន

ក៏

។ យ ើ ងទាញ

ចដរ ចដលចចកដាច់ នឹង

វ ិសមភាពយនេះមិនព ិតយទ យបើ

។ ចតរតូវចត

។ យ ើ ងយផទៀងផ្ទទត់ យដា នដយឃើ ញថា

យមលើ ។ ដូយចនេះចយមលើ គឺ

ជាច

82.  តាង

យដា

ចចកដាច់

យបើ

យ េះ

ជាចំ នន ួ គត់ ។ ដូយចនេះ

យ េះ

មិនអាច។ ដូយចនេះ

យដា ដឹងថា ចំ យ េះរគប់ ចំ នន ួ ព ិត យបើ

ចចកដាច់ នឹង ចដល

យ េះរតូវចត

ដូយចនេះ បានយទ។ ដូយចនេះ

រតូវចតយសស ដូយចនេះ

ភាពលចកដាច់ |លឹម សុ វណ្ណវិចត្ិ រ

យ ើ ងមាន

យរ េះ

។ ដូយចនេះ

យសស ដូយចនេះ 126

។ ដូយចនេះ ចំ យ េះ

។ ចតយនេះ

ចចកដាច់ នឹង សនមតថា

យ ើ ងមាន

។ ជាចំ នន ួ គូ មានន័ ថា

។ សមីោរយៅជា

មិនអាច ជាចំ នន ួ


យពលយនេះ សនមតថា

យបើ

យដា

ដូយចនេះ

យ ើ ងមាន

យពល

។ ដូចយនេះមានចត

។ ករណីយនេះ

យបើ

យដា

ដូយចនេះគ្នមនចយមលើ ។

យបើ ចត

ដូយចនេះគ្នមនចយមលើ យទៀត។ ជាសរុប ចយមលើ មានចត

83. តាង

យ េះ

យ ើ ងមាន

ដូយចនេះ

និង យសស។ យ ើ ងរតូវបង្វាញថា

ចចកដាច់

។ ។

តាមរទឹសតីបទអឺលែ យ ើ ងទាញបាន

ដូចគ្ននយ ើ ងទាញបាន ដូយចនេះ

។ យដា

ជាចំ នន ួ យសស យ េះ

84. យ ើ ងមាន លឹម សុ វណ្ណវិចត្ិ រ| ភាពលចកដាច់

127


តាមសមមតក ិ មម (i) និង (ii) យ ើ ងទាញបានសមមតក ិ មម ថ ម ី (i)

ចចកមិនដាច់ នឹង

(ii)

ចចកដាច់ នឹង

យដា

។ យដា ស្កយលខមតងមួ ៗ យ ើ ងទាញ

បាន

ជាចយមលើ មួ

យរ េះ

85. យ ើ ងមាន យបើ

យ េះ

យបើ

យ ើ

។ យដា

ជាចំ នន ួ បឋម យ េះ

បទលែម៉ា យ ើ ងទាញបាន ។ ដូយចនេះ

។ ដូយចនេះ តាមរទឹសតី ។ ដូយចនេះ

ចំ យ េះរគប់

86. យ ើ ងមាន

យដា

។ យដា 87. តាង យ ើ ងសយងកតយឃើ ញថា

128

យ េះ តាមរទឹសតីបទលែម៉ា,

ជាចំ នន ួ បឋម យ ើ

ភាពលចកដាច់ |លឹម សុ វណ្ណវិចត្ិ រ

យ ើ

ជាចំ នន ួ គត់ យ េះ ។ តាង

ចចកដាច់

ចចកដាច់


តាមរទឹសតីបទលែម៉ា ចំ យ េះចំ នន ួ បឋម

មួ

យ ើ ងទាញបាន

ដូយចនេះ (*) យ ើ ងទាញបាន

យដា

ទាំងអស់ យនេះសុ ទចធ តជាចំ នន ួ បឋម យ េះយ ើ ងទាញបាន

88. ចំ យ េះចំ នន ួ បឋមយសស

ោមួ

យ ើង ក

តាមរទឹសដីបទលែម៉ា យ ើ ងមាន

យ េះ ចំ យ េះចំ នន ួ គត់

89. យបើ បឋមយសស

តូចបំ ផត ុ ។ យ ើ ងមាន

ោមួ ចចកដាច់

យ េះ

រតូវចតជាចំ នន ួ យសស និង មានតួចចក

យ ើ

ចចកដាច់

។ តាង

ជាចំ នន ួ គត់ តច ូ បំ ផត ុ ចដល

យបើ

យ េះ

ចដល ថា

តូចជាងយគ។ ដូយចនេះ

។ តាង ដូយចនេះយ ើ ងមាន មានន័ ថា

។ ដូយចនេះ

ចដល ចចកដាច់

ផទុ ព ីសនមត ិ ។ យដា

ជាចំ នន ួ យសស ដូយចនេះ

ក៏យសសចដរ។

លឹម សុ វណ្ណវិចត្ិ រ| ភាពលចកដាច់

129


តាមរទឹសតីបទលែម៉ា យ ើ ងមាន

។ យដា

យ េះ ជាងយគរបស់

។ ដូយចនេះ

ដូចគ្ននចដរ តនមល

ដាច់

។ ផទុ ព ីសនមតចិ ដល

ជាតួចចកតូច

90.  តាមរទឹសតីបទឌប ៉ូ ីញ ៉ា ក់ តនមល ី ៉ាែ

ដូយចនេះ តនមល

ជាចំ នន ួ តូចជាងយគចដល

ធំ បំផត ុ ចដល

ធំ បំផត ុ ចដល

ចចកដាច់

ចចកដាច់

ធំ បំផត ុ ចដល

ចចកដាច់

យសមើ នង ឹ

យសមើ នង ឹ

។ យដា

យសមើ នង ឹ

។ ដូយចនេះ

ចចកមិន

យទ។

91. រគប់ ចំ នន ួ គត់ កង នុ ចំ យោមចំ នន ួ គត់ ទាង ំ អស់ ចដល

ជាចំ នន ួ យសស។ ឧទា ។ ចយ ល េះ

មានចំ នន ួ យសស

ចំ យោមចំ នន ួ គត់ ទាង ំ

, ចំ នន ួ យសសមានចំ នន ួ ចត ចតប៉ា ុយោណេះ គឺ និង

សំណ្ល់. ភាពសមមូល 92.  i) យ ើ ងមាន

130

ចតប៉ា ុយោណេះ មានន័ ថាយ ើ ង ។ ដូយចនេះ កនង ុ

ចដលយគយរជើសយរ ើសយ េះ រតូវចតមានចំ នន ួ គត់ ព ីរ ចដលមាន

យសមើ គ្នន។ តាងចំ នន ួ ព ីរយ េះយដា

បានទាំង

រណ៍

និង

យផេងគ្ននចំ នន ួ ចត

យ េះ សុ ទចធ តអាចសរយសរជារាង

សំណ្ល់. ភាពសមមូល |លឹម សុ វណ្ណវិចត្ិ រ

ចដល

។ ដូយចនេះ ករណីយនេះ

មានតនមល ចចកដាច់


ii) យ ើ ងមាន

សំ យណើព ិត។ ។ តាមរទឹសតីបទលែម៉ា យ ើ ងមាន

93. យ ើ ងមាន

និង

។ រគប់ ចំ នន ួ គត់ វ ិជជមានយសសសុ ទចធ តបឋមនឹង

។ យ ើ ងមាន

។ តាមរទឹសតី

បទអឺលែ យ ើ ងទាញបាន

។ ដូយចនេះ

ចំ យ េះ

និង

។ យដា

និង

បឋមនឹងគ្ននព ីរៗ យ េះ

យំ អា

ត្រឹសីតររលភមា៉ា ត្តង

ជាចំន្ន្ ួ រឋមន្ិងសន្មរថា

លចក មិន្ដាច់ ប ោះ

94. តាមវ ិធីចចកចបបអឺ គដ លី

ចដល

ជាចំ នន ួ គត់ ។

យ ើ ងទាញបាន

ដូយចនេះ មានន័ ថា យដា

និង យ េះ

។ យដា ។ ដូយចនេះ

ដូយចនេះ

។ ។

ចំន្ន្ ួ កាបរ 95. ចំ នន ួ គត់ ទាង ំ អស់ អាចមានរាងមួ

កនង ុ ចំ យោម

និង

។ យដា យលើកវាជាោយរ

យ ើ ងទាញបាន

លឹម សុ វណ្ណវិចត្ិ រ| ចំន្ន្ ួ កាបរ

131


96. ោយរននចំ នន ួ គត់ មួ មានរាង

។ រគប់ ចំ នន ួ ទាំងអស់ យៅកនង ុ សវុី តយនេះ មានរាង

ដូយចនេះ វាមិនអាចជា ជាោយរននចំ នន ួ គត់ មួ យទ។ យ េះ

97. យបើ

ចត យដា

ជាចំ នន ួ យសស យ េះ

។ ដូយចនេះ

។ 98. 

យៅកនង ុ យគ្នល

កំណត់ យដា

យ េះ

99. យបើ

តាង

។ ដូយចនេះ

យៅចយ ល េះោយរននចំ នន ួ គត់ ព ីរតគ្នន ដូយចនេះ មានចត ។ ដូយចនេះ

យបើ

យ េះ

យបើ

យ េះ

132

ចំន្ន្ ួ កាបរ |លឹម សុ វណ្ណវិចត្ិ រ

មិនចមនជាចំ នន ួ ោយរ។

ជាោយរននចំ នន ួ គត់ ឋិត


ដូយចនេះ

យផទៀងផ្ទទត់

100. ចំ យ េះ

យ ើ ងមាន

យបើ

ជាចំ នន ួ ោយរ។

ជាចំ នន ួ គូ យ េះ

ឋិ តយៅចយ ល េះចំ នន ួ ោយរព ីរតគ្នន ដូយចនេះវាមិនអាចជា

ចំ នន ួ ោយរបានយទ។ យបើ

ជាចំ នន ួ យសស យ េះចយ ល េះ

និង

មានចតចំ នន ួ គត់

មួ គត់ ដូយចនេះរតូវចត

ដូយចនេះ ចំ យលើ គឺ

101. សនមតថា ថា មានគូ

ចដល ចដលយផទៀងផ្ទទត់

យ ើ ងអាចសនមតថា យបើ

ជាចំ នន ួ គត់ យរជើសយរ ើស ក

ជាចំ នន ួ ោយរ។ យ ើ ងសនមត

ជាគូចដលតូចជាងយគ។

យរ េះ កយនោមខាងយលើមានលកខណៈ សុី យមរទីយធៀបនឹង

យ េះ យបើ

យបើ

ជាចំ នន ួ គត់ ។ យ ើ ងរតូវបង្វាញថា

ជាចំ នន ួ គត់ យ េះ

ជាចំ នន ួ ោយរ។

យ េះ

ជាសមីោរដឺយរកទីព ីរ យធៀបនឹង

ចដលមានផលបូកឫសយសមើ

និង ផលគុណឫសយសមើ

(តាមរទឹសតីបទចវយត)។ តាង

ជាឫសរបស់ វា ជាចំ នន ួ គត់ យ េះ ។

ដូយចនេះ

ក៏ជាចំ នន ួ គត់ ចដរ។ យ ើ ងមាន

និង

។ យដា ជាចំ នន ួ គត់ វ ិជជមាន យ ើ

និង

មិនអាចធំ ជាងសូ នយបានយទ យរ េះ យបើមិនអុី ចង ឹ យទ

លឹម សុ វណ្ណវិចត្ិ រ| ចំន្ន្ ួ កាបរ

133


មានន័ ថា មាន

ជាចំ នន ួ គត់ វ ិជជមាន(តូចជាងឬធំ ជាង

ក៏បាន) ចដល

មិនចមនជាចំ នន ួ គត់ ចដលតូចជាងយគ ចដលផទុ ព ីសនមត។ ិ ដូយចនេះ

ដូយចនេះ

និង

ចដល

យ ើ

។ ចត

ជាចំ នន ួ គត់ ។ ដូយចនេះ

យំ អា

ជាចំ នន ួ ោយរ។ 102. យ ើ ងមាន បូក

យដា យលើកវ ិសមភាពយនេះជាោយរ យ ើ ងទាញបាន

ចូលអងគទាង ំ ២ យ ើ ងទាញបាន

យបើ

ដូយចនេះ

យ េះ យ ើ ងទាញបាន

មិនព ិត។ ដូយចនេះ

។ ដូយចនេះ

ដូយចនេះសមភាពអាចយកើតមាន យពល ។ ដូយចនេះ ដូយចនេះ

រតូវចតចចកដាច់

បឋមនឹង

103. ក) សនមតថា

ដូយចនេះ

យ ើ

។ តាង

ចចកដាច់

មិនអាចយទ យបើ

។ យដា ផលគុណរបស់ វាជាចំ នន ួ ោយរ យ េះ ជាចំ នន ួ ោយរ។ យដា

ជាតួចចកបឋមរួមរបស់

និង

និង ។

ក៏រតូវចតជាចំ នន ួ ោយរចដរ។ បឋមនឹងគ្នន យ េះវារតូវចតជាចំ នន ួ

ោយរទាំងព ីរ។ ចតមិនចដលមានចំ នន ួ ោយរព ីរខុសគ្ននមួ ឯកតាយទ យលើកចលងចត

និង

យរ េះ

។ ខ) សនមតថា

ជាចំ នន ួ ោយរ។ យដា

គ្នន យ េះវារតូវចតជាចំ នន ួ ោយរទាំងព ីរ។ ចតយដើមបីឱ្យ យពលយ េះ ផលគុណ

134

ចំន្ន្ ួ កាបរ |លឹម សុ វណ្ណវិចត្ិ រ

យសមើ សូនយ។

និង ជាោយរននចំ នន ួ គត់ ទាល់ ចត

បឋមនឹង ចត


គ)

អាចជាចំ នន ួ ោយរ មានចត ។

104. ព ិនិតយចំ នន ួ គត់

។ យដា

យ េះ យដើមបីឱ្យផលគុណចំ នន ួ ទាំងព ីរជាចំ នន ួ សវ័ គុណទី គុណទី

និង

ជាចំ នន ួ សវ័ គុណទី

បឋមនឹង

។ យ ើ ងទាញបាន

ចំ យ េះ

និង ចំ យ េះ

បឋមរវាងគ្នន

យ េះវារតូវចត

ចដរ។ ដូយចនេះ

និង

និង

ជាចំ នន ួ សវ័

និង

ចដល

យ ើ ងមានកតាតទីព ីរននអងគខាងយឆវងធំ ជាង

ដូយចនេះ អងគ

ខាងយឆវងនិងស្តំមិនអាចយសមើ គ្ននបានយទ។ 105. តាង

ជាចំ នន ួ មិនោយរទី

មានចំ នន ួ ោយរចំ នន ួ

។ យគមានចំ នន ួ គត់ ធមមជាតិ

ចដលតូចជាង

និង ចំ នន ួ មិនោយរចំ នន ួ

ចដល មករតឹម

។ ដូយចនេះយ ើ ងយឃើ ញថា

។ ដូយចនេះ

យដា

សុ ទចធ តជាចំ នន ួ គត់ យ េះវ ិសមភាពខាងយលើយៅជា

មានន័ ថា

106. តាង ។ យបើ

មានន័ ថា

ជាចំ នន ួ មិនោយរ ឃ្លលតព ីចំ នន ួ ោយរ

ចំ នន ួ

ជាចំ នន ួ ោយរ។

លឹម សុ វណ្ណវិចត្ិ រ| ចំន្ន្ ួ កាបរ

135


យ ើ ងចចក សំ ណំ ុ នន

ជាសំ ណំ ុ រងព ីរ ចដលកនង ុ យ េះ

ជាសំ ណំ ុ នន

ចដលធំ ជាងចំ នន ួ ោយរចំ នន ួ ចដល

ជាសំ ណំ ុ នន

ចដលធំ ជាងចំ នន ួ ោយរចំ នន ួ ចដល

យ ើ ងសយងកតយឃើ ញថា ទាំងករណី សនមតថា

។ យដា

និង

។ ។ យ ើ ងមាន

យ េះ

ចដល

។ ដូយចនេះ

ដូយចនេះ យ ើ ងោប់ យផដម ើ ព ីករណី

ចតមដងក៏បាន។ ករណីយនេះ យ ើ ងមាន

ដូយចនេះ

មានន័ ថា ចំ នន ួ ោយរ

យរ េះ ចំ នន ួ

(យរ េះ

។ យ ើ ងសយងកតយឃើ ញថា ) ប ទ ប់ មក

ឃ្លលតព ីចំ នន ួ ោយរ

ឃ្លលតព ី ចំ នន ួ

។ ដូយចនេះ ជំហានបនតប ទ ប់ មកយទៀត តនមលចដលយលើសយ េះនឹងថ ចុេះោន់ ចតតូចយៅៗ រ ូ ត ដល់ យពលមួ តនមល យនេះយសមើ សូនយ។ យៅយពលយ េះ យ ើ ងនឹងទាញបានចំ នន ួ ោយរ។

136

ចំន្ន្ ួ កាបរ |លឹម សុ វណ្ណវិចត្ិ រ


ជាចយមលើ តូចជាងយគបងអស់របស់ សមីោរ

107. តាង យបើ

និង

សុ ទចធ តគូ យ េះ

។ ដូយចនេះ ដូយចនេះ

ឧបមាថា

ចចកដាច់ អងគខាងយឆវង ដូយចនេះ វាចចកដាច់

ជាចំ នន ួ គត់ យ ើ

ដូយចនេះ

ជាព ុ គុណនន

ក៏ជាចយមលើ របស់ សមីោរចដរ។ ករណីយនេះផទុ ព ី ោរសនមត។

ជាចំ នន ួ យសស។ គូ យ ើ

យសស យ េះ

មិនអាចជាចំ នន ួ ោយរបានយទ យរ េះចំ នន ួ ោយរសមមូលនឹង

តាម

។ ដូយចនេះ

និង

រតូវចត

យសសទាំងព ីរ។ ករណីយនេះ មិនអាចជាចំ នន ួ ោយរបានយទ។ ដូយចនេះ គូ

ចដល

ជាចំ នន ួ ោយរ មានចត

មួ ប៉ា ុយោណេះ។

108. ចំ នន ួ ចដលជាធាតុរបស់ សំ ណំ ុ យ េះ មានរាង

ដូយចនេះ ធាតុនម ី ួ ៗ មាន យ ើ ងមានបនេំ

មួ ចបបមានក់ ។ យបើគត ិ ព ីលកខណៈគូយសស នន ចំ នន ួ

មួ កនង ុ ចំ យោមករណីទាំង ចដលមាន

ចបប។ ឧទា

រណ៍ (គូ,យសស,យសស,គូ,យសស) ជាករណី

យ េះ។ យដា យ ើ ងមានចំ នន ួ គត់ ចំនន ួ

យ េះរតូវចតមានចំ នន ួ គត់ ព ីរ

តំយរៀបតាមលកខណៈគូយសសដូចគ្នន។ ដូយចនេះផលគុណននចំ នន ួ ព ីរយនេះជាចំ នន ួ ោ

យរ។ 109. ចំ នន ួ នីមួ ៗឋិ តយៅកនង ុ សំ ណំ ុ យបើគត ិ តាមលកខណៈគូយសស យ ើ ងមាន ដូយចនេះយបើយ ើ ងោប់

មានរាង ចំ នន ួ

ចបប។

ធាតុ យ េះ យ ើ ងនឹងទទួលបានធាតុព ីរខុសគ្ននចដលមាន មានលកខណៈគូយសសដូចគ្នន ដូយចនេះធាតុព ីរយនេះមានផលគុណជាចំ នន ួ ោយរ។

លឹម សុ វណ្ណវិចត្ិ រ| ចំន្ន្ ួ កាបរ

137


យដា យ ើ ងមាន

យ េះយ ើ ងអាចរកបានចំ នន ួ ខុសគ្ននមួ គូ

យ ើ ងដកមួ គូយនេះយចញ។ ព ីកនង ុ ចំ យោម គូ

ចដល

។ យ ើ ងដកមួ គូយនេះយចញ។ ព ីកនង ុ ចំ យោម

ចដល

ចដល

មានន័ ថា

គ្ននចដល និង

យៅសល់

។ យ ើ ងបនតដក ក រ ូ តបាន ។ ដូយចនេះ យ ើ ងអាចរបមូលបាន

។ ប ទ ប់ មកយទៀតបោ ត ចំ នន ួ គត់ យ ើ

យដា

ទាំង

។ ដូយចនេះ

គូ

គូចំនន ួ គត់ ខុស

មានតួចចកបឋមរបស់ វាតូច

យ េះ យ ើ ងអាចរកបានចំ នន ួ គត់

ចដល

យៅសល់ យ ើ ងអាចរកបានចំ នន ួ ខុសគ្ននមួ

យ ើ ងអាចរកបានចំ នន ួ ខុសគ្ននមួ គូ

ជាងឬយសមើ

ចដល

ព ីរខុសគ្នន តាងយដា

ជាចំ នន ួ សវ័ គុណទី៤។ ដូយចនេះមានន័ ថា

យ ើ ងអាចរកបានចំ នន ួ គត់ បួនចដលមានផលគុណរបស់ ជាចំ នន ួ សវ័ គុណទី៤។

ចំន្ន្ ួ លែលមាន្រាងណាមួយ ជាចំ នន ួ យសស យ េះយ ើ ង ក

110. យបើ យបើ

គូ យ េះ

យបើ

អាចជា

យ េះ យ ើ ង ក

។ ចំ នន ួ ព ីរយនេះបឋមរវាងគ្នន យរ េះយបើ យ េះ

យបើ

យដា

យ េះ យ ើ ង ក និង ចចកដាច់ នឹង

បឋមនឹង

។ ដូយចនេះ យ ើ ងរតូវរក

។ យ ើ ងមាន

ដូចយនេះ

។ យ ើ ងយផទៀងផ្ទទត់ យដា ោរជំនស ួ យលខមដងមួ ៗយឃើ ញថា ជាងយគចដល ចំ នន ួ គត់ វ ិជជមាន ចដល 138

111. យ ើ ងមាន និង ចដល

យសស យ េះ

។ ដូយចនេះ

ទាល់ ចត

ជាចំ នន ួ តូច ចំ យ េះ

ោមួ ។ ដូយចនេះ យ ើ ងរតូវរាប់ ចំ នន ួ ចយមលើ ជាចំ នន ួ គត់ វ ិជជមាន របស់ សមីោរ យ ើ

។ ដូយចនេះ

ចំន្ន្ ួ លែលមាន្រាងណាមួយ |លឹម សុ វណ្ណវិចត្ិ រ


ចំ យ េះ

សមីោរយៅ ជា

ដូយចនេះ

មាន

ចយមលើ ។ ចំ យ េះ មាន

សមីោរយៅ ជា

ដូយចនេះ

ចយមលើ ។

...... ចំ យ េះ

សមីោរយៅ ជា

ដូយចនេះ

មាន

ចយមលើ ។ ដូយចនេះជាសរុប មាន

ចយមលើ ។

112. រយបៀបទី១៖តួននសវុី តយនេះអាចសរយសរជា យគ្នល១០

(១)

យគ្នល៣

(២)

សំ យនរកនង ុ យគ្នល២

(៣)

យគ្នល១០

(៤)

ទំ ក់ ទំនង(១) ជាតួរបស់ សវុី តសរយសរកនង ុ យគ្នល១០។ (២) ជាតួរបស់ សវុី តសរយសរកនង ុ យគ្នល៣ ដូចជា មានន័ ថា

កនង ុ យគ្នល១០ យសមើ នង ឹ

កនង ុ យគ្នល៣។

ទំ ក់ ទំនង(៣) ជា(២) ប៉ា ុចនតោត់ ទក ុ ថាសំ យនរយនេះជាយគ្នល២។ (៤)ជាតំនលរបស់ (៣)កនង ុ យគ្នល១០។ ជារួមមានន័ ថា (១)=(២) និង (៣)=(៤) ។ ដូយចនេះតួទ១ ី ០០ននសវុី តយនេះ ជាយលខ១០០កនង ុ (៤)។ យ ើ ងទាញរក តំនលរតូវគ្នននន១០០យនេះ កនង ុ ទំ ក់ ទំ នងទី(៣)។ យ ើ ងសរយសរ១០០ យៅកនង ុ យគ្នល២គឺ ៣ គឺ

យ ើ

។ ដូយចនេះតួទ១ ី ០០របស់ សវុី តយសមើ នង ឹ

រយបៀបទី២៖យបើយ ើ ងយរបើចត៦តួដំបូងគឺ

បំ ចលងវាយៅជាយគ្នល ។ និង

យ ើ ងអាចបយងកត ើ បានសវុី តយនេះចំ នន ួ

លឹម សុ វណ្ណវិចត្ិ រ| ចំន្ន្ ួ លែលមាន្រាងណាមួយ

139


តួចតប៉ា ុយោណេះ។ យ ើ ងរតូវោរយ៉ាងតិច១០០តួ។ សវ័ គុណនន៣ធំ ប ទ ប់ គឺ ៦៣ខាងយលើ តួធំប ទ ប់ ជាបនេំនន

នឹង

ចដលមានទាំងអស់ ចំ នន ួ

ដូយចនេះយ ើ ងមានទាំងអស់ ចំ នន ួ យទៀត ដូយចនេះតួទ ី

។ ប ទ ប់ ព ីតួទាង ំ

តួ។ យដើមបីរកតួទ១ ី ០០ យ ើ ងរតូវបចនថម

គឺ ។

113. តាង ។ យ ើ ងមាន

ជាសំ យណើចដលថា «បោ ត ចំ នន ួ គត់ ព ិត។ យ ើ ងនឹងបង្វាញថា

សុ ទចធ តជាយលខព ិយសស» ក៏ព ិតចដរ មានន័ ថា «បោ ត ចំ នន ួ គត់

សុ ទចធ តជាយលខព ិយសស»។ ដូយចនេះ យ ើ ងរគ្នន់ ចតបង្វាញថា ជាយលខព ិយសស ជាបចនថមយទៀតបានយ ើ យ ើ ងនឹងបង្វាញថា យបើ

ជាយលខព ិយសស យ េះ

និង

។ និង

ក៏ជាយលខព ិយសសចដរ។ យ ើ ង

មាន

ដូចគ្ននចដរ

ដូយចនេះ យបើ

ជាយលខព ិយសស យ េះ

និង

114. ជាដំ បូងយ ើ ងបង្វាញថា

ក៏ជាយលខព ិយសសចដរ។ មិនអាចសរយសរជារាង

បានយទ។ យដើមបីរស្ បញ្ជជក់ នវូ អំ ណេះអំ ោងយនេះ ជាដំ បូងយ ើ ងសនមតថា អាចសរយសរបានសិ ន មាន ន័ ថា

140

ចំន្ន្ ួ លែលមាន្រាងណាមួយ |លឹម សុ វណ្ណវិចត្ិ រ


យដា

និង

យរ េះ

បឋមនឹងគ្ននព ីរៗ យ េះ

។ ដូយចនេះ

រតូវចចកដាច់

មិនអាចយសមើ សូនយបានយទ

។ ដូចគ្នន យ ើ ងទាញបានថា

និង

។ ចតយបើ

ដូយចនេះ យ េះ

ផទុ ព ីោរសនមត។ ដូយចនេះ

មិនអាចសរយសរជារាង

បាន

យទ។ ប ទ ប់ មកយទៀត យ ើ ងបង្វាញថា រគប់ ចំ នន ួ គត់ ទាង ំ អស់ ចដលធំ ជាង អាចសរយសរជារាង

បានទាំងអស់ ។

យ ើ ងនឹងបង្វាញថា រគប់ ចំ នន ួ គត់ ទាង ំ អស់ ចដល ធំ ជាង បានទាំងអស់ ។ យដា

បឋមនឹង

។ យបើ

ចដល

។ យដា

មានន័ ថា រគប់ ចំ នន ួ គត់ ធំជាង យដា ដឹងថា

និង

យ េះ យ េះ

សុ ទចធ តអាចសរយសរជារាង

បានទាំងអស់ ចដល ។ យ ើ ងមាន

និង

មាន

ចចកនឹង

ចដល

។ ដូយចនេះ

បានទាំងអស់ ។

បឋមនឹងគ្នន យ េះរគប់ ចំ នន ួ ទាំងអស់ ចដលធំ ជាង

សរយសរជារាង តាង

សុ ទចធ តអាចសរយសរជារាង

យ េះរគប់ ចំ នន ួ គត់ ទាង ំ អស់ យពលចចកនឹង

សំ ណល់ មួ កនង ុ ចំ យោម សល់

សុ ទចធ ត

សុ ទចធ តអាច

ជាចំ នន ួ គត់ វ ិជជមាន។ ដូយចនេះ ដូយចនេះយ ើ ងអាចសរយសរជា

រាងខាងយរោមបាន ចំ យ េះចំ នន ួ គត់ វ ិជជមាន

និង

ខលេះ។ ដូយចនេះ

យ ើ ងទាញបាន (*) & (**) យ ើ ងមាន

យ េះ

លឹម សុ វណ្ណវិចត្ិ រ| ចំន្ន្ ួ លែលមាន្រាងណាមួយ

141


យ ើ ងបានបង្វាញថា រគប់ ចំ នន ួ ទាំងអស់ ចដលមានរាង សុ ទចធ តអាចសរយសរជារាង 115. តាង

បាន។

ជាចំ នន ួ សនិទាន។ តាង

ជាចំ នន ួ សនិទាន។ យដា ន័ ថា រគប់

។ យដា យ ើ ងមាន

កនង ុ ករណីទូយៅ យ ើ ងសនមតថា

ដូយចនេះ

។ យ ើង ក

ចំ យ េះចំ នន ួ គត់ វ ិជជមាន

ចដល

។ ដូយចនេះ យគមាន

ជាចំ នន ួ សនិទានយៅចយ ល េះ

និង

និង

ចដល

។ តាមលទធផលខាងយលើ យ ើ ងទាញបាន

ោមួ ។ យ ើ ងទាញបាន

យ ើ ងទាញបានថា សំ យណើព ិត។

142

។ ដូយចនេះ

ចំន្ន្ ួ លែលមាន្រាងណាមួយ |លឹម សុ វណ្ណវិចត្ិ រ

។ មាន


ពហុ គុណរួមតូ ចបំ ផុត.តួ ចចក រួមធំ បំផុត ចំនន ួ បឋម 116.  សនមតថា

ជាចំ នន ួ ពហុ គុណ ហហើ យថា

ជាតួចចករបស់

ខុសព ីមួយ។ ហយើ ងមាន

និង

ហោយ

ខុសព ី ហ ោះ

ជាចំ នន ួ ពហុ គុណ។ ដូហចនោះ ហបើ

មិនចមនជាចំ នន ួ បឋម

មិនចមនជាចំ នន ួ បឋមហទ។ សម្គាល់ ហបើ

ជាចំ នន ួ បឋម

ហបើ

អាចជាចំ នន ួ បឋម ឬ មិនបឋម។

ជាចំ នន ួ បឋម

ក៏ជាចំ នន ួ បឋមចដរ។

117.  យក

មានកត្តាបឋមរួមគ្នន ហហើ យ

118. 

ជាចំ នន ួ បឋម។ ហោយ

ហដើមបីឱ្យ

ជាចំ នន ួ បឋម មានចត

។ ករណី

ធំ ជាង ជានិចហច ោះ ហយើ ងមាន

ជាចំ នន ួ បឋម។ 119.  ចំ ហ ោះ

ជាចំ នន ួ បឋម។ ព ិនិតយ លឹម សុ វណ្ណវិចត្ិ រ| ចំនន ួ បឋម

143


កត្តានីមួយៗ ធំ ជាង

ចំ ហ ោះ

120.  ហបើ

ជាចំ នន ួ បឋម ហ ោះ វារតូវចតជាចំ នន ួ ហសស ហហើ យ ឱ្ ំ យ

ចំ ហ ោះ

ដូហចនោះ

ហយើ ងមាន

មិនអាចជាចំ នន ួ បឋមបានហទ។ ហសស។

ជាចំ នន ួ បឋម

ករណី

ហយើ ងហឃើ ញថា ករណី ចដលធំ ជាង

ជាចំ នន ួ ហសស ហ ោះ

ដូហចនោះ វាមិនអាចជាចំ នន ួ បឋមហទ។

121.  ត្តង

ចដល

អាចបំ ចបកជាផលគុណននចំ នន ួ គត់ ព ីរ

។ ដូចហនោះ

122.  ហយើ ងហឃើ ញថា

។ ត្តមរទឹសាីបទ 3.10 រគប់ ចំ នន ួ ពហុ គុណទំងអស់ ចដល

មានកត្តាបឋមមួយ កនង ុ ចំ ហោម

ត្រឹសីត បរ 3.10៖ បបើចន ំ ន ួ គរ់ វជ្ ិ មា ជ ន ត្តង

144

ជាចំនន ួ ពហុ គុណ្ ប ោះ វាមានកត្តតបឋម

ជាសំ ណំ ុ ចំ នន ួ ពហុ គុណនន

ចដល

 100 ។ ដូហចនោះ

ពហុ គណ្ ុ រួមរូចបំផរ ុ .រួចចករួមធំបផ ំ រ ុ |លឹម សុ វណ្ណវិចត្ិ រ

មួយ ចែល


ជាចំ នន ួ ននចំ នន ួ ចដលជាពហុ គុណនន ទំង

ចដលកនង ុ ហនោះ ហយើ ងរាប់

ចដលជាចំ នន ួ បឋមចូលចដរ។ ដូហចនោះ ចំ នន ួ ននចំ នន ួ ចដលជាពហុ គុណនន

ហហើ យមិនចមនជាចំ នន ួ បឋម មានចំ នន ួ ដូហចនោះ ចំ នន ួ ចំ នន ួ បឋមចដល កនង ុ ហ ោះ ដក

ហរ ោះ

គឺ

មិនចមនជាចំ នន ួ បឋម ហហើ យក៏មិនចមនជាចំ នន ួ ពហុ គុណចដរ។

123.  ត្តង

។ ហយើ ងមាន ជាចំ នន ួ ហសសទំងអស់

ហរ ោះហបើមិនអុី ចង ឹ ហទ រគប់

ជាចំ នន ួ ហសស ដូហចនោះ

មិនអាចចចកោច់

ហទ ដូហចនោះ

ផទយ ុ ព ីសមមតក ិ មម។ ប ទ ប់ មកហទៀតហយើ ងសននោ ិ ា នថា ហ ោះ

។ ហរ ោះហបើ

រតូវចតមានរាង

ហ ោះ ហោយ

ដូហចនោះ

ជាចំ នន ួ បឋម ហហើ យ

ចំ ហ ោះរគប់

ផទយ ុ ព ីសមមតក ិ មម។ ជាចុងហរោយហយើ ងនឹងបង្ហាញថា ចំ ហ ោះរគប់

។ ហបើ

ហ ោះ

ដូហចនោះហយើ ងសននោ ិ ា នបានថា ចំ នន ួ បឋម៣តហរៀងគ្នន គឺ

ផទយ ុ ព ីសមម តក ិ មម។ ឋិ តហៅកនង ុ ចំ ហោមចំ នន ួ ចដលឱ្យ។

ចំនន ួ ពហុ គណ្ ុ 124.  ព ិនិតយ

។ ករណីហនោះ បោ ា ចំ នន ួ

សុ ទចធ តជាចំ នន ួ មិនបឋ

ម។ លឹម សុ វណ្ណវិចត្ិ រ| ចំនន ួ ពហុ គណ្ ុ

145


ពហុ គណ្ ុ រួមរូចបំផរ ុ . រួចចករួមធំបផ ំ រ ុ 125.  ត្តង

។ ដូហចនោះ

។ ដូហចនោះ

ចចកោច់

ចចកោច់ ។ ដូចគ្នន

។ ហោយ ។ ដូហចនោះ

ហ ោះ

មានន័យថា

126.  ត្តង

ដូហចនោះ : ចចកោច់ នឹង ចចកោច់ នឹង

ដូហចនោះ

a d  1។ ដូចគ្នន

។ (*)។

ត្តមរទឹសាីបទ Bachet-Bézout ហគអាចរកបានចំ នន ួ គត់ ថា

។ ដូហចនោះ

ចដល

។ គួរកត់ សមាគល់

រតូវចតមានសញ្ញាផទយ ុ គ្នន។ វាមិនអាចមានសញ្ញាដកទំង២ហទ ហរ ោះ

សញ្ញាបូកទំង២ហទ ហរ ោះ ឱ្ ំ យ

ចដលត្តមព ិត

។ ត្តង

។ ដូហចនោះ ហយើ ងសនមតថា

។ ដូហចនោះ

ដូហចនោះ

។ វាមិនអាច និង

(**)។ (*) និង (**)

សំ ហណើព ិត។

127.  ហយើ ងមាន

ដូហចនោះ មានន័យថា អាចមានតនមលហសមើ

ដូហចនោះ

146

ចំ ហ ោះរគប់ ចំ នន ួ គត់ ឬហទ?។ ឱ្យ

អាចមានតនមលរហូ តដល់

។ ដូហចនោះ

មិនអាចធំ ហលើសព ី

ហ ោះ

។ មានន័យថា

ពហុ គណ្ ុ រួមរូចបំផរ ុ .រួចចករួមធំបផ ំ រ ុ |លឹម សុ វណ្ណវិចត្ិ រ

ហទ។ ហតើ


128.  ត្តង

។ ដូហចនោះ

និង

ចំ ហ ោះចំ នន ួ គត់ ធមមជាតិ

ដូហចនោះ

ចដល

ដូចគ្នន

ហបើសិនជា

ហហើ យហោយ

រតូវចតជាចំ នន ួ ហសស និង ។

ជាចំ នន ួ ហសស ហ ោះ

ជាចំ នន ួ ហសស។ ហោយ

129. បោ ា ចំ នន ួ ចដលមានរាង ចំ នន ួ

និង មានផលសងរួម

ហ ោះ

បហងកត ើ បានជាសវុី តនព វនា ចដលមានតួ ។ សនមតថា

ដូហចនោះ

។ មានន័យថា តួព ីរៗននសវុី តហនោះ បឋម

រវាងគ្នន។ 130. ហយើ ងមាន

។ ដូហចនោះ ហយើ ងរតូវរកចំ នន ួ ននចំ នន ួ វ ិជ្ជមានចដល

ហហើ យចដលចចកមិនោច់ នឹង ជាពហុ គុណនន , ត្តង

និងនឹង ។ ត្តង

ជាសំ ណំ ុ ននចំ នន ួ គត់

ជាសំ ណំ ុ ននចំ នន ួ គត់

ចដលជាពហុ គុណនន

ចដល

,...។ល។

ហយើ ងដឹងថា

ដូហចនោះចំ នន ួ ចំ នន ួ គត់ ចដលរកគឺ 131. ត្តង ដូចគ្នន

ចចក

ជាតួចចករបស់ រួមរបស់ ោច់ ។ ហោយ

និង

និង

។ ដូហចនោះ

បឋមនឹងគ្នន ហ ោះ

និង

ចចកោច់

ក៏បឋមនឹងគ្ននចដរ។ ដូហចនោះ

លឹម សុ វណ្ណវិចត្ិ រ| ពហុ គណ្ ុ រួមរូចបំផរ ុ . រួចចករួមធំបផ ំ រ ុ

147


132. ហយើ ងដឹងថា

ត្តង

ជាតួចចករួមរវាង

ចចក

ោច់ ហ ោះ

ហោយសារ

និង

ក៏ចចក

និង

។ សនមតថា

។ ត្តមទំ ក់ ទំនងខាងហលើ ហោយសារ

ោច់ ចដរ។ ហោយសារ

ជាចំ នន ួ ហសស ហ ោះ

ចចក

ោច់ ហ ោះ

ជាតួចចកក៏ហសសចដរ ហហើ យចចក

ចចក

ោច់ ។

ោច់ មានន័យថា

។ 133. ហយើ ងមាន

និង

ចចកោច់ នឹង

។ ត្តមរទឹសាីបទភែម្គ៉ា ហយើ ងមាន

សនមតថា ដូហចនោះ

មានន័យថា

ចចកោច់ នឹង

។ ហោយ

បឋមនឹង

ហ ោះ

ចចកោច់ នឹង

ត្រឹសីតបរចភមា៉ា ត្តង

ជាចំនន ួ បឋមនិងសនមរថា

ចចក

មិនដាច់ ប ោះ

ដូហចនោះ រគប់ ចំ នន ួ បឋមទំងអស់ ចចកោច់ យ៉ាងតិចតួមួយរបស់ សវុី តហនោះ។ មានន័យថា មិនអាចមាន ចំ នន ួ ោមួយហរៅព ី ចដលបឋមនឹងរគប់ តួទង ំ អស់ ននសវុី តហនោះហទ។ 134. 

និង

ជាតួចចករបស់

គត់ មិនអវ ិជ្ជមាន

ខលោះ។ ហោយ និង

ដូហចនោះ

ដូហចនោះ ជា

នន

ចំ ហ ោះ ចំ នន ួ ដូហចនោះ

អាចជា

។ ដូហចនោះហយើ ងមាន

ចំ នន ួ ចបប។ ដូចគ្ននហយើ ងមាន

និង

ចបបននចំ នន ួ គត់ វ ិជ្ជមាន 148

និង

ចំ នន ួ

និង ចដលមាន

ពហុ គណ្ ុ រួមរូចបំផរ ុ .រួចចករួមធំបផ ំ រ ុ |លឹម សុ វណ្ណវិចត្ិ រ

ចបប។ ដូហចនោះហយើ ងមាន ។


135. យ ើ ងមាន

ជាចំ នន ួ គត់ យបើ

ចចកដាច់ ។ យដា

បឋមនឹងគ្នាព ីរៗ យ ោះ ចចកដាច់

ដូចយនោះ ចចកដាច់

ចចកដាច់

។ យដា

បឋមនឹង

ចចកដាច់

ដូយចាោះ ចចកដាច់

ដាច់

និង

សនមតថា

ធំ ជាងយគ។ ដូយចាោះ យ ើ ងទាញបាន ឬក៏

ករណី

ចចកដាច់

ដូយចាោះ

(យទ្រោះ

)។ យដា

បឋមនឹង

។ លកខខណឌយៅជា

ជាចំ នន ួ គត់ សមមូលនឹង បឋមនឹង

ជាចំ នន ួ គត់ ។

យ ោះកាង ុ ចំ យណាមចំ នន ួ ព ីរយនោះ ទ្តូវមានមួ ជាចំ នន ួ យសស។ យ ើ ងអាចសនមតថា

យសស។ ដូយចាោះ ចចកដាច់ អាចចតយពលណា ឬ

។ យដា

គ្នាព ីរៗ យ ោះ

យដា

ចចក

។ យដា អយថរទាំងអស់ យនោះមានលកខណៈសី យមទ្ទី យ ោះយ ើ ងអាច

អាចមានចតយពលណា

ករណី

។ ដូចគ្នាយ ើ ងទាញបាន

សមមូលនឹង ចចកដាច់

បយណាណោះ (យទ្រោះ បឋមនឹង

)។ រួចយ ើ

ចចក

។ ករណីយនោះ

ដាច់ ។ ដូយចាោះ

។ យបើ

យ ើ ងទាញបាន

យបើ

យ ើ ងទាញបាន

ដូយចាោះជាសរបចយមលើ មាន

និង ចំ លាស់ ទាំងអស់ ចដលអាច

មានននចយមលើ យនោះ ។

លឹម សុ វណ្ណវិចត្ិ រ| ពហុ គណ្ ុ រួមរូចបំផរ ុ . រួចចករួមធំបផ ំ រ ុ

149


136.  ហយើ ងព ិនិតយផលគុណ

ោមួយ និងចំ នន ួ បឋម

ោមួយ។ សនមតថា

។ ហោយ

និងហោយ

ហ ោះ

ដូហចនោះមានន័យថា សវ័ យគុណ ពី

ហទ មានន័យថា

ននចំ នន ួ បឋម

ោមួយចដល

។ ចតកនង ុ ករណី

ហ ោះ

មានន័យថា កនង ុ ចំ ហោមបោ ា ផលគុណរបស់ មួយចដលហផទៀងផ្ទទត់

, មានតនមល ធំមិនហលើស

ទំងអស់ មានយ៉ាងហោចោស់ ផលគុណ

។ ដូហចនោះ ពហុ គុណរួមតូចបំ ផត ុ ននបោ ា ផលគុណកនង ុ សំ នួរ

កំណត់ ហោយ

ត្តមរូបមនាឌ ីប៉ា ូលញ ី ៉ាក់ សវ័ យគុណធំ បំផត ុ

នន

បឋម

ចដល

ចចកោច់

បឋម

ដូហចនោះ

137.  ហយើ ងមាន

បឋម

គហនោះ មិនអាចមានកត្តារួមធំ ជាង

បឋម

ដូហចនោះ ភាគយកនិងភាគចបងរបស់ របភា

ហទ។ មានន័យថា វាមិនអាចសរមួលបានហឡើយ។

138. ត្តង

ចដល 150

កំណត់ ហោយ

ជាចំ នន ួ បឋម។ ហយើ ងមាន ពហុ គណ្ ុ រួមរូចបំផរ ុ .រួចចករួមធំបផ ំ រ ុ |លឹម សុ វណ្ណវិចត្ិ រ


ហយើ ងសនមតថា

។ សមភាពខាងហលើហៅជា

ព ិត។ 139. ត្តមរទឹសាីបទរគឹោះននចំ នន ួ នព វនា ហយើ ងទញបាន

រតូវចតមានរាង

។ ជាងហនោះហៅហទៀត រតូវមានយ៉ាងហោចោស់ ធាតុព ីរ កនង ុ ចំ ហោមធាតុទង ំ បួន ចដល

រតូវហសមើ នង ឹ

រតូវហសមើ នង ឹ

ហហើ យនិង រតូវមានយ៉ាងហោចោស់ ធាតុព ីរកនង ុ ចំ ហោមធាតុទង ំ បួន ចដល

ហយើ ងព ិនិតយករណី

សិ ន។ ហយើ ងមាន

ករណី (1)

ទំងបួនមាន

ករណី (2) មានបីកង នុ ចំ ហោម ករណីហនោះ ហយើ ងមាន , ហយើ ងមាន , ហយើ ងមាន

ដូចគ្នន (ហយើ ងមាន មួយករណី)។ មាន

, ហយើ ងមាន ហយើ ងមាន

រហបៀប ហរ ោះកនង ុ ោរហរជ្ើសហរ ើសចំ នន ួ ទីបន ួ នីមួយៗចដល រហបៀប, ហហើ យកនង ុ ោរហរជ្ើសហរ ើស យកបីកង នុ ចំ ហោមបួន ចដល រហបៀប។ ដូហចនោះសរុបមាន

ករណី (3) មានព ីរកនង ុ ចំ ហោម ករណីហនោះ ហយើ ងមាន

ហហើ យមួយហទៀត មាន

មាន

រហបៀប។ ហហើ យព ីរហទៀត មាន

រហបៀប ហរ ោះ កនង ុ ោរហរជ្ើសហរ ើសចំ នន ួ ទីបី និង បួននីមួយៗចដល រហបៀប ហហើ យកនង ុ ោរហរជ្ើសហរ ើស យកព ីរកនង ុ ចំ ហោមបួន ចដល រហបៀប។ ដូហចនោះសរុបមាន

សរុបករណីទំងបីបញ្ចូ លគ្នន ហយើ ងមានទំងអស់

រហបៀប។ ចបបកនង ុ ោរហរជ្ើសហរ ើសឱ្យបានយ៉ាង

ហោចោស់ ព ីរកនង ុ ចំ ហោមធាតុបន ួ ហដើមបីឱ្យបានសវ័ យគុណ ។ ដូចគ្ននចដរ មានទំងអស់

លឹម សុ វណ្ណវិចត្ិ រ| ពហុ គណ្ ុ រួមរូចបំផរ ុ . រួចចករួមធំបផ ំ រ ុ

151


ចបបកនង ុ ោរហរជ្ើសហរ ើសឱ្យបានយ៉ាងហោចោស់ ព ីរកនង ុ ចំ ហោមបួន ហដើមបីឱ្យបានសវ័ យគុណ ។ ដូហចនោះជាសរុបហគមានចតុធាតុទង ំ អស់ ចំ នន ួ

បំចបកជាកត្តតបឋម 140. ហយើ ងមាន

។ ដូហចនោះ

ជាតួចចករបស់

បាន ចដល និង

និង

ហរៀងគ្នន ចំ នន ួ

និង

ហផេងគ្នន ។ ដូហចនោះ

។ ហបើ ចចកទំងព ីរហសមើ ជា

លុ ោះរត្តចត

ហ ោះ

។ ដូហចនោះ ហយើ ងមាន

មានតួចចកវ ិជ្ជមានចំ នន ួ

ក៏ជាតួចចករបស់

។ ដូហចនោះ ហយើ ងអាចចចកតួចចកទំង

គូតចួ ចក ចដលមានរាង

អាចសរហសរជារាង

របស់

ចដរ ហហើ យផលគុណននតួ ហលើកចលងចត

ហហើ យផលគុណននតួចចកព ីរហនោះ ហសមើ

ហចញ

។ ដូហចនោះចហមលើយគឺ

។ 141. តួចចកគូរបស់

មានរាង

ចដល

។ ផលបូកតួចចកគូវ ិជ្ជមានរបស់

142. ហយើ ងមាន

និង

។ ចំ ហ ោះ

មានរាង

143.  ត្តង 1) បង្ហាញថា ហបើ

152

ជា

យ៉ាង ដូហចនោះ

។ ដូហចនោះហយើ ងមានជ្ហរមើស

ចបប កនង ុ ចំ ហោមតួចចកចំ នន ួ

។ ដូហចនោះរបូបាបីលហី តចដលចង់ បានគឺ

នន

និង

។ កនង ុ ចំ ហោមចំ នន ួ ទំងហនោះ បោ ា ពហុ គុណ

រតូវហផទៀងផ្ទទត់ លកខខណឌ ចបប ដូហចនោះ មាន

ចដល

នីមួយៗ ហយើ ងមានជ្ហរមើសចំ នន ួ

មានតួចចកគត់ វ ិជ្ជមានចំ នន ួ នន

និង

ហសមើ នង ឹ

។ ដូហចនោះតួចចករបស់

ចំ នន ួ គត់ ចដល

ជាចំ នន ួ គត់ ចដល

ជាផលបូកតួចចកវ ិជ្ជមានទំងអស់ របស់ ហ ោះ

ពហុ គណ្ ុ រួមរូចបំផរ ុ .រួចចករួមធំបផ ំ រ ុ |លឹម សុ វណ្ណវិចត្ិ រ

។ ។

នន

ចំ នន ួ

ចដលជាពហុ គុណ


សនមតថា

ជាចំ នន ួ បឋម ហ ោះ

។ ហោយ

1=1 ហ ោះ

ដូហចនោះ ផលបូកតួចចករបស់ ដូហចនោះ

ហរៅព ីខលន ួ វា ហសមើ នង ឹ

ជាចំ នន ួ សមបុណណ ហលខ។

2) បង្ហាញថា ហបើ រាសមកវ ិញ ត្តង ហោយ

ហ ោះ

ជាចំ នន ួ គត់ គ។ ូ ត្តង

ហសស ហ ោះ

ជាសមបុណណ ហលខ ហ ោះ

។ ដូហចនោះ ចំ ហ ោះចំ នន ួ គត់ ធមមជាតិ

ោមួយ។ ដូហចនោះ

។ ហយើ ងចង់ បង្ហាញថា ហបើ

។ ហយើ ងសហងកតហឃើ ញថា

ហ ោះ មានយ៉ាងតិចតួចចករបស់

ចំ នន ួ ៣ ត្តងហោយ

ផទយ ុ ព ីោរព ិតចដល ហយើ ងទញបាន ម។ ហបើ

ដូហចនោះ

។ ដូហចនោះ និង

មិនចមនជាចំ នន ួ បឋម ហ ោះ

។ ដូហចនោះ

រតូវចតជាចំ នន ួ បឋ

។ ដូហចនោះ

មិនអាចជាចំ នន ួ បឋមហទ។ ដូហចនោះ ជាចំ នន ួ បឋម ហហើ យ

ហហើ យ ឱ្ ំ យ

រតូវចតជាចំ នន ួ បឋម

និង

លឹម សុ វណ្ណវិចត្ិ រ| បំចបកជាកត្តតបឋម

153


154

ពហុ គណ្ ុ រួមរូចបំផរ ុ .រួចចករួមធំបផ ំ រ ុ |លឹម សុ វណ្ណវិចត្ិ រ


ផ្នែកគត់ 144. យ ើ ងដឹងថា

ចំ យ ោះគ្គប់ ចំ នន ួ គត់

។ យគ្ ោះ

ព ិត។ បន្ទាប់ មកយទៀតយ ើ ងនឹងបង្ហាញថា គ្មានចំ នន ួ គត់

ដដល

យទ។

យលើកវ ិសមភាពជាកាយេ យ ើ ងដឹងថា

ដូយចនោះ

ចំ នន ួ គត់ ដូយចនោះ យបើ

យន្ទោះ

។ ដតយ ើ ងដឹងថា

។ យោ សាេ

មិនអាចជាកាយេននចំ នន ួ គត់ បានយទ ។ យគ្ ោះ

។ ដតយបើ មានន័ ថា

ជា

មិនអាចយសាើ

យន្ទោះ

។ ដត យទ។ ដូយចនោះ

មិនអាចជាចំ នន ួ គត់

បានយទ។ ដូយចនោះ 145. អនុគមន៍

ព ិត។ ជាអនុគមន៍ចុោះ។ ដូយចនោះ ចំ យ ោះចំ នន ួ គត់ វ ិជ្ជមាន

និង

លឹម សុ វណ្ណវិចត្ិ រ| ផ្នែកគរ់

155


យ ើ ងមាន

ដូយចនោះ

146. យ ើ ងមាន

ជាចំ នន ួ គត់ គ។ ូ យោ

យន្ទោះ

- កេណី

ជាចំ នន ួ យសស យន្ទោះ

ជាចំ នន ួ គត់ គូ - កេណី

ជាចំ នន ួ គូ យន្ទោះ

ជាចំ នន ួ យសស។

156

ផ្នែកគរ់ |លឹម សុវណ្ណវិចត្ិ រ


ដូយចនោះមានន័ ថា ចំ យ ោះ

យន្ទោះ

មានតនមៃ យសស,គូ,យសស,…ឆ្លៃស់ គ្មនេហូ

ត។ 147. វ ិសមភាពសមមូលនឹង

យដើមបីឱ្យមាន

ដតមួ គត់ ទាល់ ដត

បាន

ដូយចនោះ មានដត

។ ចំ យ ោះ មួ គត់ ។

យន្ទោះ

148. យោ ។ ដូយចនោះ

យ ើ ងទាញ

។ យោ

យន្ទោះ

។ ដូយចនោះ ។ ដូយចនោះ

ដដល មានឫសវ ិជ្ជមានដតមួ គត់ គ ឺ និង

។ ដូយចនោះ ចំ យ ោះតនមៃ

យនោះ យ ើ ងមាន

។ យ ើ ងទាញបាន

149. យ ើ ងមាន

ដូយចនោះ

កណ្ត ា លនន ដូយចនោះ

និង

គឺ

។ ដូយចនោះ

យៅជ្ិត

ជាង។

150. តាង

ដូយចនោះយ ើ ងនឹងគណន្ទតនមៃេបស់

ដតកនង ុ កេណី

យ ើ ងមាន លឹម សុ វណ្ណវិចត្ិ រ| ផ្នែកគរ់

157


យបើ

យន្ទោះ

ដូយចនោះ

យបើ

យន្ទោះ

ដូយចនោះ

យបើ

យន្ទោះ

ដូយចនោះ

យបើ

យន្ទោះ

ដូយចនោះ

ដូយចនោះ 151. យ ើ ងយឃើ ញថា យៅកនង ុ ផលបូកខាងយលើ ចំ នន ួ តួដដលខុសព ី មានចំ នន ួ កំណត់ គឺថា យសាើ សូនយយពលដដល

សំ យណើ៖ ចំ យ ោះគ្គប់ ចំ នន ួ ព ិត

យគមាន

សគ្មា បញ្ជជក់ ៖ យបើ

យន្ទោះ យន្ទោះ

និង និង

។ យបើ ។

ដូយចនោះ

152. យបើ

ជាចំ នន ួ គត់ យន្ទោះ យ ើ ងយឃើ ញថា សមភាពព ិត។ យ ើ ងសនាតថា

មានន័ ថា

តាម (*) យ ើ ងទាញបាន

158

ផ្នែកគរ់ |លឹម សុវណ្ណវិចត្ិ រ

ដូយចនោះ

មិនដមនជាចំ នន ួ គត់


ដូយចនោះ

តាម (**) ។ ។ យ ើ ងយឃើ ញថា យបើ

153. តាង

។ យ ើ ងទាញបានថា យបើចំនន ួ គត់ ដដល

ជាចំ នន ួ គត់ យន្ទោះចំ យ ោះ

មួ អាចសេយសេជារាង

យគអាចសេយសេ

ដូចគ្មន មានន័ ថា

ជាចំ នន ួ គត់ វ ិជ្ជមាន យន្ទោះ បាន

ជារាងដដលចង់ បាន បាន

។ យោ យហតុយនោះយហើ

គ្គ្មន់ ដតកំណត់ ថាយតើកង នុ ចំ យណ្តម២០ចំ នន ួ គត់ ដំបូងមួ ណ្តដដលអាចជាតនមៃេបស់

ជាដំ បូងយ ើ ង ចំ យ ោះ

។ យ ើ ងសយងេតយឃើ ញថា យពល

យកើន តនមៃ េបស់

មានតនមៃ គត់ បុយណ្តណោះ យហើ កំយណើន

បាូេជាតនមៃថដតយពលណ្តដដល ាី

យពលយន្ទោះតនមៃេបស់

យកើនធំ ជាងមុន។ យពល

ដបបយនោះ យកើតមានដតយពលណ្តដដល

មានរាង

ឬ ដគ្បគ្បួលចយន្ទៃោះ

ដដល

និង

ឬ ។ គ្បភាគដបបយនោះមានចំ នន ួ ១២យងគឺ

គ្តូវនឹង

មានតនមៃ

ដូយចនោះកនង ុ ចំ យន្ទម ២០ចំ នន ួ គត់ វ ិជ្ជមានដំ បូងមានដតចំ នន ួ ១២បុយណ្តណោះដដលអាចសេយសេតាមរាងចង់ បា ន។ យោ

ដូយចនោះមាន

ចំ នន ួ គត់ វ ិជ្ជមានដដលអាចសេយសេជារាងចង់

បាន។

លឹម សុ វណ្ណវិចត្ិ រ| ផ្នែកគរ់

159


154. ផលបូកដដលឱ្យមាន

តួ ដដលតួនម ី ួ ៗ យសាើ នង ឹ

ដូយចនោះមានន័ ថា តាង

ជាចំ នន ួ តួដដលយសាើ

យោ យសាើ

។ តួទ ី

មាន

។ ដូយចនោះ

គឺ

និង តួទ ី

155. ចំ យ ោះ

។ ដូយចនោះ

តួដំបូងយសាើ

គឺ

និង

តួបន្ទាប់

។ យ ើ ងទាញបាន

តាង

យោ

យន្ទោះ

ចំ យ ោះចំ នន ួ គត់

ដដល

យន្ទោះ

។ ដូយចនោះ

ចំ យ ោះចំ នន ួ គត់

ដដល

យន្ទោះ

យោ

មានតនមៃ ខុសៗគ្មន។ យោ

។ យោ ដឹងថា ស្ុី តយនោះមិនដមនជាស្ុី តថ

យន្ទោះមានន័ ថា

។ដូយចនោះគ្គប់ ចំ នន ួ គត់ វ ិជ្ជមានទាំងអស់ ដដលមានតនមៃតច ូ ជាង យ ើ ងមាន

និង

)។

156. យ ើ ងយឃើ ញថា គ្មានតួណ្តមួ កនង ុ ចំ យណ្តម អុី ចង ឹ យទ យ ើ ងនឹងមាន ផ្នែកគរ់ |លឹម សុវណ្ណវិចត្ិ រ

ជាតួននស្ុី តយនោះ។

។ ដូយចនោះចំ នន ួ តួដដលមានតនមៃខុសគ្មនគឺ

(តនមៃទាង ំ យន្ទោះគឺ

160

។ ដត

ដូយចនោះចំ នន ួ តួដដលយសាើ

ដូយចនោះ

េ ឺក៏

។ ដូយចនោះ

ដដលដចកោច់ តួមួ យទៀតយទ យគ្ ោះយបើមិន ដដល

ជាចំ នន ួ យសស។ យ ើ ងយឃើ ញ


ថា

ខុសគ្មនទាំងអស់ ។ យោ យ ើ ងមាន

គត់ យសសវ ិជ្ជមាន ដដលតូចជាង ឥលូ វយ ើ ង យ ើ ងព ិនិតយ ។ ដូយចនោះ

ជាចំ នន ួ យសសដដល

ទាំងអស់ ចំ នន ួ

ជាសំ ណំ ុ ចំ នន ួ

។ ។ យបើ

យន្ទោះ

ដូយចនោះ

។ ដូយចនោះ ដូយចនោះវាផាុ ព ីសមាតក ិ មា ដដល

ដូយចនោះ សំ ណំ ុ េបស់

និង

ចំ យ ោះតនមៃ ណ្តមួ

និង

ឬក៏

លឹម សុ វណ្ណវិចត្ិ រ| ផ្នែកគរ់

161


162

ផ្នែកគរ់ |លឹម សុវណ្ណវិចត្ិ រ


សមី ការដ្យូផង់ ់។ មយើ ងមឃើ ញថា សមី 157. សមីការដដលចមមលើយរបស់ វាជាចំ នន ួ គត់ មៅថាសមីការដ្យផង ូ ការដយូផង់ មានចមមលើយជាចំ នន ួ គត់ ដតក្នង ុ ក្រណី

បុម្ណោះ។ ប្បមាណវ ិធីរបស់ អឺ គដ ល ី ជា

មមធោបាយដ៏មានប្បសិ ទភា ធិ ពមួយ ក្នង ុ ការដសែ ងរក្ចមមលើយរបស់ សមីការមនោះ។ មយើ ងមាន

ដូមចនោះចមមលើយងាយរបស់ សមីការមនោះ គឺ

។ មយើ ងមឃើ ញថា

ក្៏ជាចមមលើយរបស់ សមីការមនោះដដរ។ សមីការមនោះគ្មានចមមលើយមផេងព ីមនោះមទ (សូ មអាន ប្ទឹសតីបទក្នង ុ មមមរៀនទី៣)។ 158. មយើ ងមាន គុណអងគសងា​ាងននសមីការនឹង ដូមចនោះ 159. មោយ

មយើ ងទាញបាន ។ មានន័យថា អងគខាងមឆែងដចក្ោច់ នឹង

ដតអងគខាងស្តំដចក្

មិនោច់ នឹង២ ដូមចនោះសមីការគ្មានចមមលើយជាចំ នន ួ គត់ ។

លឹម សុ វណ្ណវិចត្ិ រ| សមីការដ្យូផង់

163


160. មយើ ងមាន

។ មយើ ងទាញបាន

។ 161. សនាតផទយ ុ មៅវ ិញថា សមីការមានឫសជាចំ នន ួ គត់ វ ិជជមាន។ ដូមចនោះ

ក្រណីទី១ មយើ ងទាញបាន

មិនយក្មប្រោះ

ក្រណីទី២ មយើ ងទាញបាន

មិនយក្មប្រោះ

ប្តូវដតជាចំ នន ួ គត់ វ ិជជមាន។ ប្តូវដតជាចំ នន ួ គត់ វ ិជជមាន

162. សនាតថា សមីការមានចមមលើយវ ិជជមាន ខុសព ីសូ នយ មផេងមទៀត ម ើ យមយើ ងតាង

ជា

ចមមលើយដដលតូចជាងមគ។ ប្តូវដតជាចំ នន ួ គូ ប្តូវដតជាចំ នន ួ គូ ប្តូវដតជាចំ នន ួ គូ មយើ ងមឃើ ញថា

ក្៏ជាចមមលើយរបស់ សមីការមនោះដដរ ដដល

ដូមចនោះ ផទយ ុ ព ីការសនាត ។ ដូមចនោះ សមីការមិនមានចមមលើយ មប្ៅព ី

មទ។

163.164. ។ មយើ ងព ិនិតយក្រណី

165. មបើ ។

ដូមចនោះគ្មានចំ នន ួ គត់

ដដលកាមររបស់ វាសមមូលនឹង

166. ចំ នន ួ គូបសមមូលនឹង សមមូលនឹង

មទ។ ដូមចនោះសមីការគ្មានចមមលើយ។

។ ចំ នន ួ សែ័ យគុណនន២ (គឺចំនន ួ ដដលមានរាង ។ ដូមចនោះ

មទ។ 164

តាម

សមីការដ្យូផង់ |លឹម សុ វណ្ណវិចត្ិ រ

)

។ ដូមចនោះអងគទាង ំ ២មិនអាចមសាើ គ្មន


167. ប្គប់ ចំ នន ួ សែ័ យគុណបួនសុ ទដធ តសមមូលនឹង យងមប្ចើនសមមូលនឹង

។ មានន័យថា

។ ដត

។ ដូមចនោះ សមីការ

មិនអាចមានចមមលើយមទ។ 168.  សមីការសមមូលនឹង ដូមចនោះ

និង

ដដលខុសគ្មនចំ នន ួ

ប្តូវដតជាសែ័ យគុណនន ឯក្តា (

ជាចមមលើយដតមួយគត់ ម ើ យ

ទាំងព ីរ។ មយើ ងដឹងថាចំ នន ួ សែ័ យគុណនន

) មានដត

និង

បុម្ណោះ។ ដូមចនោះ

169. សមីការមនោះអាចសរមសរមៅជា

ជាមួយគ្មនមនោះដដរ មយើ ងមាន ដូមចនោះ មបើ

ម ោះ មយើ ងទាញបាន

សមីការគ្មាន

រ ឺស។ ដូមចនោះទាល់ ដត ក្រណី

មានន័យថា

។ ក្រណី

មយើ ងទាញបាន

មិនដមនជាចំ នន ួ គត់ ។

170. មោយ

មានលក្ាណៈសុី មមប្ទីនង ឹ គ្មន ម ោះមយើ ងអាចសនាតថា

សិ ន ។

តាមលក្ាខណឌមនោះ មយើ ងមាន

មបើ

ម ោះ

មិនអាចមប្រោះ

មបើ

ម ោះ

។ មោយ

។ ម ោះ

លឹម សុ វណ្ណវិចត្ិ រ| សមីការដ្យូផង់

165


មោយ

ដូមចនោះ ។ មបើ

មបើ

។ ដូមចនោះ ម ោះ

មិនអាចមាន

។ មបើ

ម ោះ

ម ោះ

ដូមចនោះ

ដូមចនោះជាសរុប ចមមលើយរបស់ សមីការមាន

និងប្គប់ តំមរៀបទាំងអស់ ដដល

អាចមានននប្តីធាតុទាង ំ អស់ មនោះ (

)។

171. តាង ។ មបើ

ម ោះ

ជំនស ួ តនមល

ទាំងមនោះចូល មយើ ងទាញបាន

គ្មានចមមលើយជាចំ នន ួ គត់ ទាង ំ ព ីរ។ ដូមចនោះ មយើ ងមាន

សមីការគ្មានចមមលើយ។

ដូមចនោះ

ជាចមមលើយ ទាល់ ដត

ដតសមីការគ្មានចមមលើយចំ មរោះ

ជាចមមលើយដដរ។

មទ។ ដូមចនោះ ប្ ត ចមមលើយ

ប្តូវដត

។ មយើ ងមាន

មិនដមនជាចំ នន ួ គូប។

ដូមចនោះ មយើ ងទាញបានចមមលើយ

និង

មទៀតគឺ

និង

172. មយើ ងមាន មាន

166

និង ។ ម ើ យគូចមមលើយមផេង ។

ចំ មរោះប្គប់ ចំ មរោះប្គប់

សមីការដ្យូផង់ |លឹម សុ វណ្ណវិចត្ិ រ

។ ដូមចនោះសមីការមនោះគ្មានឫស។

។ ដតមយើ ង


ប្តូវដតជាចំ នន ួ សែ័ យគុណនន

173. ក្) ដូមចនោះ

ជាចំ នន ួ គូ។ ដូមចនោះ

ខ)

ជាចំ នន ួ គូ ម ើ យ ។ មោយ

ដតជាសែ័ យគុណនន

ដូមចនោះ

។ សមីការមៅជា

បឋមនឹង

មបើ

ទាំងព ីរ។ ដតមិនដដលមានសែ័ យគុណនន

ម ោះ

និង

ប្តូវ

ដដលខុសគ្មនចំ នន ួ ឯក្តាមទ។ ដូមចនោះ

សមីការគ្មានចមមលើយ។ គ) មោយ សមមូលនឹង

។ ដតមយើ ងដឹងថាចំ នន ួ កាមរ ឬ

។ ដូមចនោះសមីការគ្មានចមមលើយ។

174. សនាតថាសមីការមានចមមលើយ។ សនាតថា

ជាចមមលើយវ ិជជមានឬសូ នយ ដដលតូចជាងមគ

ក្នង ុ ចំ ម្មចមមលើយដដលអាចមានននសមីការ។ ជាព ុ គុណនន

ដូមចនោះ

។ តាង

ដូមចនោះ

។ មយើ ងទាញបាន

ដដល

ក្៏ជាឫសរបស់ សមីការ

ដតមយើ ងបានសនាតថា

ដដរ។

ជាចមមលើយដដលតូចជាងមគ ដូមចនោះ មានដត

175. សមីការមានឫសងាយ ភាគដបងរបស់

ជាព ុ គុណនន

មសាើ នង ឹ

និង

។ មបើមយើ ងគុណ

ជាចំ នន ួ គត់ ម ោះមយើ ងទទួលបានដផនក្ទសភាគរបស់

នឹង

ដដល

មៅរក្ាតនមល មដើមដដដល។ មប្រោះ

គត់

ដូមចនោះ ប្ ត ចំ នន ួ

លឹម សុ វណ្ណវិចត្ិ រ| សមីការដ្យូផង់

167


សុ ទដធ តជាចមមលើយរបស់ សមីការ។ ដូមចនោះសមីការមានឫសមប្ចើនរាប់ មិនអស់ ។ 176. មយើ ងមាន និង

ដូមចនោះ

និង និង

។ មោយ

។ ដូមចនោះ

ម ោះសមីការ

ដដល

។ ដូមចនោះ ជាចំ នន ួ គត់ ដដលដចក្មិនោច់ នឹង

មិនអាចមានចមមលើយមទ។ ដូមចនោះសមីការគ្មានចមមលើយ។

177. សមីការអាចសរមសរមៅជា

មោយសិ ក្ាមៅមលើអនុគមន៍ ម ើ យចុោះមលើ

ដូមចនោះ មយើ ងទាញបានថា មបើ វាប្តូវដត និង

មយើ ងទាញបានថា អនុគមន៍មនោះជាអនុគមន៍មក្ើនមលើ

និង

និង

ជាចំ នន ួ គត់ ព ីរខុសគ្មន(សនាតថា

។ ចម ល ោះ

មៅ

មយើ ងមានចំ នន ួ គត់ ដត

។ សរុបសមីការមានចមមលើយ

178.  តាង

ចំ មរោះ 168

។ ។ មយើ ងមាន

មយើ ងមាន សមីការដ្យូផង់ |លឹម សុ វណ្ណវិចត្ិ រ

) ដដល មួយគត់ ។ ដូមចនោះ

ម ោះ


ដូមចនោះមានន័យថា ចំ មរោះ ដដល

មគមាន

ដចក្ោច់ នឹង

ជាចំ នន ួ គត់ ដចក្មិនោច់ នឹង

។ មយើ ងមាន ដូមចនោះ

។ មោយ

។ ចំ មរោះ

មិនអាចមសាើ

ដតដចក្មិនោច់ នឹង ម ោះ ប្តូវដត

មប្រោះ

មយើ ងមាន

បានមទ។ មយើ ងទាញបាន ថា សមីការគ្មានចមមលើយមទ មពល

មយើ ងព ិនិតយក្រណីនីមួយៗខាងមប្កាម ចមមលើយសមីការ : គ្មានចមមលើយ ចមមលើយសមីការ មិនអាចជាចំ នន ួ សែ័ យគុណមទ មិនអាចជាចំ នន ួ សែ័ យគុណមទ មិនអាចជាចំ នន ួ សែ័ យគុណមទ មិនអាចជាចំ នន ួ សែ័ យគុណមទ ជាចមមលើយរបស់ សមីការ មបើសិនជាមាន។ មយើ ងមាន

179. តាង មបើ

ជាចំ នន ួ មសស ម ោះ

និង

ក្រណីមនោះមិនអាចមាន។ ដូមចនោះ

ដូមចនោះ

បឋមនឹងគ្មន ម ើ យជាចំ នន ួ គូបព ីរដដលខុសគ្មន ជាចំ នន ួ គូ

ឯក្តា។

ជាចំ នន ួ គូដដរ។ សមីការមៅជា

ជាចំ នន ួ គូ។ ដូមចនោះមយើ ងទាញបាន ។ ដូមចនោះ

ដូមចនោះ មោយ

និង និង

ប្តូវដតជាចំ នន ួ គូ

ឱ្ ំ យ

ជាចំ នន ួ មសស។ ដូមចនោះ

បឋមនឹងគ្មន ម ោះ វាប្តូវដតគូបទាំងព ីរ ដូមចនោះ

ម ើយ

លឹម សុ វណ្ណវិចត្ិ រ| សមីការដ្យូផង់

។ 169


ដូមចនោះ សមីការមានចមមលើយព ីរគត់ គឺ

ម ោះ មយើ ងទាញបាន

180. មបើ

មទ ម ោះមានមួយក្នង ុ ចំ ម្ម ខាត) ដូមចនោះ ធំ ជាងឬមសាើ មបើសិនជាមួយម ោះជា

និង

។ សនាតថា ដដលប្តូវដតធំ ជាង

។ មបើមគមិនមាន

ោច់ ខាត(និងមួយមទៀតតូចជាងោច់

។ មយើ ងទាញបាន

ផទយ ុ ព ីសមាតក្ ិ មា។ មបើសិនជាមួយម ោះជា

ម ោះមយើ ងទាញបាន

ក្៏ផយ ទុ ព ីសមាតក្ ិ មាដដរ។ 181. មយើ ងដឹងថា

ជាចមមលើយរបស់ សមីការជានិច។ ច មោងវ ិញមទៀត មបើ

ចមមលើយមួយននសមីការ

(*)ម ោះ

ម ើ យ ឫសមួយមទៀតរបស់ ព ុ ធាមនោះ គឺ សមីការ(*)មានឫស

ជាឫសរបស់ ព ុ ធា

(មប្រោះ

- មបើ

ម ើ យមបើ

។ និង

មយើ ងមាន

ជាចមមលើយ ម ោះ មយើ ងអាចរក្បាន ចមមលើយមផេងមួយមទៀត ក្ំណត់ មោយ ដដល ម ោះ

បាន ចមមលើយមផេងមួយមទៀត ក្ំណត់ មោយ

170

។ ជាសរុបមយើ ងទាញ

ម ោះ

មានន័យថា មបើ - មបើ

)។ ដូមចនោះមយើ ងទាញបាន

។ ដូចគ្មន សមីការ(*)មានរ ឺស

បានឫសរបស់ សមីការ(*) មាន មោងវ ិញមទៀត មបើ

ជា

សមីការដ្យូផង់ |លឹម សុ វណ្ណវិចត្ិ រ

។ មានន័យថា មបើ ដដល

ជាចមមលើយ ម ោះ មយើ ងអាចរក្ ។


តាង

និង

តននចមមលើយ

។ តាមលក្ាណៈ ដូចដដលមយើ ងបានមរៀបរាប់ ព ីខាងមដើម មយើ ងទាញរក្សែុី ដដល មោយ មោយ មោយ

..................... លក្ាខណឌចុងមប្កាយមនោះ ធា ថា ចមមលើយទាំងអស់ របស់ សមីការខុសគ្មនព ីរៗ។ ក្នង ុ ក្រណីមនោះសមីការ មានឫសមប្ចើនរាប់ មិនអស់ ។ មបើ

សមីការសរមសរមៅជា

ដូចគ្មនក្រណី មបើ

មានចមមលើយមប្ចើនរាប់ មិនអស់ ។

ម ោះសមីការ មៅជា និង

។ ប្ ត ចំ នន ួ

ប្តូវដតជាចំ នន ួ វ ិជជមាន

មិនអស់ មទ។ ដូចគ្មន មពល ជាចុងមប្កាយ មបើ

។ ដូមចនោះ សមីការមានឫសគត់

រាប់

ម ោះសមីការមៅជា

ដដលឫសរបស់ សមីការមនោះមានដត

មានចមមលើយមានចំ នន ួ ក្ំណត់ ។

ដូមចនោះ មដើមបីឱ្យសមីការមានចមមលើយមប្ចើនរាប់ មិនអស់ សំ ណំ ុ ចំ នន ួ គត់ , 182. មយើ ងមឃើ ញថា សមីការមានឫស

ប្តូវដត

។សនាតថា សមីការមានចមមលើយមផេងមទៀត។ តាង

ជាចមមលើយតូចបំ ផត ុ របស់ សមីការដដលខុសព ីសូ នយ។ - មបើ

ដចក្ោច់

។ ដូមចនោះ

ដចក្ោច់ តូចជាង

ម ោះ

ប្តូវដតដចក្ោច់

។ តាង

និង

ទាំងព ីរ ដូមចនោះ ។ ដូមចនោះ

ដចក្ោច់ នឹង ។ ដូមចនោះ

ក្៏ជាចមមលើយសមីការដដរ។ ក្រណីមនោះផទយ ុ ព ីការសនាតដដល

ជា

ចមមលើយតូចបំ ផត ុ ។ - ដូមចនោះ

និង

ប្តូវដតបឋមនឹង

។ មយើ ងមាន លឹម សុ វណ្ណវិចត្ិ រ| សមីការដ្យូផង់

171


តាមប្ទឹសតីបទ Bachet-Bézout មយើ ងអាចរក្បាន ។ ដូមចនោះ តាម

ដដល

។ ដូមចនោះ

។ ដតចំ នន ួ កាមរ មិនអាចសមមូលនឹង

មទ។ ដូមចនោះសមីការមានចមមលើយដតមួយគត់ គ ឺ

183. ក្) ដំ បូងមយើ ងសនាតថា ។

និង

មិនអាចមសាើ

។ មយើ ងទាញបាន

មទ។ មបើ

សមីការមៅជា

ដូមចនោះ ចមមលើយរបស់ សមីការ គឺ

ខ) តាមរមបៀបដូចខាងមលើ មយើ ងទាញបានក្រណី មយើ ងយក្

និង

សមីការគ្មានឫស។ ចំ មរោះ

មយើ ងនឹងបងា​ាញថា

មិនមផទៀងផ្ទទត់ មទ។ មប្រោះមបើ ម ោះ មយើ ងទាញបាន រួចម ើ យ

មោយដឹងថា

ដូមចនោះ

មផទៀងផ្ទទត់ មោយ ដូមចនោះ

។ ដូចគ្មន មយើ ងទាញបាន

មិនអាច។

ម ោះ

ជាចមមលើយមួយរបស់ សមីការក្រណី

មយើ ងមាន

ដូមចនោះ

។ ជាចមមលើយមួយរបស់ សមីការក្រណី

។ មយើ ងដឹងថា មបើ

មផទៀងផ្ទទត់ សមីការ ម ោះ

ផ្ទទត់ សមីការដដរ មានន័យថា មបើ

មផទៀងផ្ទទត់ លក្ាខណឌសមីការមានឫស ម ោះ

រ។ ដូមចនោះ ប្គប់ ចំ នន ួ គត់ វ ិជជមាន 184. សនាតថា

172

ខុសព ី

សុ ទដធ តមផទៀងផ្ទទត់ លក្ាខណឌទាំងអស់ ។

ជាសំ ណំ ុ ចមមលើយដដលតូចជាងមគ។

សមីការដ្យូផង់ |លឹម សុ វណ្ណវិចត្ិ រ

ក្៏មផទៀង ក្៏មផទៀងផ្ទទត់ ដដ


មោយគុណអងគទាង ំ ព ីរនឹង

មយើ ងទាញបាន

គណ សមមូលតាម

សមីការមនោះមៅជា

មោយចំ នន ួ កាមរ តាម

សមមូលនឹង

ម ោះ

ដូមចនោះមានដត ោច់ នឹង

។ ដូមចនោះ

ប្តូវដតដចក្

។ មយើ ងមាន

មោយអងគខាងមឆែងដចក្ោច់ នឹង

ដូមចនោះ

និង

ប្តូវដតដចក្ោច់

និង

ដូមចនោះប្តូវដត

។ ដូមចនោះ

។ មយើ ងមាន

ដចក្ោច់

។ មយើ ងទាញបាន

ជា

ចមមលើយមួយមទៀតរបស់ សមីការដដលតូចជាងមុន។ ក្រណីមនោះផទយ ុ ព ីការសនាត។ ដូមចនោះសមីការគ្មាន ចមមលើយមផេងព ីសូ នយមទ។ 185. មយើ ងមាន ដូមចនោះ

អាចសរមសរជាផលបូក្ននចំ នន ួ គូបបួន។ មយើ ងមាន

មប្រោះ

។ ដតថាប្គប់ ចំ នន ួ គូបសុ ទដធ តសមមូលនឹង

បានថា ផលបូក្ននចំ នន ួ គូបបីឬតិចជាងមនោះ មិនអាចសមមូលនឹង ដូមចនោះ

តាម តាម

។ ដូមចនោះមយើ ងទាញ

មទ។

។ លឹម សុ វណ្ណវិចត្ិ រ| សមីការដ្យូផង់

173


186. សមីការ មានរ ឺសងាយ

មប្រោះ

សនាតថាសមីការមានចមមលើយមផេងមទៀត

។ សមីការមានលក្ាណៈសុី មមប្ទី។ សនាតថា

។ មបើ

ម ោះ

មបើ

ម ោះ

ដូមចនោះ

។ ។ មយើ ងសរមសរសមីការ(*)ជាសមីការដឺមប្ក្ទី២មធៀបនឹង

ថា មបើ

ជាឫសមួយ ម ោះ

ជាឫសមួយមទៀត។ តាម (**) មយើ ងមាន

ដូមចនោះ ឧទា

រណ៍

។ មយើ ងទាញបាន

ជាចមមលើយមួយមទៀតដដលធំ ជាងមុន (គឺ )។

ជាឫសមួយរបស់ សមីការ ម ោះឫសមួយមទៀតក្ំណត់ មោយ ជាឫសរបស់ សមីការ(*)។ ប្តឹមជំហានមនោះ ឫសរបស់

សមីការ(*) មិនដមនមានដត

និង

ដតតាមព ិត ចមាលស់ របស់ ឫសមនោះក្៏ជា

ឫសរបស់ សមីការដដរ ដូចជា

ប ទ ប់ មក្មទៀត ចំ មរោះចមមលើយថាមនោះ ី គឺ

មយើ ងមរៀបឫសសមីការមនោះតាម

មានន័យថា

ក្៏ជាឫសរបស់ សមីការដដរ។ ចំ មរោះ

ទាញបានឫសថាមួ ី យមទៀតដដលធំ ជាងមុន គឺ បានឫសបដនថមរបស់ សមីការ (*) មាន មាន

។ ដូមចនោះមយើ ងទាញ និងចមាលស់ របស់ ឫសមនោះ ដដលក្នង ុ ម ោះ

ដដរ។ ដូមចនោះជាបនតប ទ ប់ មយើ ងទាញបានធាតុតច ូ ជាងមគមចោះដតមក្ើនមៅៗ។

ដូមចនោះជាចុងមប្កាយ មៅប ទ ប់ ព ីបុ ា នជំហានមប្កាយមក្មយើ ងនឹងទាញបានចមមលើយដដលមាន ដដលនីមួយៗធំ ជាង

ទាំងអស់ ។

187.  មបើ

មយើ ង

ជាតួដចក្បឋមរបស់

បឋមនឹង 174

ម ោះ

។ ដតមយើ ងមាន សមីការដ្យូផង់ |លឹម សុ វណ្ណវិចត្ិ រ

ដចក្ោច់

ម ើ យដចក្មិនោច់

មទ មប្រោះ


ដូមចនោះ

ក្៏ដចក្មិនោច់

ដដរ។ ដូមចនោះ

តាមទំ ក្់ ទំនង (*) មយើ ងទាញបាន

បឋមនឹង

ជាចំ នន ួ មានរាង

មប្រោះ

។ ដូមចនោះ ។ ដតវាមិនអាចមានមទ

។ ដូមចនោះសមីការគ្មានចមមលើយ។ ម ោះ

188. មបើ តាង

។ ដូមចនោះ

។ មយើ ងនឹងបងា​ាញថា

ជាចំ នន ួ គត់

ជាចំ នន ួ គត់ (មប្រោះ

ជាចំ នន ួ គត់ ។ មយើ ងមាន

ជាចំ នន ួ សនិទាន)។

មយើ ងមាន មបើ

មយើ ងទាញបាន មយើ ងទាញបាន

។ មបើ

ម ោះសមភាព (*) មិនមានមទ មពល

ម ើ យចំ មរោះ

មយើ ងទាញបាន

។ ចំ មរោះ

។ ចមមលើយរបស់ សមីការគឺ

។ មបើ

ម ោះមយើ ងមាន

ដូមចនោះ

វ ិសមភាពមនោះមិនអាចព ិតមទ មបើ

។ ដូមចនោះ មយើ ងទាញបានចមមលើយសមីការ

ជាសរុបសមីការមានចមមលើយ 189. ក្) ប្គប់ ចំ នន ួ គត់ មប្រោះ មបើ

។ សុ ទដធ តមផទៀងផ្ទទត់ ទាង ំ អស់ មលើក្ដលងដតក្រណី

មយើ ងយក្ ។ មយើ ងនឹងបងា​ាញថា

។ មបើ

មយើ ងយក្

មចញ។ និង

មិនមផទៀងផ្ទទត់ មទ។ មយើ ងសនាតផទយ ុ ថា មានចំ នន ួ គត់

ដដល ទំ ក្់ ទំនងមនោះបងា​ាញថា ប្គប់ ចំ នន ួ បឋមដដលដចក្ មឃើ ញថា មបើ

ម ោះ ។ ដូមចនោះវាប្តូវដត

ោច់ សុ ទដធ តដចក្

ម ើ យ មបើ

ោច់ ទាំងអស់ ។ មយើ ង

ម ោះ

។ លឹម សុ វណ្ណវិចត្ិ រ| សមីការដ្យូផង់

175


តាង

ជាចំ នន ួ បឋម ដដលដចក្ោច់

ដូមចនោះដចក្ោច់

ដដរ។ តាង

។ មយើ ង

មាន

ប ទ ប់ មក្មទៀត មោយដឹងថា ប្គប់ ចំ នន ួ បឋម ដដលដចក្ោច់ ម ោះមយើ ងទាញបានថា

ជាព ុ គុណនន

សុ ទដធ តដចក្

ោច់ ដដរ ម ើ យ

ដតវាមិនអាចមប្រោះ

។ ដូមចនោះ

មិនមផទៀងផ្ទទត់ ។ ខ) សមីការមានឫសងាយ

។ សនាតថា

ជាចំ នន ួ គត់ ដដល

តាមរមបៀបប្ស្យបញ្ជជក្់ ដច ូ ក្នង ុ សំ នួរក្) មយើ ងទាញបានថា ដូមចនោះ

ដដល

ម ើ យថា

ដចក្ោច់

។ សមីការមៅជា

ចំ មរោះ

មយើ ងមាន

ចំ មរោះ

មយើ ងទាញបាន

ចំ មរោះ

មយើ ងទាញបាន

មិនអាច។ មិនអាច។ ។ មយើ ងអាចមផទៀងផ្ទទត់ ថាវាជាចមមលើយរបស់ សមី

ការ។ ដូមចនោះសមីការមានចមមលើយ 190. តាង មយើ ងទាញបានថា

និង ដចក្ោច់

។ ដដល ដូមចនោះ

និង

ជាចំ នន ួ គត់ បឋមនឹងគ្មន។ សមីការមៅជា ចំ មរោះចំ នន ួ គត់

។ សមីការបំ ដលងមៅជា

ប្តូវដតជាចំ នន ួ គត់ ម ើ យមោយវាជាចំ នន ួ វ ិជជមាន

តំនលរបស់ 176

តូចជាង

ោច់ ខាត មពល

សមីការដ្យូផង់ |លឹម សុ វណ្ណវិចត្ិ រ

។ ដូមចនោះ មបើ

ម ោះ


ម ើ យសមីការ (*) មៅជា ជាសមីការដឺមប្ក្ទី២មធៀបនឹង

ដដលមាន

មប្រោះ

។ ដូមចនោះ ក្រណីមនោះ គ្មានចមមលើយ។

ដូមចនោះមៅសល់ ក្រណី ចំ មរោះ

និង

មិនអាចជាចំ នន ួ កាមរបានមទ

មទៀត។

មយើ ងមាន

ដូមចនោះ មយើ ងទទួលបានចមមលើយព ីរគឺ មបើ

មយើ ងទាញបាន

ដូមចនោះ

។ មយើ ងទទួលបានចមមលើយព ីរមទៀតគឺ

សរុបសមីការមានចមមលើយគឺ

ជាតួដចក្រួមរបស់

191. មបើ ដចក្ោច់

ម ើយ

និង

ម ោះ

ជាតួដចក្រួមរបស់

គ្មន។ ដូចគ្មនមយើ ងទាញបាន មបើ

និង

ដចក្ោច់ ។ តាមសមាតក្ ិ មា

។ ដូមចនោះ

។ ដូមចនោះ បឋមនឹង

បឋមនឹងគ្មនព ីរៗ។

មសសទាំងព ីរ ម ោះ មយើ ងមាន មិនអាច មប្រោះចំ នន ួ កាមរសមមូលនឹង

សនាតថា

ដូមចនោះ ដចក្ោច់

ជាចំ នន ួ គូ។ តាង

មោយ

ជាចំ នន ួ គូ ម ោះមបើ

ទាំងព ីរ

និង

និង ។ ដូមចនោះ

ម ោះ ដចក្

តាម ។ ដូមចនោះប្តូវដតមានមួយជាចំ នន ួ គូ។

។ សមីការមៅជា គូ មយើ ងទាញបាន

គូដដរ, ដតមបើ

គូទាង ំ ព ីរដូចគ្មន។ មោងវ ិញមទៀត មបើ

មសស ម ោះ

មសសដដរ។ ក្តាត

ជាតួដចក្រួមធំ បំផត ុ របស់

ដចក្ោច់ ផលបូក្និងផលសងននចំ នន ួ ទាំង២ មានន័យថា ដចក្ោច់ ោច់ មប្រោះ

បឋមនឹងគ្មន។ មានន័យថា

និង

។ ដតមោយ

លឹម សុ វណ្ណវិចត្ិ រ| សមីការដ្យូផង់

177


និង

គូទាង ំ ព ីរដូចគ្មន ម ោះមានន័យថា

។ មយើ ងទាញបាន ចំ នន ួ គត់

និង

ជា

ចំ នន ួ បឋមនឹងគ្មន។ មោយផលគុណរបស់ វាជាចំ នន ួ កាមរ ម ោះវាប្តូវដតជាចំ នន ួ កាមរទាំងព ីរ

ចំ មរោះចំ នន ួ គត់ វ ិជជមាន

បឋមរវាងគ្មន ។ មោយជំនស ួ ចូលក្នង ុ សមីការ មយើ ងទាញបាន (មប្រោះសនាតថា

ខុសគ្មន មប្រោះមបើ 192. ក្) តាង

មសសទាំងព ីរ ម ោះ

វ ិជជមាន) ។ ជាបញ្ច ប់ នឹងជាចំ នន ួ គូទាង ំ ព ីរ ដត

ជាចំ នន ួ គត់ វ ិជជមាន ដដល

និង

និង

មានលក្ាណៈគូមសស

បឋមនឹង ។ ជាចំ នន ួ កាមរ។ សនាតថា

ជាចមមលើយតូចបំ ផត ុ ដូមចនោះ ព ួក្បឋមរវាងគ្មនព ីរៗ។ តាមប្ទឹសតីបទព ីតាក្រ មយើ ងអាចរក្បាន ដដល ដដល

និង

ជាចំ នន ួ វ ិជជមាន និង បឋមនឹងគ្មន ម ើ យមានលក្ាណៈគូមសសផទយ ុ គ្មន។ ដូមចនោះប្ក្ឡានផទ

ប្តីមកាណមៅជា

។ មោយ

និង

មានលក្ាណៈគូមសសផទយ ុ គ្មន ម ោះ

និង

មសសទាំង២ ដូមចនោះបឋមនឹងគ្មនដដរ។ ដូមចនោះ ក្តាតទាំង៤សុ ទដធ តបឋមនឹងគ្មន២ៗ ម ើ យនីមួយៗ ជាចំ នន ួ កាមរ។ ដូមចនោះ មានចំ នន ួ គត់ មយើ ងមាន

និង ចំ នន ួ គត់ មសស មប្រោះ

ដដល

មសសទាំងព ីរ។ ដូមចនោះ

ជាចំ នន ួ គូ។ តាង

ម ោះ

ដូមចនោះប្តូវដតមានយងមហាចមួយក្នង ុ ចំ ម្ម

និង

ជាចំ នន ួ គូ។ មោយ

ទាំង២ ម ោះ មានដតមួយបុម្ណោះក្នង ុ ចំ ម្ម

និង

ដដលជាចំ នន ួ គូ។

មបើ

ជាចំ នន ួ គូម ោះ

មោយ

178

និង

បឋមនឹងគ្មន ម ោះ មាន

សមីការដ្យូផង់ |លឹម សុ វណ្ណវិចត្ិ រ

ដដល

និង

មសស


c  d  4s 2 ; c  d  2r 2 (*) ដូមចនោះ ប្តីធាតុ

ជាចមមលើយមួយមទៀតដដរ ដដល

។ ផទយ ុ ព ីការសនាត ដដលថា

តូច

បំ ផត ុ ។ ដូមចនោះសមីការគ្មានចំ មលើយ។ ខ) តាង

ជាចំ នន ួ គត់ វ ិជជមានោច់ ខាត ដដល

តាង

និង

។ មយើ ងមាន

ដូមចនោះ X , Y និង Z ជារងាែស់ ប្ជុងរបស់ ប្តីមកាណដក្ងមួយ ដដលមានប្ក្លានផទជាចំ នន ួ កាមរ។ តាមសំ នួ រ ក្) ក្រណីមនោះអាចមៅរួចលុ ោះប្តាដត

និង

។ ដត

ដូមចនោះ

ចំ ម្ទគ្មានចមមលើយ។ 193. សនាតថា ប្តីធាតុដបបម ោះមាន។ ក្នង ុ ចំ ម្មចមមលើយទាំងម ោះ មយើ ងយក្

ដដល

ឬក្៏ជាចមមលើយដដលតូចជាងមគបងអស់។ មៅក្នង ុ លក្ាខណឌអស់ មនោះ តាមប្ទឹសតី បទត្រធា ី រុ ពតា ី ករ ដូមចនោះ

ម ោះមគមានចំ នន ួ គត់ និង ចំ នន ួ គត់

ចំ នន ួ មសស (មប្រោះ

ដដលបឋមនឹងគ្មន ដដល

មានតួដចក្រួមធំ បំផត ុ មសាើ

។ មោងវ ិញមទៀត

ជា

ជាចំ នន ួ គូ) ម ើ យដូមចនោះ តាមប្ទឹសតីបទដដដល មគមានចំ នន ួ បឋមនឹងគ្មន

ដដល

។ មបើ ោច់ ។ ដត

បឋមនឹង

ដូមចនោះ

ជាតួដចក្រួមធំ បំផត ុ រវាង ។ ដូមចនោះ

និង

ម ោះ

ដចក្

បឋមរវាងគ្មនព ីរៗ ម ើ យ

ជាចំ នន ួ កាមរ។ ដូមចនោះ ចំ នន ួ នីមួយៗប្តូវដតជាចំ នន ួ កាមរ មានន័យថា

លឹម សុ វណ្ណវិចត្ិ រ| សមីការដ្យូផង់

179


មយើ ងទាញបានថា

ជាឬសរបស់ សមីការដដរ ដដល

។ ដូមចនោះវាផទយ ុ ព ីការសនាត ដដលថា

ជាចមមលើយវ ិជជមានដដលតូចជាងមគ។ ដូមចនោះសមី

ការគ្មានចមមលើយគត់ វ ិជជមានមទ។ 194. ចំ ម្ទមនោះសមមូលនឹង ការក្ំណត់ ប្គប់ ចំ ណុចដដលមានក្ូអរមោមនជាចំ នន ួ សនិទាន ឋិ ត មៅមលើរងែង់មួយដដលមានកាំរងាែស់ មួយឯក្តា។ មៅមលើរងែង់មនោះ មយើ ងយក្ចំ ណុច ក្ូអរមោមនជាចំ នន ួ សនិទាន។ មយើ ងគូសប ទ ត់ ប្តង់ ចំ ណុច

មួយ(មិនឈរ)កាត់ តាម

ដដលមាន

។ ប ទ ត់ មនោះកាត់ រងែង់

មួយមទៀត។

តាមព ិតមៅចំ ណុច មានក្ូអរមោមនជាចំ នន ួ សនិទាន លុ ោះប្តាដត មមគុណប្បាប់ ទិសរបស់ ប ទ ត់ មនោះជា ចំ នន ួ សនិទាន។ មប្រោះ មបើ ចំ ណុចប្បសព ែរវាង

មិនដមនជាប ទ ត់ ឈរមទ សមីការប ទ ត់

ជាមួយរងែង់មផទៀងផ្ទទត់

សមីការមនោះមានឫសមួយមសាើ

។ ផលបូក្ឫសទាំង២របស់ សមីការ មសាើ

មានតនមល

អរមោមន

180

មានរាង

ក្ំណត់ មោយ

សមីការដ្យូផង់ |លឹម សុ វណ្ណវិចត្ិ រ

។ ឫសមួយមទៀត


ចំ នន ួ ព ីរមនោះជាចំ នន ួ សនិទានមបើ

ជាចំ នន ួ សនិទានដដរ។

ប្ាសមក្វ ិញ មយើ ងមឃើ ញថា មបើចំណុច

និង

មានក្ូអរមោមនជាចំ នន ួ សនិទាន ម ោះប ទ ត់

ជាមមគុណប្បាប់ ទិសជាចំ នន ួ សនិទាន។ ដូមចនោះ មយើ ងបានបងា​ាញថា ប្គប់ ចមមលើយសនិទាន មប្ៅព ី ក្ំណត់ មោយ

195. មយើ ងមឃើ ញថា

និង

ជាចមមលើយមួយ។ តាង

ជាចំ នន ួ សនិទាន ដដល

។ សមីការមៅជា ចមមលើយរបស់ សមីការមនោះគឺ

និង

។ តនមល

ប្តូវគ្មន

។ ជាបញ្ច ប់

ចមមលើយរបស់ សមីការគឺ

ចំ មរោះប្គប់

ជាចំ នន ួ សនិទាន។

196. មយើ ងនឹងបងា​ាញថា សមីការមនោះគ្មានឫសមទ។ មយើ ងដឹងថា

មយើ ងសរមសរ

មោយ

។ ដូមចនោះ

មៅក្នង ុ មគ្មល២

។ ដូមចនោះ

បូក្សមភាពទាំងអស់ មនោះបញ្ចូ លគ្មន មយើ ងទាញបាន លឹម សុ វណ្ណវិចត្ិ រ| សមីការដ្យូផង់

181


សមីការមនោះមិនអាចមានចមមលើយមទមប្រោះ

លុ ោះប្តាដត

197.  មក្ើតមានមបើ

ដដល

និងប្ាសមក្វ ិញ ម ើ យក្រណីមនោះ

និងប្ាសមក្វ ិញ។ ដូមចនោះ សមីការដដលឱ្យមានឫស លុ ោះប្តាដត ។ វ ិសមភាពមនោះមានចមមលើយ

198. មយើ ងមាន

ស្ក្

មតងមួយៗ មយើ ងមឃើ ញថា មានដត

ផ្ទទត់ ។ ដូមចនោះ ឫសសមីការគឺ

ដតបុម្ណោះដដលមផទៀង ។ ប្ាសមក្វ ិញ មយើ ងយក្ចមមលើយទាំងមនោះ

មៅមផទៀងផ្ទទត់ ស្រម ង ើ វ ិញក្នង ុ សមីការ មយើ ងនឹងមឃើ ញថា ឫសទាំងមនោះជាចមមលើយព ិតប្បាក្ដ។ 199. មយើ ងមាន

ចំ មរោះប្គប់ ចំ នន ួ ព ិត

។ បូក្អងគនង ិ អងគននសមីការ មយើ ងទាញ

បាន ជំនស ួ សមីការមប្កាយមនោះចូលក្នង ុ សមីការទាំង៣ដដលឱ្យ

ដូចគ្មនមយើ ងទាញបាន 200.  មយើ ងមាន 182

សមីការដ្យូផង់ |លឹម សុ វណ្ណវិចត្ិ រ

។ ដូចគ្មន


មបើ

ម ោះ ក្តាតផលគុណខាងមលើខុសគ្មនទាំងអស់ ។ ដូមចនោះមយើ ងមានទាំងអស់ ៥ក្តាត។

មយើ ងមាន គត់

មានន័យថា

អាចបំ ដបក្បានជាផលគុណននយងមប្ចើនចំ នន ួ

មផេងគ្មន។ ដូមចនោះ វាមិនអាចមសាើ គ្មនមទ។

មបើ

ក្មនាមខាងមលើមៅជា

ម ើ យវាក្៏មិនអាចមសាើ

201. ចំ នន ួ គត់ វ ិជជមាន

មានចំ នន ួ

សុ ទដធ តខុសគ្មនទាំងអស់ ម ើ យមានចំ នន ួ សរុប ។ មយើ ងមាន

មប្រោះ

ដដរ។

។ ចំ នន ួ គត់ វ ិជជមាន ។ សំ ណំ ុ

មានធាតុសរុបចំ នន ួ

។ ធាតុនម ី ួយៗននសំ ណំ ុ មនោះ មានតនមល ធំមិនមលើសព ី

មទ ដូមចនោះវាប្តូវមានយងមហាច្ស់ ធាតុព ីរដដលមានតនមល មសាើ គ្មន។ មោយ អស់ និង

សុ ទដធ តខុសគ្មនទាំង

សុ ទដធ តខុសគ្មនទាំងអស់ ដដរ ម ោះ ធាតុព ីរដដលមសាើ គ្មនម ោះ ប្តូវដតជា

្មួយ។ ដូមចនោះ

្មួយ និង

សែុី ត

ដដល

មផទៀងផ្ទទត់ សមា តក្ ិ មា។ មយើ ងមាន

។ មបើ

ម ោះ

ការ មបើ

។ មោយ

ដូមចនោះសមី

គ្មានចមមលើយ។ ម ោះ

សមីការ

គ្មានចមមលើយ

ដដរ។ ដូមចនោះក្រណី

សមីការគ្មានចមមលើយ។ ដូមចនោះម ើ យវាប្តូវដត

202. មយើ ងមាន មោងវ ិញមទៀត មយើ ងមាន

មប្រោះ

។ ដូមចនោះ

ព ីមប្រោះ ជាព ុ គុណនន

មិនដមនជាព ុ គុណនន

មទ ដូមចនោះ

។ មោយ និង

មសស ម ោះ មិនអាចជាព

លឹម សុ វណ្ណវិចត្ិ រ| សមីការដ្យូផង់

183


ុ គុណនន

ទាំងព ីរបានមទ។ មយើ ងទាញបានថា មានមួយក្នង ុ ចំ ម្មចំ នន ួ ព ីរមនោះ ដចក្ោច់ នឹង

។ ដត

ដូមចនោះ មានដត

ជាព ុ គុណ នន

ដូមចនោះ

មបើ

ម ោះ

។ មយើ ងមានវ ិសមភាពមួយមផេងមទៀត

ប្តូវដតដចក្ោច់

សុ ទដធ តជាចំ នន ួ មសសទាំងព ីរ ម ោះ តាមរមបៀបដូចគ្មន មយើ ងបងា​ាញថា

ដូមចនោះ

មានន័យថា និង

និង

ប្តូវដតមានរាង

។ មោយ

និង

បឋមនឹងគ្មន។

បឋមនឹងគ្មន តាមរយៈ

។ ជាបញ្ច ប់

ដចក្ោច់

ម ើ យបឋមនឹង

និង នឹង

។ ដូមចនោះ

203. មបើ

ម ោះ

មបើ

ជាចំ នន ួ បឋមមសស។ ចំ មរោះចំ នន ួ មសស

ម ោះ

សំ មណើព ិត។

ដចក្ោច់

មយើ ងមាន

មបើ

ម ោះ

ក្៏ដចក្ោច់

ោច់ នឹង

។ មោយ

ដចក្ោច់

ដដរ។ ដូមចនោះ

ដចក្

ដូមចនោះ

មានន័យថា

មប្រោះ

ជាចំ នន ួ បឋម។ ដត

មិន

ដមនជាចំ នន ួ សែ័ យគុណមទ។ ដូមចនោះ សំ មណើព ិត។ 204. មយើ ងតាង

ជាគូតនមល ដដលមផទៀងផ្ទទត់ លក្ាខណឌ។ មបើ

បុម្ណោះ ដដលមផទៀងផ្ទទត់ លក្ាខណឌ។ ឥលូ វមយើ ងសនាតថា មោយ តាង 184

ជាគូតនមលដដលមផទៀងផ្ទទត់ លក្ាខណឌ។ ម ោះប្តូវដត

។ ដូមចនោះ សមីការដ្យូផង់ |លឹម សុ វណ្ណវិចត្ិ រ

ម ើយ

ម ោះមានដត

និង


ដូមចនោះ

ក្៏ជាគូតនមលដដលមផទៀងផ្ទទត់ លក្ាខណឌដដរ។ ដូមចនោះ មយើ ងទាញបានគូ

ដដលមផទៀង

ផ្ទទត់ មាន

(គូប ទ ប់ មក្មទៀត គឺ របស់

មិនយក្មប្រោះមានតំនលធំ ជាង

គឺ

)។ ម ើ យតនមល ធំបំផត ុ

មតើមយើ ងមធែដ ើ ច ូ មមតច មទើបដឹងថាគ្មានគូតនមល មានគូតនមល

មផេងមទៀត មប្ៅព ីគូតនមល ខាងមលើមនោះ។ សនាតថា

មផេងមទៀត ដដលមផទៀងផ្ទទត់ លក្ាខណឌដដរ។ សនាត ។ មយើ ងអាចបងា​ាញថា

មបើមាន

។ មយើ ងតាង

ក្៏ជាគូតនមល ដដលមផទៀងផ្ទទត់ លក្ាខណឌដដរ។ មានន័យថា

មផទៀងផ្ទទត់ ម ោះ

ក្៏មផទៀងផ្ទទត់ ដដរ។ ដូមចនោះមានប្ ត គូតនមល

ដដលមផទៀងផ្ទទត់ លក្ាខណឌ នឹងមានតនមល កាន់ ដតតូចមៅៗ ម ើ យប្តូវចប់ មៅប្តឹម គូ មានដត គូចមមលើយ

និង

ដតបុម្ណោះ។ ដូមចនោះ មានន័យថា មិនអាចមានគូចមមលើយ ្មប្ៅអំ ព ី

ដដលបានមរៀបរាប់ ខាងមលើមទ។ ជាតនមល តច ូ បំ ផត ុ ដដលមយើ ងប្តូវក្ំណត់ ។ មយើ ងមាន

205. តាង ។ ដូមចនោះ ។ ដូមចនោះមយើ ងទាញបាន ដូមចនោះ

។ ដតគូតនមល

ដចក្មិនោច់ នឹង ឬក្៏

(មពល

ផង។ ដូចគ្មនដដរ

។ ក្រណី

ដចក្មិនោច់ នឹង

មិនអាចមៅមក្ើតមទ មប្រោះ

)។

206.  មយើ ងមាន

ជាចំ នន ួ មសស។ ដូមចនោះប្តូវមានមួយក្នង ុ ចំ ម្ម

និង មួយមទៀតមសស។ មោយ មយើ ងសនាតថា

ផង និងនឹង

និង

និង

គូ

សុ ទដធ តជាចំ នន ួ បឋម ម ោះប្តូវដតមានមួយមសាើ ។ ជាដំ បូង

។ ប ទ ប់ មក្មទៀត

ជាចំ នន ួ បឋមមសស ដដល

លឹម សុ វណ្ណវិចត្ិ រ| សមីការដ្យូផង់

185


មបើ

ជាចំ នន ួ មសស ម ោះ

ដូមចនោះ

ជាព ុ គុណនន

វាមិនអាចជា

ដដលជាចំ នន ួ មសសធំ ជាង

មទ។ មយើ ងទាញបានថា

ក្រណី

:

ជាចំ នន ួ គូ។ តាង

ជាកាមរននចំ នន ួ មសស ដូមចនោះ វាសមមូលនឹង ។ តនមល

ក្រណី

: ផលគុណ

សែ័ យគុណនន មទ។ ដូមចនោះ

និង

ចំ នន ួ ព ីរ ដដលខុសគ្មន

ដូមចនោះ

ដដល មបើ

ជាចំ នន ួ គត់ ។

ម ោះ

ចំ មរោះប្គប់

ដូមចនោះ ក្តាតនីមួយៗប្តូវដតជា

ឯក្តា មិនមានអែី មប្ៅដតព ី និង ។

និង

និង

ដូមចនោះសមីការមានចមមលើយ 207.  មោយ

។ ដូមចនោះ

មនោះមយើ ងមិនយក្ មប្រោះ

ជាសែ័ យគុណនន

ដដរ។ ចំ នន ួ សែ័ យគុណនន

ោច់ ខាត។ ដូមចនោះ

ជាសែុី តនព ែនត តាង និង

និង

ជាផលសងរួមរបស់ សែុី ត

និង

មរៀងគ្មន។

ជាសែុី តមថរ។

មគមាន

និង

ជាចមមលើយរបស់ សមីការ

ដូមចនោះ ឌ ីសប្គីមីណង់

ប្តូវដតជាចំ នន ួ គត់ កាមរ ចំ មរោះប្គប់ ចំ នន ួ គត់

។ មយើ ងមានសមភាព ដដល

មិនអាស្ស័ យនឹង

។ ដូមចនោះមបើ

ចំ មរោះ

ធំ ប្គប់ ប្គ្មន់ ្

មួយ ម ោះមយើ ងមានវ ិសមភាព ដតមោយ

ប្តូវដតជាចំ នន ួ គត់ កាមរ ចំ មរោះប្គប់

ដូមចនោះ

។ ដូមចនោះសមីការខាងមលើ មានឫសមសាើ នង ឹ ឬ

186

(មទាោះធំ ប្គប់ ប្គ្មន់ រ ឺមិនប្គប់ ប្គ្មន់ ) ម ោះ

សមីការដ្យូផង់ |លឹម សុ វណ្ណវិចត្ិ រ

។ ដូមចនោះ ចំ មរោះប្គប់

មយើ ងមាន


208. សនាតថាផទយ ុ ថា មាន

និង

អងគខាងស្តំប្តូវដតដចក្ោច់ នឹង មោយ

និង

ជាចំ នន ួ បឋមនឹងគ្មន ដដល

និង នឹង ។

ជាចំ នន ួ បឋមនឹងគ្មន ដូមចនោះ

សុ ទដធ តជាចំ នន ួ មសស ម ោះ មោយ

និង

ប្តូវដតដចក្ោច់

និង

ប្តូវដតដចក្ោច់

។ មោយ

ប្តូវដតជាចំ នន ួ មសសដដរ។ ដូមចនោះ

ជាចំ នន ួ មសស ម ោះ

ដូមចនោះ មិនព ិត

។ ដូមចនោះ

។ ដតផទយ ុ ព ី (*)។ ដូមចនោះការសនាត

សមីការគ្មានឫសសនិទាន។

209. មយើ ងមាន

។ សនាតថា

ចំ មរោះតនមលខុសៗគ្មន នន

។ ដូមចនោះ ដដលក្នង ុ ម ោះ ព ុ ធា

មានមមគុណជាចំ នន ួ គត់ ។ សនាតថា មានចំ នន ួ គត់

ដដល

ដូមចនោះ មោយក្តាត មហាច្ស់ ក្តាតបុម្ណោះ គឺ

មានតនមលខុសគ្មនទាំងអស់ ម ោះ

ប្តូវដតអាចបំ ដបក្ជាផលគុណននយង

ចំ នន ួ គត់ ខុសគ្មនដដរ។ ដតវាមិនអាច មប្រោះ ជាចំ នន ួ បឋម អាចបំ ដបក្បាន ដតព ីរ ។

លឹម សុ វណ្ណវិចត្ិ រ| សមីការដ្យូផង់

187


188

សមីការដ្យូផង់ |លឹម សុ វណ្ណវិចត្ិ រ


ឯកសារដ ម ើ

ស ៀវសៅស េះដកស្ ង់សេញពីស ៀវសៅខាងសរោមស េះ 1. Pierre Bornsztein, Xavier Caruso, Pierre Nolin, Mehdi Tibouchi, Inégalité Cours d’Arithmétique, 2004 2. David A. Santos, Number Theory for Mathematical Contests, 2005 3. D.O. Shklarsky, N.N. Chentzov, I. M. Yaglom, The USSR Olympiad Problem Book, Dover Publications, INC. New York, 1993. 4. Dusan Djukic, Vladimir Jankovic, Ivan Matic, Nikola Petrovic, The IMO Compendium, Springer, 2006

189


កម្មងលំហាត់ គណិតវិទ្យា, កម្មិតវិទ្យាល័យ, ភាគ ១- នពវន ត ដោយ លឹម សុ វណណវិចិម្ត

ដសៀវដៅកំណណលំហាត់ ណផែកនពវន ត ណែលរួមមានកំណណលំហាត់ ដលើបនសំ តកកវទ្យ ិ ា និងម្ទ្យឹសីដ នន ចំនន ួ ណែលនិយាយអំពីភាពណចកោច់ តួណចករួមធំបផ ំ ត ុ ពហុ គុណរួមតូចបំផត ុ ចំនន ួ បឋម សមីការមានឫសជាចំនន ួ គត់ ។ល។ ដសៀវដៅដនេះម្បជុដុំ ោយលំហាត់ ងាយ និងពិបាកចំនន ួ ម្បណហលជា២០០លំហាត់ ែកម្សង់ដចញពី ការម្បឡងសិ សសពូណកដៅម្បដទ្យសនានាជុវុំ ញ ិ ពិភពដោក។ ដបើអែកចង់កា​ាយជាសិ សសពូណកមាែក់ ដសៀវដៅមួយកាលដនេះ ជាដសៀវដៅណែល អែ កគួរណតមាន៕


Issuu converts static files into: digital portfolios, online yearbooks, online catalogs, digital photo albums and more. Sign up and create your flipbook.